You are on page 1of 92

Book 2

2010 Self-Assessment
Examination
Preferred Responses, Discussions and References
2010 Self-Assessment Examination
Overview
This examination is offered as a convenient • Differentiate and apply the range of non-
self-education exercise by the American operative and surgical treatment options
Society for Surgery of the Hand and to deal with upper extremity problems that
is available to the medical profession, present to a hand surgeon’s practice.
especially those health care professionals • Develop informed, evidenced-based,
focusing on the care of the hand and upper guided clinical decision practice
extremity. Designed to assist the physician options in the treatment of upper
in reviewing basic principles of hand care, extremity disorders.
the exam also helps to keep the physician
• Evaluate, by self-assessment, basic
abreast of new developments and concepts
cognitive abilities and clinical skills
within the specialty. needed by hand surgeons to successfully
The examination covers diagnostic and complete the maintenance of certification.
therapeutic problems, both surgical and
non-operative, basic science knowledge,
and fundamental principles of hand surgery. Timelines
The interpretation of the illustrative material June 11, 2010
(clinical photographs and radiographs) is an Answer sheets due back to the ASSH Office.
integral part of this examination.
July 30, 2010
The 2010 examination consists of the Individual scores, Book 2, containing preferred
Question Book (Book 1) containing 200 responses, discussions, and references, and
questions, an answer sheet, evaluation, an evaluation mailed to participants.
and the Discussion Book (Book 2) with
Please note: late submissions received
preferred responses, discussion, and
at the central office after June 11, 2010,
literature references.
cannot be scored and will not be eligible
The American Society for Surgery of for CME credit.
the Hand guarantees anonymity of
examination results. Remember to record
your examination number, as the scoring Accreditation
will be by the number and not by name. The American Society for Surgery of the Hand
Please keep a copy of your answer sheet is accredited by the Accreditation Council
for your record; originals cannot be for Continuing Medical Education to provide
returned. It is recommended that answers continuing medical education for physicians.
sheets be mailed via overnight or second-
day delivery so that the receipt of the score
sheets can be verified. Self-Assessment in MOC™
Credits earned by completing this Self-
Assessment Examination can be applied to
Objectives the scored and recorded self-assessment
• Detect and differentiate injury requirements as mandated by the ABOS
mechanisms, pathologies and applied Maintenance of Certification (MOC™)
anatomy associated with upper extremity process and will count toward the CME
conditions treated by hand surgeons. credit requirement as well.

2010 Self-Assessment Examination | 1


Credit Hours Disclosures
The ASSH designates this educational activity for a maximum • Kyle D. Bickel, MD, FACS – Ownership interests in Pacific
of 20.00 AMA PRA Category 1 Credits™. Physicians should Heights Surgery Center, LLC
only claim credit commensurate with the extent of their • David T. Netscher, MD – Deputy Editor of the Journal of
participation in the activity. Hand Surgery (published by Elsevier)
Answer sheets must be received by ASSH Central Office by • Jay F. Pomerance, MD – Paid Consultant for Wyeth
June 11, 2010, to be eligible for Category 1 CME credit. Pharmaceuticals and for Gerson Lehrman LLC
• David B. Siegel, MD – Ownership interests in Tucson

2010 Self-Assessment Surgery Center, LLC


• Andrew L. Terrono, MD – Ownership interests in Boston
Examination Committee Outpatient Surgical Services
◆ John M. Bednar, MD
Committee Chair
Kyle Bickel, MD, FACS Disclaimer
◆ Louis W. Catalano, MD The material presented in this continuing medical education
◆ Jonathan Cheng, MD program is being made available by the American Society
◆ Fred G. Corley, MD for Surgery of the Hand for educational purposes only. This
◆ Seth D. Dodds, MD material is not intended to represent the only, or necessarily
◆ Scott G. Edwards, MD the best, methods or procedures appropriate for the medical
◆ Harry A. Hoyen, MD situation discussed. Rather, this educational material is
intended to present an approach, view, statement, or opinion
◆ Richard S. Idler, MD
of the authors or presenters, which may be helpful, or of
◆ Howard M. Matsuba, MD
interest, to other practitioners.
◆ Kimberly K. Mezera, MD
David T. Netscher, MD The examinees agree to participate in this medical education
◆ Jerome W. Oakey, MD program sponsored by ASSH with full knowledge and
◆ Zubin J. Panthaki, MD awareness that they waive any claim that they may have
◆ Miguel A. Pirela-Cruz, MD against ASSH for reliance on any information presented in
Jay F. Pomerance, MD this examination.
◆ Gregory Rafijah, MD
◆ Lance A. Rettig, MD
◆ Marco Rizzo, MD Confidentiality
◆ Devesh Sharma, MD The Self-Assessment Exam is the sole property of the
◆ Russell A. Shatford, MD, MPH, MSEd American Society for Surgery of the Hand (ASSH) and
David B. Siegel, MD scores are confidential. Individual results will only be shared
Andrew L. Terrono, MD with the test-taker. Each participant is assigned a number
◆ Eric P. Thorson, MD and no names are used in the scoring process to further
◆ Raymond Wittstadt, MD ensure confidentiality.
◆ Montri D. Wongworawat, MD
◆ David S. Zelouf, MD
◆ Ephraim M. Zinberg, MD
◆ Bridget Treccia
ASSH Staff Liaison

◆ Nothing of value
The authors preceded by a ◆ have indicated that they have
not received anything of value in the form of: research or
institutional support, stock or stock options, equipment or
services, paid travel, royalties or as a consultant or employee
of a commercial company or institution related directly or
indirectly to the subject of the program.

2 | American Society for Surgery of the Hand


1. A 56 year-old man presented with an enlarging, 2. A 22 year-old man presents with a painful, swollen,
minimally tender, diffuse mass on the flexor surface stiff ring finger proximal interphalangeal joint (PIP)
of his right palm. He had suffered a laceration 5 days after an axial loading injury. Total active
while cleaning his boat 9 months earlier, which range of motion measures 25-50°. Radiographs are
he allowed to heal secondarily. Hand examination shown in Figures 1 and 2. Stable closed reduction
revealed thickening of the subcutaneous tissues on could not be maintained with a dorsal block splint.
the flexor surface of the palm, minimal tenderness, Appropriate treatment of this injury consists of:
no erythema or increased temperature, and limited
A. Closed reduction and K-wire fixation of fracture fragment
digital flexion. Surgical exploration was undertaken,
with findings depicted in Figure 1. In addition to B. Open reduction and excision of fracture fragment
thorough debridement of the involved structures, C. Closed reduction and dynamic external fixation
appropriate treatment consists of: D. Open reduction, internal fixation of fracture fragment
A. Continuous sterile saline irrigation via an indwelling E. PIP joint implant arthroplasty
catheter for 72 hours
Preferred Response: C
B. Postoperative external beam radiation
Discussion: PIP joint fracture-dislocations are relatively common
C. Postoperative brachytherapy radiation
injuries resulting from axial loading on the PIP joint, with
D. Antibiotic therapy with trimethoprim-sulfamethoxazole disruption of the volar plate insertion on the middle phalanx and
and rifampin for 6-12 months resultant dorsal instability. Appropriate treatment requires
E. Oral prednisone and rheumatology consultation restoration of a stable volar buttress of the middle phalangeal
base to restore articular stability. In this case, the volar fragment
Preferred Response: D
is quite small and is not amenable to internal fixation. Excision of
Discussion: Atypical mycobacterial infections of the hand and the volar fragment will worsen the instability. Use of a dynamic
upper extremity present as indolent, progressive soft-tissue external fixator, which provides distal traction and ligamentotaxis,
infections, most often involving the flexor tenosynovium. provides restoration of a concentric arc of rotation around
The average time to accurate diagnosis and treatment is a the proximal phalangeal head and reduction of the fracture
year or more after inoculation. The original injury occurred fragment (Figures 3 & 4). This device also allows for immediate
on a boat which suggests, and final cultures confirmed, range of motion to prevent excessive stiffness during healing.
that this infection was caused by Mycobacterium marinum.
References:
These infections can affect both immunocompetent
1. Ruland RT, Hogan CJ, Cannon DL, Slade JF. Use of dynamic
and immunocompromised patients. Confirmation of the
distraction external fixation for unstable fracture-dislocations of
infectious agent is often delayed, as cultures grow slowly, the proximal interphalangeal joint. J Hand Surg 2008;33A:19-25.
and can take several weeks after surgical debridement. 2. Ellis SJ, Cheng R, Prokopis P, Chetboun A, Wolfe S, et
Effective treatment, therefore, requires an understanding of al. Treatment of proximal interphalangeal dorsal fracture-
the pathophysiology and a high index of suspicion, leading dislocation injuries with dynamic external fixation: A pins and
to thorough debridement of infectious material and initiation rubber band system. J Hand Surg 2007;32A:1242-50.
of appropriate antimicrobial therapy in a timely fashion.
References:
1. Kozin SH, Bishop AT. Atypical mycobacterium infections of the
upper extremity. J Hand Surg 1994;19A:480-7.
2. Bhatty MA, Turner DPJ, Chamberlain ST. Mycobacterium
marinum hand infection: Case reports and review of the
literature. Br J Plas Surg 2000;53:161-5.

Figure 1 Figure 2

Figure 1 Figure 3 Figure 4

2010 Self-Assessment Examination | 3


3. A 35 year-old man suffered a deep glass laceration
to the medial forearm and presented 6 weeks after
the injury with absence of ulnar nerve sensation and
loss of ulnar intrinsic function in the hand. Findings
of wound exploration are shown in Figure 1. Resection
to healthy axons on either side of the injury
produced a nerve gap of 5cm, despite transposition.
Appropriate management of the injury involves:
A. Primary repair of the nerve ends with the elbow and
wrist in full flexion to reduce tension
B. Sural nerve grafting across the gap
C. Repair with a synthetic nerve conduit
D. Tagging of the nerve ends for later repair or grafting
Figure 1
E. No repair of the nerve and tendon transfers to restore
intrinsic function
Preferred Response: B
Discussion: Several options are now available for repair
of nerve lacerations in the upper extremity. Primary repair
is optimal when excessive tension can be avoided. In this
case, the need for maximal flexion of the elbow and wrist
to achieve primary repair is suboptimal. Nerve conduits
have been shown in some series to be as effective as
autologous nerve grafting in short nerve defects of up to
2-3 cm. The 5 cm nerve defect in this case exceeds the gap
limitations of nerve conduits, and autologous nerve grafting
is indicated (Figure 2). Tagging and delayed repair is not
indicated in this case, as the injury is clean and should not
evolve substantially over time, and the proximal location of Figure 2
the defect requires early repair to minimize functional motor
loss in the extremity. Tendon transfers may be necessary to
augment any loss of intrinsic function that may occur from 4. A 17 year-old male football player presents with
the proximal injury, but early repair should still be performed a swollen, tender ring finger and inability to flex
to restore sensory function to the hand and proximal the distal interphalangeal joint (DIP) joint of his
extrinsic motor function. dominant hand. The injury occurred from a missed
References: tackle 5 weeks earlier. Physical examination reveals
1. Lohmeyer JA, Siemers F, Machens H-G, Mailander P. The tenderness over the middle phalanx. He can actively
clinical use of artificial nerve conduits for digital nerve repair: flex the PIP joint without pain. Radiographs are
a prospective cohort study and literature review. J Reconstr normal. Appropriate management of the injury is:
Microsurg 2009;25:55-61.
2. Battiston B, Geuna S, Ferrero M, Tos P. Nerve repair by A. Reinsertion of the avulsed FDP tendon into the
means of tubulization: literature review and personal clinical distal phalanx
experience comparing biological and synthetic conduits for B. Primary tendon grafting and pulley reconstruction
sensory nerve repair. Microsurgery 2005;25:258-267.
C. Two-stage FDP tendon reconstruction with silicone
rod insertion and delayed grafting
D. Conversion to a one-tendon finger
E. Fusion of the DIP joint in slight flexion
Preferred Response: A
Discussion: Avulsion of the FDP tendon from the distal
phalanx most often involves the ring finger in athletes.
Leddy and Packer described 3 types of injuries: Type I
involves retraction of the tendon into the palm; Type II
involves retraction to the level of the PIP joint; Type III
involves little or no proximal retraction. Type I injuries,

4 | American Society for Surgery of the Hand


with an empty digital flexor sheath, require early exploration Discussion: The use of laminar matrix or dermal substitute
and repair, generally within 10 days to prevent shortening can be helpful in providing coverage for these wounds. They
and scarring of the proximal musculotendinous unit. can allow for tendon gliding in the absence of a paratenon.
Radiographs may or may not show associated avulsion Upon ensuring adherence to the wound, the dermal
fractures. McCallister, et al. showed that radiographic template can be skin grafted. They are resistant to wound
findings may be misleading, and they recommended early contracture and can therefore be appropriately covered with
exploration and repair within 10 days for all FDP avulsions. a split thickness skin graft.
True Type III injuries (Figure 1) can be successfully
Because of the absence of paratenon, the use of a split
repaired for up to six weeks after injury. Primary or
thickness or full thickness skin graft alone will likely lead to
secondary tendon grafting is useful in delayed treatment of
tendon adhesions and subsequent stiffness. Split-thickness
longstanding avulsion injuries with proximal contraction and
grafts also undergo more secondary contraction during
scarring of the musculotendinous unit. Fusion of the DIP
healing than full-thickness grafts, minimizing the resultant
joint is an option when no useful proximal motor remains
surface area of the wounds. The remaining answers are not
and the DIP joint hyperextends.
necessary for coverage of a dermal template in this case.
References:
Once an adequate vascular wound bed with subcutaneous
1. Leddy JP, Packer JW. Avulsion of the profundus tendon
granulation tissue over joint and tendon structures has been
insertion in athletes. J Hand Surg 1977;2:66-69.
restored, free grafts are appropriate wound coverage. If
2. McCallister WV, Ambrose HC, Katolik LI, Trumble TE.
Comparison of pullout button versus suture anchor for zone I exposure of relatively avascular structures remains, transfer
flexor tendon repair. J Hand Surg 2006;31A:246-251. of vascular tissue with either local pedicled or distant
microvascular flaps is necessary for adequate wound
coverage. Split-thickness skin grafts are more suitable for
relatively large surface areas, and they revascularize more
easily than full-thickness grafts.
References:
1. Carothers JT, Brigman BE, Lawson RD, Rizzo M. Stacking of a
dermal regeneration template for reconstruction of a soft-tissue
defect after tumor excision from the palm of the hand: A case
report. J Hand Surg 2005;30A:1322-1326.
2. Wolter TP, Noah EM, Pallua N. The use of Integra® in an upper
extremity avulsion injury. Br J Plast Surg 2005;58:416-418.

Question 4, Figure 1

5. A 50 year-old woman involved in an auto rollover


accident experienced avulsion injuries of the dorsal
soft tissues of the index thru small fingers (Figure 1).
After initial debridement and application of a dermal
substitute for 2 weeks, the extensor tendons are Figure 1
covered with healthy granulation tissue and no
joint structures are exposed (Figure 2). Appropriate
wound coverage at this stage consists of:
A. Split-thickness skin grafts
B. Full-thickness skin grafts
C. Dorsal metacarpal island flaps
D. Abdominal flap
E. Free fasciocutaneous flap
Preferred Response: A
Figure 2

2010 Self-Assessment Examination | 5


6. A 7 year-old boy suffered an amputation of the upper 7. A patient presents one week after injury with a painful
extremity at the level of the elbow (Figure 1). The right wrist after a fall onto his outstretched hand while
amputation is a high-energy avulsion. The patient snowboarding. Radiographs on the day of injury
is stabilized quickly and transported to the hospital were normal. Examination reveals tenderness in the
within 2 hours of the injury. The appropriate treatment anatomic snuffbox and with loaded wrist extension.
after debridement is: There is mild dorsal wrist swelling. Repeat plain
radiographs are again normal. A scaphoid fracture
A. Bone shortening and primary closure of the stump
is suspected based on the clinical presentation. The
B. Bone shortening and split-thickness skin grafting best imaging study to demonstrate an abnormality
C. No bone shortening and free microvascular latissimus providing support for the clinical diagnosis is:
flap coverage of the proximal stump
A. Tomographic radiographs
D. Bone shortening and replantation of the distal extremity
B. Ultrasound
E. Bone shortening and upper extremity transplantation
C. Bone scan
Preferred Response: D D. CT scan
Discussion: Replantation of proximal upper extremity E. MRI
amputations in high-energy avulsion injuries is generally
Preferred Response: E
contraindicated in adults. The zone of injury is wide, with a
high risk of ischemic failure after replantation, and functional Discussion: Non-displaced scaphoid fractures may
restoration of the distal part is usually poor. Replantation present with clinical symptoms and findings but normal
criteria in children, however, are more liberal and functional radiographs. Confirmatory imaging studies are, therefore,
results are generally superior to adults when replantation helpful in appropriate treatment and avoiding under-treatment
is successful. When technically feasible and medically of fractures. Of the commonly-used imaging studies, MRI has
not contraindicated by comorbidities, replantation of even been shown to have the highest sensitivity, specificity, and
proximal injuries is therefore recommended in children. accuracy, at 98%, 99%, and 96% respectively, with very high
positive and negative predictive values (Figure 1). CT scanning
References:
has been shown to have similar sensitivity, specificity, and
1. Kim JYS, Brown RJ, Jones NF. Pediatric upper extremity
accuracy, but with lower predictive values. CT and tomography
replantation. Clin Plast Surg 2005;32:1-7.
also require substantial exposure to radiation, which is avoided
2. Buncke GM, Buntic RF, Romeo O. Pediatric mutilating hand
injuries. Hand Clin 2003;19:121-131. with MRI. Bone scan and ultrasound both have been shown
to have low specificity and positive predictive values,
making their use less reliable.
References:
1. Ring D, Lozano-Calderon S. Imaging for suspected scaphoid
fracture. J Hand Surg 2008;33A:954-957.
2. Bhat M, McCarthy M, Davis TR, Oni JA, Dawson S. MRI and
plain radiography in the assessment of displaced fractures
of the waist of the carpal scaphoid. J Bone Joint Surg
2004;86B:705-713.

Figure 1

Figure 1

6 | American Society for Surgery of the Hand


8. The use of 500mg daily of vitamin C has been shown 9. A wrist fusion is planned in a cerebral palsy patient
in two recent randomized clinical trials to reduce the with spastic flexion deformities of the wrist and
incidence of what complication? good hand function. Digital flexor tightness can be
improved by what concomitant procedure?
A. Postoperative hematoma
B. Complex regional pain syndrome after distal A. Profundus to superficialis transfers
radius fractures B. Radial shortening osteotomy
C. Non-union after operative treatment of scaphoid fractures C. Proximal row carpectomy
D. Synovitis and tendon rupture after internal fixation of D. Superficialis to EDC transfers
radius fractures E. Superficialis tenotomy
E. Postoperative wound infection after internal fixation of
Preferred Response: C
radius fractures
Discussion: Wrist fusion has been used successfully to
Preferred Response: B
treat severe flexion deformity of the wrist in patients with
Discussion: Two randomized, placebo-controlled clinical cerebral palsy. Proximal row carpectomy concomitant
trials have shown a reduced incidence of complex regional with wrist arthrodesis aids in positioning of the wrist
pain syndrome in patients with distal radius fractures treated and correcting severe flexion. By changing the relative
with oral vitamin C. The effect appeared to be dose-related, length of the digital musculotendinous units, proximal
with optimal effect reaching a plateau at 500mg per day, row carpectomy also aids in rebalancing severe flexion
and with no statistically-significant benefit shown at 200mg tightness in the digital flexors. FCU and FCR tenotomies
per day. This is now a current recommendation in the AAOS are commonly performed with wrist fusion to assist in
guideline on the treatment of distal radius fractures. releasing the tight wrist in spastic wrist deformities, but they
do not alter the digital tightness. Superficialis to profundus
References:
tendon transfers are useful adjuncts to relieve digital flexion
1. Zollinger PE, Tuinebreijer WE, Breederveld RS, Kreis RW. Can
tightness in patient’s with poor hand function but not
vitamin C prevent complex regional pain syndrome in patients
with wrist fractures? (A randomized, controlled, multicenter profundus to superficialis transfers, superficialis to EDC
dose-response study). J Bone Joint Surg 2007;89A:1424-1431. transfers, or superficialis tenotomy.
2. Shah AS, Verma MK, Jebsen PJL. Use of oral vitamin C after References:
fractures of the distal radius. J Hand Surg 2009;34A:1736-1738.
1. Van Heest AE, Strothman D. Wrist arthrodesis in cerebral palsy.
3. American Academy of Orthopaedic Surgeons: Clinical Practice J Hand Surg 2009;34A:1216-1224.
Guidelines. Guideline on Treatment of Distal Radius Fractures.
2009, AAOS.org. 2. Pomerance JF, Keenan ME. Correction of severe spastic
flexion contractures in the nonfunctional hand. J Hand Surg
1996;21A:828-833.
3. Rayan GM, Young BT. Arthrodesis of the spastic wrist. J Hand
Surg 1998;24A:944-952.

2010 Self-Assessment Examination | 7


10. A 56 year-old man presents with a one-year history
of progressive ulnar-sided wrist pain. A coronal
MRI of his wrist is shown in Figure 1. He underwent
arthroscopic exploration which demonstrated
lunatotriquetral ligament insufficiency, a large
central triangular fibrocartilage tear, and lunate
chondromalacia (Figure 2). In addition to debridement
of the TFC lesion and the chondromalacia, what
treatment is most appropriate?
A. Ulnar shortening osteotomy
B. Lunotriquetral fusion
Figure 1
C. Hemiresection interposition arthroplasty
D. Proximal row carpectomy
E. Vascularized bone grafting of the lunate
Preferred Response: A
Discussion: The patient in this case has ulnocarpal
impaction syndrome. The MRI changes demonstrate edema
in the lunate. While similar changes can be seen with
Kienbock’s Disease, the eccentric location of the changes
in the lunate is more suggestive of impaction. In addition,
the MRI demonstrates positive ulnar variance. Both an ulnar Figure 2
shortening osteotomy and arthroscopic wafer resection
can be successful in alleviating the impaction. However,
the ulnar shortening may be more appropriate in cases
of L-T instability or hypermobility, as it has been shown 11. Which factor is associated with a poor level of
experimentally to increase ulnotriquetral and ulnolunate patient satisfaction after carpal tunnel release?
strains effectively helping to stabilize and reduce motion A. Diabetes mellitus
between carpal bones. Ulnar shortening osteotomy,
B. Thyroid disorders
however, has a higher incidence of complications related
to non-union and hardware problems than arthroscopic C. Symptom duration
ulnar shortening. Lunotriquetral fusion and proximal row D. Symptom severity
carpectomy do not unload the ulnar wrist and are not useful E. Age > 70 years
in treating ulnar impaction. Hemiresection interposition
arthroplasty is used for a painfully arthritic distal radioulnar Preferred Response: D
joint. Vascularized bone grafting of the lunate is a Discussion: Symptom severity has been shown to
treatment option for pre-collapse Kienbock’s disease and negatively impact the degree of symptom relief after
inappropriate in this case. surgery for carpal tunnel syndrome. Diabetes mellitus,
References: thyroid disease, duration, and advanced age have not been
1. Sachar K. Ulnar-sided wrist pain: Evaluation and treatment of shown to negatively impact symptom relief after carpal
triangular fibrocartilage complex tears. Ulnocarpal impaction tunnel release, although they may all be implicated in the
syndrome, and lunotriquetral ligament tears. J Hand Surg development of carpal tunnel symptoms.
2008;33A:1669-79.
References:
2. Gupta R, Bingenheimer E, Fornalski S, McGarry MH, Osterman
AL. The effect of ulnar shortening on lunate and triquetrum 1. Mallick A, Clarke M, Kershaw CJ. Comparing the outcome of a
motion – a cadaveric study. Clin Biomech 2005;20:839-45. carpal tunnel decompression at 2 weeks and 6 months. J Hand
Surg 2007;32A:1154-8.
3. Constantine KJ, Tomaino MM, Herndon JH, Sotereanos DG.
Comparison of ulnar shortening osteotomy and the wafer 2. Leit ME, Weiser RW, Tomaino MM. Patient-reported outcome
resection procedure as a treatment for ulnar impaction after carpal tunnel release for advanced disease: a prospective
syndrome. J Hand Surg 2000;25A:55-60. and longitudinal assessment in patients older than age 70.
J Hand Surg 2004;29A:379-83.
3. Thomsen NOB, Cederlund R, Rosen I, Bjork J, Dahlin LB.
Clinical outcomes of surgical release among diabetic patients
with carpal tunnel syndrome: prospective follow-up with
matched controls. J Hand Surg 2009;34A:1177-87.

8 | American Society for Surgery of the Hand


12. An 18 year-old high school pitcher has developed 14. Estrogen is a critical hormone in bone physiology
medial-sided elbow pain while throwing his fastball. because it:
MRI reveals a medial (ulnar) collateral ligament
A. Stimulates osteoclast activity
strain without a tear. What muscle group(s) function
as dynamic stabilizers for this injury: B. Suppresses osteoclast activity
C. Stimulates osteoblast activity
A. Flexor carpi ulnaris
D. Suppresses osteoblast activity
B. Flexor digitorum profundus
E. Stimulates vitamin D activity
C. Biceps and brachialis
D. Triceps Preferred Response: B
E. Supinator Discussion: Estrogen is an important mediator of bone
resorption because it suppresses osteoclast activity in both
Preferred Response: A
men and women. After menopause, the decrease in estrogen
Disscussion: The flexor-pronator mass dynamically leads to a rapid loss of bone mineral density in women.
stabilizes the elbow against valgus torque, therefore Testosterone helps osteoblasts form bone. Vitamin D assists
protecting the medial (ulnar) collateral ligament. A well-done with calcium absorption in the gut and decreases calcium
cadaveric study by Park & Ahmad revealed that the FCU is excretion. Sunlight converts Vitamin D to its active form.
the primary stabilizer and the FDS a secondary stabilizer
References:
against valgus stress.
1. Dell RM, Greene D, Anderson D, Williams K. Osteoporosis
References: disease management: What every orthopaedic surgeon should
1. Park MC, Ahmad CS. Dynamic contributions of the flexor- know. J Bone Joint Surg Am 2009;91 Suppl 6:79-86.
pronator mass to elbow valgus stability. J Bone Joint Surg Am 2. Ahlberg HG, Johnell O, Turner CH, Rannevik G, Karlsson MK.
2004 Oct;86A(10):2268-74. Bone loss and bone size after menopause. N Engl J Med
2. Maloney MD, Mohr KJ, El Attrache NS. Elbow injuries 2003;349:327-34.
in the throwing athlete. Difficult diagnoses and surgical
complications. Clin Sports Med 1999 Oct;18(4):795-809.
15. The scaphoid shift test, as described by Watson,
requires:
13. When using a 6-hole locking plate to treat a
A. Ulnar-directed pressure on the scaphoid
comminuted radial shaft fracture in a patient with
severe osteoporosis, what screw configuration will B. Volarly-directed pressure on the scaphoid proximal pole
reduce the risk of peri-implant fracture and give the C. Volarly-directed pressure on the lunate
strongest construct: D. Dorsally-directed pressure on the lunate
A. All locking screws E. Dorsally-directed pressure on the scaphoid distal pole
B. All nonlocking screws Preferred Response: E
C. Only 2 locked screws furthest from the fracture
Discussion: The scaphoid shift test is used to diagnose
D. Alternating locked and unlocked screws scapholunate instability. The test starts with the wrist in
E. A nonlocked screw at each end of the plate ulnar deviation. A dorsally directed force is applied to
the scaphoid distal pole and then the wrist is moved into
Preferred Response: E
radial deviation. This maneuver dislocates or subluxes the
Discussion: A recent biomechanical study published scaphoid dorsally over the dorsal ridge of the distal radius.
by Bottlang et al in JBJS highlighted a few important If scapholunate dissociation is present, a painful “clunk” will
biomechanical points regarding locking plates. By occur when pressure is released and the scaphoid reduces.
substituting the locked end screw with a conventional Comparison to the contralateral side is important. A bilateral
end screw, binding strength was increased by 40%. In non painful “clunk” is a negative test. The “clunk” must be
addition, stress concentration at the end of the plate was painful for the test to be considered positive.
reduced when a conventional end screw was used, thereby
References:
lessening the risk of a fracture at the end of the plate.
1. Watson HK, Ashmead D IV, Makhlouf MV. Examination of the
References: scaphoid. J Hand Surg 1988;13A:657-60.
1. Bottlang M, Doornink J, Byrd GD, Fitzpatrick DC, Madey SM. 2. Easterling KJ, Wolfe SW. Scaphoid shift in the uninjured wrist.
A nonlocking end screw can decrease fracture risk caused by J Hand Surg 1994;19A:604-606.
locked plating in the osteoporotic diaphysis. J Bone Joint Surg
Am 2009;91:620-7.
2. Egol KA, Kubiak EN, Fulkerson E, Kummer FJ, Koval KJ.
Biomechanics of locked plates and screws. J Orthop Trauma
2004;18:488-93.

2010 Self-Assessment Examination | 9


16. A 22 year-old male injures the ulnar aspect of his volar shear fractures that the lunate facet be reduced and
thumb MP joint in a fall while skiing. Surgery should well fixed (or well buttressed).
be recommended if:
References:
A. The MP joint is lax in extension, but stable in 30° flexion 1. Harness NG, Jupiter JB, Orbay, JL, Raskin KB, Fernandez DL. Loss
B. The MP joint is lax in 30° flexion, but stable in extension of fixation of the volar lunate facet fragment in fractures of the
distal part of the radius. J Bone Joint Surg 2004;86A:1900-1908.
C. A tender mass is palpable proximal to the MP joint 2. Rozental TD and Blazar PE. Functional Outcome and
D. The MP joint is lax in hyperextension, but an Complications after Volar Plating for Dorsally Displaced, Unstable
endpoint exists Fractures of the Distal Radius. J Hand Surg 2006;31A:359-365.
3. McKay SD, MacDermid JC, Roth JH, Richards RS. Assessment
E. A nondisplaced proximal phalangeal fracture is present
of complications of distal radius fractures and development of a
Preferred Response: C complication checklist. J Hand Surg 2001;26A:916-922.

Discussion: Skiers thumb, or rupture of the ulnar collateral


ligament of the thumb MP joint, is a common injury. The
indication for surgery is instability of the MP joint (greater
than 35° angulation or greater than 20° difference from the
uninjured side) without an endpoint during valgus stress
testing in extension and 30° flexion. Testing the MP joint in
extension evaluates the integrity of the accessory collateral
ligament/volar plate while testing the MP joint in 30° flexion
tests the proper collateral ligament. The joint must be
unstable in both extension and flexion to warrant surgery.
A Stener’s lesion is created by retraction of the collateral
ligament proximal to the adductor aponeurosis. If palpable,
the ligament is completely torn and retracted, necessitating PA film day of injury Lateral film day PA film 2 days post op
surgery. In the study by Heyman et al, a Stener’s lesion was of injury
present in 87% of surgical cases. A fracture of the ulnar
proximal base of the proximal phalanx may be present in
these injuries. However, surgery is indicated only if the joint
is unstable regardless of the non-displaced fracture.
References:
1. Heyman P, Gelberman RH, Duncan K, Hipp JA. Injuries of the
ulnar collateral ligament of the thumb metacarpophalangeal
joint. Clin Orthop 1993;292:165-171.
2. Malik AK, Morris T, Chou D, Sorene E, Taylor E. Clinical testing
of ulnar collateral ligament injuries of the thumb. J Hand Surg
2009 June;34B(3):363-6.

17. Recurrent volar subluxation, as seen in the figures, Lateral film 2 days PA film 14 days post op Lateral Film 14 days
can occur after ORIF of a volar Barton (shear) post op post op
fracture of the distal radius if:
A. Volar tilt is not restored 18. An 80 year-old woman falls at home and injures her
B. Radial height (inclination) is not restored neck. Afterwards, she has profound upper extremity
weakness, numbness, and mild lower extremity
C. Radial length is not restored
weakness. Her expectations for recovery should be:
D. The scaphoid facet is not reduced/fixed
A. No recovery in the upper or lower extremities
E. The lunate facet is not reduced/fixed
B. Full recovery in both upper and lower extremities
Preferred Response: E
C. Fair recovery of both upper and lower extremities
Discussion: Harness et al in JBJS presented a series of 7 D. Full recovery in the upper extremities, but no recovery
cases of repeat carpal displacement following ORIF of a in the lower extremities
volar shear fracture of the distal radius (as demonstrated
E. Full recovery in the upper extremities, with only fair
in the figure). The cause is inadequate fixation and/or re-
recovery in the lower extremities
displacement of the lunate facet allowing the carpus to
displace volarly. It is critical when performing ORIF of these Preferred Response: C

10 | American Society for Surgery of the Hand


Discussion: Central cord syndrome is the most common References:
incomplete spinal cord injury and is becoming more 1. Shoesmith CL, Strong MJ. Amyotrophic lateral sclerosis.
common as the age of the population increases. With a Can Fam Physician 2006;52:1563-1569.
hyperextension injury to the cervical spine in a patient 2. Ross MA, Miller RG, Berchert L, Parry G, Barohn RJ, Armon
with preexisting cervical spondylosis (most commonly C, et al. Toward earlier diagnosis of amyotrophic lateral
elderly patients), the spinal cord is compressed anteriorly sclerosis: revised criteria. rhCNTF ALS Study Group. Neurology
1998;50(3):768-72.
by osteophytes and posteriorly by hypertrophied
ligaments. The central grey matter is most affected by the
compression resulting in significantly greater loss of motor
function in the upper compared to the lower extremities. 20. A 45 year-old male cut his right hand while fishing
Also, variable upper extremity sensory loss occurs. Only fair three months ago. Since the injury, he has noted
recovery can be expected in central cord syndrome. an enlarging painful wound. Examination revealed
a tender, indurated ulcer with an erythematous
References: base, but no fluctuance at the site of his laceration.
1. Marar BC. The pattern of neurological damage as an aid to the Intraoperative cultures are negative at five days.
diagnosis of the mechanism in cervical-spine injuries. J Bone Initial antibiotic treatment is:
Joint Surg 1974;56A:1648-1654.
2. Maroon JC, et al. Central cord syndrome. Clin Neurosurg A. Penicillin
1991;37:612-621. B. Clarithromycin (Biaxin)
3. McKinley W, Santos K, Meade M, Brooke K. Incidence and
outcomes of spinal cord injury clinical syndromes. J Spinal C. Vancomycin
Cord Med 2007;30(3):215-224. D. Gentamycin
E. Amoxicillin + Clavulinic Acid (Augmentin)
Preferred Response: B
19. A 49 year-old male presents with gradually
worsening weakness in both hands over the past Discussion: Mycobacteria marinum infections result from
3 months. Physical examination reveals mild injuries sustained while exposed to water (e.g. cleaning
atrophy of the interosseous and thenar musculature a fish tank). The resultant infection is indolent and often
bilaterally. Tongue fasciculations were noted as well. results in chronic nodules or ulcers. Lymphangitis,
His most likely diagnosis is: tenosynovitis, and osteomyelitis may also occur. The
organism does not grow well in temperatures above
A. Guillain-Barré Syndrome
32°C, so the laboratory must ensure proper incubation
B. Charcot Marie Tooth Syndrome temperatures in order to obtain growth after a biopsy.
C. Multiple Sclerosis
Surgical debridement may be necessary, especially for
D. Myasthenia Gravis more severe, or deeper, infections. Although multiple
E. Amyotrophic Lateral Sclerosis (ALS) drug therapy is recommended for more severe infections,
the superficial ulcer in this patient can be managed with
Preferred Response: E
single drug therapy. The most effective antibiotics against
Discussion: ALS is a progressive degenerative disorder mycobateria marinum include clarithromycin, sulfonamides,
caused by loss of brain stem and spinal motor neurons. rifampin, ethambutol, and trimethopriam sulfamethoxazole.
As a result, painless weakness and atrophy results with
References:
minimal or no sensory loss. The first symptoms of ALS
include weakness in the upper and lower extremities, 1. Harris DM, Keating MR. Mycobacterium marinum: Current
recommended pharmacologic therapy. J Hand Surg
dysphagia, dysarthria, and head drop. Tongue 2009;34A:1734-35.
fasciculations are nearly pathognomonic of ALS.
2. Lewis F, Marsh B, von Reyn C. Fish tank exposure and
Guillain-Barré Syndrome is an ascending motor paralysis cutaneous infections due to Mycobacterium marinum:
tuberculin skin testing, treatment, and prevention. Clin Infect
that starts in the lower extremites and can ascend to reach
Dis 2003;37:390-397.
the upper extremities and the diaphragm. Charcot Marie
Tooth Syndrome is a demyelinating neuropathy that mainly
involves the lower extremities, but can result in intrinsic
wasting of the hands. Multiple Sclerosis is a demyelinating
disease that produces muscle weakness, ataxia, sensory
changes, visual problems, dysarthria and unstable moods.
Myasthenia Gravis is an autoimmune disorder in which
antibodies block the acetylcholine receptors of the post
synaptic neuromuscular junction. Muscles become weak
and easily fatigue. The muscles about the eye and mouth
are particularly susceptible.

2010 Self-Assessment Examination | 11


21. A 35 year-old technician at an industrial aerospace 22. A 53 year-old butcher suffered a 3cm transverse
plant presents 2 hours after an industrial knife laceration to the radial border of the wrist
fluoroscopy accident. He has sustained an 2 years ago. He has numbness of the dorsal first
estimated 200 Gy of radiation to the left hand and webspace, significant pain, and hypersensitivity just
forearm. There is erythema and pain in the exposed proximal to the laceration. There is a Tinel’s sign
areas. What is the most likely clinical outcome for at this location. His neuropathic pain is most likely
this patient in the absence of treatment? attributed to which of the following reactions to
external stimuli:
A. Swelling followed by resolution of erythema
B. Mild blistering and reepithelialization A. Synaptic impulse conduction
C. Extensive scarring and ulceration B. Ephaptic impulse conduction
D. Necrosis of the entire hand and forearm C. Fast axonal transport
E. Pseudomonas soft tissue infection D. Slow axonal transport
E. Retrograde axonal transport
Preferred Response: C
Preferred Response: B
Discussion: Low energy injuries (0.5-10 Gy) are
characterized by transient erythema, pruritis, and epilation Discussion: This patient has a painful neuroma of the
(hair loss), which present after 1-3 weeks. superficial radial nerve, which is characterized by numbness
in the first webspace, Tinel’s over the neuroma, and pain and
Moderate energy injuries (10-20 Gy) result in transient
hypersensitivity at the neuroma. Pain and hypersensitivity
early erythema which returns after 1-2 weeks, with typical
results from spontaneous nerve impulses from the nerve
desquamation and impaired motion from swelling. These
fibers within the neuroma and ephaptic conduction,
injuries recover with time and local wound care.
(“cross-talk”) which allows magnification of normal and
Radiation injuries in the modern era show a preponderance spontaneous nerve impulses by inducing aberrant nerve
of upper extremity involvement, high energy exposure, and impulses within uninvolved adjacent nerve fibers.
tissue and digit loss, due to the industrial use of fluoroscopy
Synaptic conduction of nerve action potentials is normal
and electron accelerators and the proliferation of nuclear
transmission which occurs between the axons and
reactors. Recommendations, based on experience with
dendrites of first, second, and third order sensory neurons
case series and animal studies, are for early excision and
at nerve synapses. An action potential is generated at a
coverage of high energy wounds on the lower end of the
peripheral sensory receptor in response to a stimulus, and
energy spectrum (>20Gy). High energy wounds show initial
is transmitted along a sensory nerve fiber back to the first
clinical signs of erythema and pain within 30 minutes, with
order sensory neurons in the dorsal root ganglion. Synaptic
evolution to extensive scarring and ulceration based on
conduction allows transmission of this impulse to the
energy and depth of penetration over a period of weeks
second order sensory neuron in the spinal cord. There is
to months. Commonly, isolated digit necrosis requires
normally no peripheral transmission between nerve fibers in
ray amputation. Many reports indicate Pseudomonas
the absence of a synapse.
involvement prior to amputation, but it is not clear whether
necrosis or soft tissue infection occurred first. Fast and retrograde axonal transport represents physiologic
processes within the peripheral nerve, which are not
Referneces:
particularly contributory to neuropathic pain.
1. Chambers JA, Long JN. Radiation injury and the hand surgeon.
J Hand Surg 2008;33A:601-611. Although the radial nerve is anatomically more susceptible
2. Krizek T, Ariyan S. Severe acute radiation injuries of the hands: to neuropathic pain than other distal cutaneous nerves,
report of two cases. Plast Reconstr Surg 1973;51:14-22. compression neuropathy superimposed upon a
3. Milanov NO, Shilov BL, Tjulenev AV. Surgical treatment of traumatic neuroma is not likely to be the primary cause
radiation injuries of the hand. Plast Reconstr Surg 1993;92:294. of neuropathic pain.
References:
1. Mackinnon SE. Evaluation and treatment of the painful
neuroma. Tech Hand Up Extrem Surg 1997;1:195-212.
2. Dellon AL, Mackinnon SE. Susceptibility of the superficial
sensory branch of the radial nerve to form painful neuromas.
J Hand Surg 1984;9B:42-45.

12 | American Society for Surgery of the Hand


23. A healthy 52 year-old has a radial arterial line placed 24. Which combination of imaging device and hand
prior to undergoing pituitary adenoma resection. position provides the greatest decrease in radiation
Following surgery, he is noted to have severe dose to the patient?
ischemia of the thumb and index finger. There is
A. Mini C-arm, near image intensifier
a palpable ulnar pulse and a palpable radial pulse
proximal to the arterial cannulation site. There B. Mini C-arm, near x-ray tube
is no radial pulse or Doppler signal distal to the C. Standard C-arm, behind image intensifier
cannulation site. What is the next step? D. Standard C-arm, on magnification mode
A. Angiography E. Standard C-arm, near x-ray tube
B. Heparinization Preferred Response: A
C. Catheter-directed thrombolysis
Discussion: The mini C-arm offers significant advantages
D. Exploration and thrombectomy in terms of 5-10 fold lower radiation exposure and horizontal
E. Excision and vein grafting positioning, which allows the surgeon to stand behind
the image intensifier in the location best shielded from
Preferred Response: E
radiation scatter.
Discussion: This clinical scenario clearly points to a
As the intensity of radiation exposure follows the “inverse
cannulation-induced injury and thrombosis of the radial
square law” and varies proportionately to 1/r2, with
artery distal to the cannulation site. The radial artery most
r=distance from the radiation source, the surgeon must
likely remains patent proximal to the cannulation site, due to
consciously position the patient’s hand as far from the
an intact arterial branch providing alternative outflow from
x-ray tube and as close to the image intensifier as possible.
the artery. The thumb and index finger are ischemic despite
This position is facilitated by rotating a standard C-arm
intact ulnar artery inflow due to an incomplete palmar arch.
inverted with the tube up, and placing the image intensifier
Unless an inflow problem proximal to the site of cannulation
down for use as a working platform. With a mini C-arm,
is suspected, angiography is unnecessary and time
this position is achieved by placing the mini C-arm in a
consuming, which adds to morbidity.
horizontal position with the image intensifier toward the
Heparinization is unlikely to result in recannulation of the surgeon as indicated above.
thrombosed artery, if at all, with sufficient speed to prevent
Finally, there is some advantage to using the magnification
infarction of the affected digits. Thrombolysis with TPA
function. As the beam is collimated, scatter is decreased,
is contraindicated due to the intracranial procedure. This
and therefore the surgeon and other operating personnel
thrombotic event is most likely the result of intimal injury
are exposed to less radiation.
caused by the physical act of cannulation, and is best
treated with resection and vein grafting. Thrombectomy is Refernces:
possible, but would not address the underlying intravascular 1. Athwal GS, Bueno RA Jr, Wolfe SW. Radiation exposure in
injury and would require heparinization, which is also hand surgery: mini versus standard C-arm. J Hand Surg
contraindicated after an intracranial procedure. 2005;30A:1310-1316.
2. Giordano BD, Baumhauer JF, Morgan TL, Rechtine GR 2nd.
References: Patient and surgeon radiation exposure: Comparison of
1. Koman LA, Ruch DS, Smith BP, Smith TL. Vascular disorders: standard and mini-C-arm fluoroscopy. J Bone Joint Surg
Cannulation injuries. In: Green DP, Hotchkiss RN, Pederson 2009;91A:297-304.
WC, Wolfe SW, eds. Green’s Operative Hand Surgery. 5th ed. 3. Tremains MR, Georgiadis GM, Dennis MJ. Radiation exposure
Philadelphia: Churchill Livingstone/Elsevier, 2005;64:2289-2290. with use of the inverted-C-arm technique in upper-extremity
2. McCollum CH, Mavor E. Brachial artery injury after cardiac surgery. J Bone Joint Surg 2001;83A:674-678.
catheterization. J Vasc Surg 1986;4:355-359.

2010 Self-Assessment Examination | 13


25. A 42 year-old manual laborer presents with numbness 26. Which muscle is most likely to be involved in a mild
and tingling in the median distribution of both Volkmann’s ischemic contracture of the forearm?
hands. He is confused about his options regarding
A. Flexor pollicis longus (FPL)
endoscopic versus open carpal tunnel release. Which
of the following short term advantages of endoscopic B. Flexor carpi ulnaris (FCU)
release is supported by Level I evidence? C. Palmaris longus (PL)
A. Decreased neuropraxia D. Flexor digitorum superficialis (FDS)
B. Faster return to work E. Pronator teres (PT)
C. Greater relief of numbness Preferred Response: A
D. Less scar tenderness Discussion: Volkmann’s ischemic contracture of the forearm
E. Higher pinch and grip strength results from an untreated or prolonged ischemia from an
acute compartment syndrome. The necrotic muscle is
Preferred Response: D
replaced with fibrotic scar tissue, resulting in contractures.
Discussion: Endoscopic carpal tunnel release has become Ulnar and/or median nerve injury may be irreversible.
an accepted modality for treatment of carpal tunnel
The most vulnerable muscles have the deeper muscle
syndrome. However, open carpal tunnel release remains
bellies and include the flexor pollicis longus and the flexor
a mainstay in many hand surgery practices. The general
digitorum profundus. In milder ischemic contractures,
consensus is that long term results are comparable among
only portions of these two muscles may be involved. The
the two.
superficial muscles in the volar forearm (e.g. FCR/FCU/
A systematic review of randomized controlled trials (Level PT/PL/FDS) are damaged only with more severe and/or
I evidence) by Thoma et al. in 2004, concluded that short- prolonged compartment syndromes.
term (6-12 weeks postoperative) scar tenderness and grip/
References:
pinch strength were improved when comparing endoscopic
1. Stevanovic M, Sharpe F. Management of established
vs. open release. This study incorporated trials between
Volkmann’s contracture of the forearm in children. Hand Clin
1992 and 2000. Thoma showed increased neuropraxia 2006;22:99-111.
in endoscopic release, which is a known drawback of 2. Tsuge K. Treatment of established Volkmann’s contracture.
the endoscopic treatment; return to work was equivocal J Bone Joint Surg 1975;57A:925-929.
between the two treatments. Subsequent Level I studies
by Ferdinand and MacLean showed no difference in
pinch/grip strength between treatments; MacDermid
27. Which of the following is associated with decreased
et al. demonstrated no difference in sensory threshold,
success of use of the isolated 1,2 intercompartmental
suggesting no difference in neuropraxia; Atroshi et al.
vascularized bone graft for scaphoid nonunion?
showed no difference in symptomatic relief.
A. VISI deformity
The heterogeneity of these Level I reports suggests the
importance of variability in patient selection, surgical B. Incomplete palmar arch
technique, and assessment measures in comparing open C. Avascular necrosis
vs. endoscopic carpal tunnel release. D. Prior fixation
References: E. Humpback deformity
1. Thoma A, Veltri K, Haines T, Duku E. A meta-analysis of Preferred Response: E
randomized controlled trials comparing endoscopic and
open carpal tunnel decompression. Plast Reconstr Surg Discussion: The 1,2 intercompartmental supraretinacular
2004;114:1137. artery (1,2 ICSRA) pedicled vascularized dorsal radius
2. Ferdinand RD, MacLean JGB. Endoscopic versus open carpal bone graft has been developed extensively for treatment
tunnel release in bilateral carpal tunnel syndrome. J Bone Joint
of scaphoid nonunions and proximal pole avascular
Surg 2002;84B:375-379.
necrosis (AVN). Success rates are 67-100% when using this
3. MacDermid JC, Richards RS, Roth JH, Ross DC, King GJW.
Endoscopic versus open carpal tunnel release: A randomized technique. The blood supply of this flap is based on a distal
trial. J Hand Surg 2003;28A:475-480. branch of the radial artery at the wrist. Despite an abnormal
4. Atroshi I, Hofer M, Larsson GU, Ornstein E, Johnsson R, Allen’s test, this flap may be performed as long as the radial
Ranstam J. Open compared with 2-portal endoscopic carpal artery has a palpable pulse. Prior fixation or surgery is not a
tunnel release: A 5-year follow-up of a randomized controlled contraindication to its use.
trial. J Hand Surg 2009;34A:266-272.
5. Trumble TE, Diao E, Abrams RA, Gilbert-Anderson MM. Single- In a retrospective review of 48 cases of scaphoid nonunion
portal endoscopic carpal tunnel release compared with open and AVN treated with 1,2 ICSRA vascularized graft, 9 of 14
release: A prospective, randomized trial. J Bone Joint Surg failures were attributed to humpback deformity, indicating
2002;84A:1107-1115.

14 | American Society for Surgery of the Hand


advanced carpal collapse. In the setting of humpback The Paneva-Holevich technique of two-stage tendon
deformity, a free vascularized bone graft from the medial reconstruction (which uses the FDS tendon as a graft that
femoral condyle may be a more appropriate treatment. is sutured to the proximal FDP stump at the same time
as silicone rod placement) provides an advantage of one
References:
less tendon repair site at the second-stage reconstruction,
1. Chang MA, Bishop AT, Moran SL, Shin AY. The outcomes and
when compared with a free palmaris longus graft. However,
complications of 1,2 intercompartmental supraretinacular artery
pedicled vascularized bone grafting of scaphoid nonunions. the Paneva-Holevich technique would require sacrificing
J Hand Surg 2006;31A:387-396. the intact FDS tendon for use as a graft to reconstruct the
2. Larson AN, Bishop AT, Shin AY. Dorsal distal radius lacerated FDP tendon.
vascularized pedicled bone grafts for scaphoid nonunions.
Tech Hand Up Extrem Surg 2006;10:212-223. The recommended postoperative regimen for tendon
repairs in younger children (generally less than 10 years of
3. Jones DB Jr, Burger H, Bishop AT, Shin AY. Treatment of
scaphoid waist nonunions with an avascular proximal pole and age) is complete immobilization for four weeks. Recent work
carpal collapse. A comparison of two vascularized bone grafts. suggests that early motion is possible in this group, but
J Bone Joint Surg 2008;90A:2616-2625. immobilization remains the preferred regimen.
References:
1. Taras JS, Kaufmann RA. Flexor tendon injury: Flexor tendon
28. A 3 year-old presents 6 months after sustaining a reconstruction. In: Green DP, Hotchkiss RN, Pederson WC,
window glass midpalm laceration over the index Wolfe SW, eds. Green’s Operative Hand Surgery. 5th ed.
metacarpal, which was closed at an urgent care center. Philadelphia: Churchill Livingstone/Elsevier, 2005;7:254-272.
On exam, all joints are supple and a palmaris longus 2. Stark HH, Anderson DR, Boyes JH, et al. Bridge grafts of flexor
tendon is present. DIP joint active flexion is absent, tendons. Orthop Trans 1983;7:44.
and PIP joint active flexion is intact. Exploration reveals 3. Valenti P, Gilbert A. Two-stage flexor tendon grafting in
a zone 3 FDP laceration with a 2cm gap between the children. Hand Clin 2000;16:573-578.
tendon stumps and an intact FDS tendon. What is 4. O’Connell SJ, Moore MM, Strickland JW, Frazier GT, Dell PC.
the best reconstructive option for finger flexion? Results of zone I and zone II flexor tendon repairs in children.
J Hand Surg 1994;19A:48-52.
A. FDP excision 5. Moehrlen U, Mazzone L, Bieli C, Weber DM. Early mobilization
B. FDP primary repair after flexor tendon repair in children. Eur J Pediatr Surg
2009;19:83-86.
C. FDP reconstruction with interpositional palmaris
tendon graft
D. FDP excision, silicone rod placement
E. FDP 2-stage reconstruction with FDS tendon graft,
silicone rod placement

Preferred Response: C
Discussion: In children, many advocate for waiting until
after age 7 to reconstruct tendon injuries with delayed
presentation; specifically, in Zone 2 injuries requiring
2-stage silicone rod reconstruction, due to inability to
comply with early motion protocols. However, the tendon
injury in this case can be localized to zone 3, or midpalmar,
which is outside of the flexor tendon sheath. A tendon
defect outside of the sheath is suitable for single-stage
tendon graft repair without the use of a silicone rod, due to
decreased concern for tendon adhesions. Primary end-to- Figure 1 Figure 2
end tendon repair is not possible due to significant retraction
of the tendon stumps and degeneration of the tendon ends,
requiring freshening by sharp transection. Excising the FDP
tendon is not necessary and would create the potential
for scarring within the flexor sheath, which would impair
existing FDS excursion.

2010 Self-Assessment Examination | 15


29. A 55 year-old man presents with triggering of the A. Proximal prominence of the pretendinous cord with
right ring finger. On further questioning, he has a skin pitting
2-year history of morning stiffness of both hands, B. Lack of skin adherence between palmar crease and
which gradually improves with use. Examination palmodigital crease
shows diffuse volar swelling of the ring finger with
C. Absent proximal interphalangeal joint flexion contracture
palpable crepitant triggering, and nonpainful boggy
enlargement of the wrists bilaterally and MCP joints D. Impaired metacarpophalangeal joint passive and
of all fingers. Radiographs show bony erosion and active extension
destruction of the wrist and MCP joints. What is the E. Normal fingertip two point discrimination and
appropriate treatment for the ring finger? monofilament exam
A. Intravenous vancomycin Preferred Response: B
B. Oral clarithromycin/ethambutol Discussion: This untreated Dupuytren’s contracture fails
C. Closed catheter irrigation the Hueston “tabletop test”, which some use as a criterion
D. Open tenosynovectomy for surgical correction. The ring finger displays a typical
early Dupuytren’s presentation with pretendinous cord and
E. Release of the A1 pulley
adjacent dermal pitting, restriction of MP joint extension,
Preferred Response: D and minimal PIP joint involvement.
Discussion: This patient meets clinical criteria for a Short and Watson have indicated that the palmar skin
diagnosis of rheumatoid arthritis based on the presence of and soft tissues between the distal palmar crease and
morning stiffness, symmetric joint involvement, involvement the palmodigital crease should be examined closely prior
of 3 hand joints, MCP and wrist involvement, and erosive to surgical treatment. The presence of dense adhesion
arthritis on x-rays. between the dermis and the pretendinous cord/central cord
structure in this location precludes the presence of spiral
The triggering represents true tenosynovitis, in contrast
cords. Thus, when the skin is completely adherent to the
to the tendovaginitis present in the idiopathic trigger
longitudinal cords along this segment, the neurovascular
digit. Infectious etiologies such as suppurative flexor
bundles are not pathologically displaced to the superficial
tenosynovitis or mycobacterium marinum infection are
and midaxial position that renders them susceptible to
unlikely, given the chronicity and polyarticular involvement.
iatrogenic injury. This has implications for minimizing
Recommended first treatment is open tenosynovectomy of
iatrogenic nerve injury in open treatment, which is the
the flexor tendon within the sheath, with possible stepwise
traditional option, and percutaneous release, which is ideal
progression to ulnar superficialis slip excision and reductive
for the primary (early) palmar disease present in this finger.
flexor tenoplasty as needed to address associated
enlargement of the flexor tendons. References:
1. Short WH, Watson HK. Prediction of the spiral nerve in
Release of the A1 pulley is not recommended in rheumatoid
Dupuytren’s contracture. J Hand Surg 1982;7A:84-86.
arthritis patients due to the potential for worsening deformity.
2. Foucher G, Medina J, Malizos K. Percutaneous needle
References: fasciotomy in Dupuytren’s disease. Tech Hand Up Extrem Surg
2001;5:161-164.
1. Arnett FC, Edworthy SM, Bloch DA, McShane DJ, Fries JF,
Cooper NS, Healey LA, Kaplan SR, Liang MH, Luthra HS, et al.
The American Rheumatism Association 1987 revised criteria
for the classification of rheumatoid arthritis. Arthritis Rheum
1988;31:315-24.
2. Ryzewicz M, Wolf JM. Trigger digits: principles, management, and
complications. J Hand Surg 2006;31:135-146.
3. Ferlic DC, Clayton MC. Flexor tenosynovectomy in the
rheumatoid finger. J Hand Surg 1978;3:364-367.

30. A 64 year-old man presents with the deformity shown


in Figures 1 and 2. He is not able to flatten the palm
against a table. Which of the following physical
findings suggests that the digital neurovascular
bundles are displaced to a vulnerable superficial
position near the midaxis of the ring finger?

Figure 1 Figure 2

16 | American Society for Surgery of the Hand


31. A 46 year-old woman presents with extension Preferred Response: D
contracture of the ring and small finger MCP joints
Discussion: Congenital clasped thumb is a rare disorder
after ulnar nerve injury. She comes for surgical
where a child’s thumb displays a flexed posture at the MCP
release of the contractures. The joints are ranged
joint. This is most often caused by an abnormal extensor
following excision of the dorsal MCP joint capsule
mechanism. A hypoplastic extensor pollicis brevis tendon is
and incision of the dorsal 50% of the collateral
commonly encountered.
ligaments. During passive range of motion, full flexion
is obtained. However, when moved into extension Congenital clasped thumb was described Weckesser
there is noted to be a “jump” in the motion of the joint and Heiple in 1968 and classified into four groups. Group
prior to reaching full extension. What is the next step? one includes the cases most commonly encountered.
Conservative therapy can be successful here. Late or
A. Volar plate release
secondary changes in the intrinsic muscles and soft tissues
B. Flexor tenolysis can complicate surgical procedures. The principal defect
C. Collateral ligament reconstruction involves the extensor pollicis brevis muscle and tendon.
D. Collateral ligament release Group two includes cases in which the thumb is clasped in
the palm and the thumb extensors are weak or absent. There
E. Wound closure
are significant flexion contractures of the other digits. This is
Preferred Response: D thought to be the result of mild-to-moderate arthrogryposis.
Group three has diffuse alterations in all structures of the
Discussion: The primary goal of restoring full MCP flexion
thumb, including hypoplasia of the extensors, flexors, and
has been obtained. The “jump” noted when moving to
thenar muscles, as well as the osseous components. Group
extension indicates residual tightness of the collateral
four includes all cases, which do not fall into these other
ligaments. The collateral ligaments are snapping over the
three categories. There may be duplication of the digits and
condyles of the metacarpal head when passing from full
the extensor tendon may be abnormally distributed.
flexion to full extension, due to inadequate ligament length
and extensibility. The next step is to release the collateral Inability to extend the thumb, following splinting, can be
ligaments further by dividing the insertions to the metacarpal addressed with a tendon transfer. The flexor digitorum
head. If the problem persists, the accessory collaterals sublimus of the ring and long finger is transferred to the
should be taken down as well. This problem must be thumb extensor mechanism.
corrected because active MCP flexion may be significantly
Fixed deformities with web-space contracture may need
impaired by the physical restriction of ligament excursion.
MP joint release, flexor pollicis longus lengthening, or flexor
Volar plate release is not necessary in extension sheath release.
contractures, but intraarticular adhesions of the volar
Flexion deformities of the IP joint of the thumb are more
synovial pouch may need to be gently released. Flexor
commonly seen with congenital trigger thumb. They are
tenolysis may be necessary in these cases, but tendon
often associated with postural hyperextension of the MP
adhesions generally do not cause triggering.
joint. Inability to extend the MP joint of the thumb is rarely
References: seen in a patient with congenital trigger thumb.
1. Shin AY, Amadio PC. Stiff finger joints. In: Green DP, Hotchkiss
References:
RN, Pederson WC, Wolfe SW, eds. Green’s Operative Hand
Surgery. 5th ed. Philadelphia: Churchill Livingstone/Elsevier, 1. Lipskeir E, Weizenbluth M. Surgical treatment of the clasp
2005;11:417-438. thumb. J Hand Surg 1989;128:72-79.
2. Young VL, Weeks CR, Weeks PM. The surgical management of 2. Weckesser EC, Reed JR, Heiple KG. Congential clasp thumb.
stiff joints in the hand. Plast Reconstr Surg 1978;62:835-841. (Congential flexion-adduction deformity of the thumb) J Bone
Joint Surg 1968;50A:1417-1436.
3. Upton J. Congenital anomalies of the hand and forearm. In:
32. An 8-month-old child is seen with the thumb McCarthy JG, May JW, Littler JW. Plastic Surgery: The Hand
Part 2. Philadelphia: WB Saunders, 1990:5213-3398.
deformity in Figure 1. The MCP joint is flexed, but
supple, and easily passively extended. The child
cannot actively extend the MCP or the IP joint of the
thumb. There are no other abnormalities seen in this
child. In considering surgery for this deformity, what
common abnormality is found?
A. Thickening in the flexor pollicis longus tendon
B. Stenotic A1 pulley of the thumb
C. Absence of the flexor pollicis longus
D. Hypoplastic extensor pollicis brevis
E. Absence of the brachioradialis Figure 1

2010 Self-Assessment Examination | 17


33. The abnormality evident in the nail bed of the patient Discussion: The mechanism of development of the
seen in Figure 1 is associated with which of the chemotherapy-induced peripheral neuropathy (CIPN) is
following disorders? unknown. A painful sensory neuropathy may develop with
cisplatin, oxaliplatin, carboplatin. A mixed sensorimotor
A. Sarcoidosis
neuropathy, with or without involvement of the autonomic
B. Psoriasis nervous system, is more common with vincristine, taxol,
C. Bronchogenic carcinoma and suramin. Neurotoxicity is believed to most likely
D. Cirrhosis depend on the total cumulative dose and the type of drug
used. Patients with diabetes mellitus, alcohol or inherited
E. Arsenic toxicity
neuropathy are more susceptible to development of
Preferred Response: D chemotherapy-induced peripheral neuropathy. Gender of
the patient has no known connection to development of the
Discussion: In 1954, Terry was the first to describe opacities of
neuropathy. Use of dose-dense chemotherapy regimens, as
the nail in patients with cirrhosis. The proximal nail is affected,
well as use of multimodal chemotherapy regiment, has been
is opaque, and obscures the lunula. The discoloration
found to increase the incidence of CIPN. Stage of cancer,
usually stops 1-2 millimeters from the distal edge of the
location of cancer, as well as the type of cancer, has no
nail. The anomaly involves all nails uniformly. Arsenic toxicity
known relation to the development of CIPN.
manifests with formation of arsenic keratosis. They are a
keratotic papule or plaque usually on the lateral nail fold. They References:
are found in patients with large amounts of arsenic ingestion. 1. Quanstoff S, Hartung P. Chemotherapy-induced peripheral
Patients with bronchogenic carcinoma have clubbing of the neuropathy. J of Neurology 2002;249:9-17.
nails with loss of the normal depression at the base of the nail. 2. Malik B, Stillman M. Chemotherapy-induced peripheral
Patients with hepatitis may have palmar erythema and spider neuropathy. Cr Neurology and Neuroscience Report
angiomata. The palmar erythema is usually most prominent on 2008;8:56-65.
the thenar and hypothenar eminences. Sarcoidosis does not
cause nail change. Psoriasis is associated with nail pitting.
35. A cross finger flap is used to cover the pulp defect
References:
of the ring finger. The flap is raised from the dorsum
1. Terry, RB. White nails in hepatic cirrhosis. Lancet 1955;4:757. of the middle phalanx of the long finger. If impaired
2. Bradburn H, Chase R, Fessel J. The Hand in Systemic Disease. vascularity is encountered, which of the following
J Bone Joint Surg 1962;44A:1395-1411.
will aid in improving flap vascularity?
A. Incising Cleland’s ligament
B. Pinning the PIP joint of the ring finger in 60° of flexion
C. Pinning the PIP joint of the long finger in 60° of flexion
D. Incising the paratenon over the extensor tendon
E. Excising the paratenon over the extensor tendon
Preferred Response: A
Discussion: The cross finger flap is a random flap raised
from the dorsum of the finger at the proximal or middle
Figure 1 phalanx level. It is based on the subdermal plexus of
vessels supplied by both the dorsal digital arteries and the
proper digital arteries.
34. A patient presents to clinic complaining of
numbness and burning pain in both hands. The The main indication for the flap is the volar pulp wound, with
patient is currently being treated with vincristine for extensive soft tissue loss.
acute leukemia. Chemotherapy induced peripheral Contraindications include multiple injuries to the hand,
neuropathy is most closely related to: Berger’s and Raynaud’s disease, and Dupuytren’s
A. Cumulative dose contracture.
B. Patients gender In the case above, to improve vascularity and lengthen
C. Avoidance of drug-dense dosing the flap, one can divide Cleland’s ligament to allow more
mobility. The paratenon over the extensor tendon must be
D. Stage of cancer
preserved to allow skin graft coverage. PIP pin fixation in
E. Use of a single agent chemotherapy patients over 50 can result in significant stiffness of the
Preferred Response: A PIP joint.

18 | American Society for Surgery of the Hand


References: Discussion: The photographs show a dorsal dislocation of
1. Thomson HG, Sorokolit WT. The cross-finger flap in children: a the metacarpophalangeal joint of the small finger. In reducible
follow up study. Plast Reconstr Surg 1967;39(5):482-487. dorsal dislocations of the MP joint, the proximal phalanx
2. Barclay TL. The late results of finger-tip injuries. Br J Plast Surg is perched in a hyperextended position on the metacarpal
1955;8:38-42. head with flexion of the proximal phalangeal joint. When a
3. Porter RW. Functional assessment of transplanted skin in volar dorsal metacarpophalangeal joint dislocation is complex,
defects of the digits. J Bone Joint Surg 1968;50A:955-963. the proximal phalanx lies dorsal to the metacarpal shaft in
4. Friedrich JB, Katolik LI, Vedder NB. Soft tissue reconstruction a bayonet position. The border digits, index and small, are
of the hand. J Hand Surg 2009;34A:1148-1155. more commonly injured. Dimpling in the skin of the proximal
palmer crease is pathognomonic for a dorsal dislocation.
Closed reduction is often attempted, and simple distraction
36. Patients with pre-existing lymphedema secondary is usually unsuccessful. This can convert a reducible
to axillary node dissection that undergo a hand dislocation into an irreducible one. The closed reduction
surgical procedure have an increased rate of: maneuver for dorsal dislocations flexes the wrist and
A. Infection proximal phalangeal joint of the injured digit to relax the
flexor tendons. Then, a palmarly directed force is applied
B. Complex regional pain syndrome
to the base of the proximal phalanx in an attempt to slide
C. Prolonged stiffness the proximal phalanx and its attached volar plate over the
D. Delayed wound healing metacarpal head into a reduced position.
E. Blood loss Open reductions are either done through a dorsal or volar
Preferred Response: D approach. The volar approach endangers the radial digital
neurovascular bundle when the index MP joint is dislocated,
Discussion: Elective hand surgery in post-mastectomy and the ulnar digital neurovascular bundle when the small
lymphedema can be tolerated, but a delay in wound healing finger is dislocated. These neurovascular bundles are
has been noted in the literature. To limit post-operative placed in a precarious position by the dislocation. Through
lymphedema, sterile technique and judicious use of the a zig zag or chevron approach, the metacarpal head is
tourniquet are recommended. exposed and the A1 pulley is divided longitudinally. This
The rates of infection, regional pain syndrome, blood loss loosens the noose formed by the flexor tendon ulnarly and
and stiffness were comparable with surgery done in normal the lumbrical tendon radially. The lumbrical lies radial to the
limbs. Only a slightly prolonged healing time separated the metacarpal head. Releasing the pulley relieves the tension
post axillary node dissection group from the normal. on this noose and allows one to hyperextend the joint and
removes the volar plate, facilitating reduction.
References:
1. Smith WK, Giddins GEB. Lymphedema and hand surgery. The hand is usually splinted in a dorsal splint for 2-3 weeks,
J Hand Surg 1999;24A:138. allowing active flexion within the range of stability.
2. Gharbaoui IS, Netscher DT, Thornby J, Kessler FB. Safety of References:
upper extremity surgery after prior treatment for ipsilateral
breast cancer: results of an American Society for Surgery of the 1. Green DP, Terry GC. Complex dislocation of the
Hand membership survey and literature review. J Am Soc Surg metacarpophalangeal joint: Correlative pathological anatomy.
Hand 2005;5:232–38. J Bone Joint Surg 1973;55A:1480-1486.
2. Dihn P, et al. Metacarpophalangeal joint dislocation. J Am Acad
Orthop Surg 2009;17(5):318-324.
3. Baldwin LW, Miller DL, Lockhart LD, Evans EB.
37. Attempts at closed reduction of the small finger Metacarpalphalangeal dislocations of the fingers. A comparison
metacarpophalangeal joint seen in Figures 1 and of the pathological anatomy of the index and little finger. J Bone
2 have proven unsuccessful. A volar approach is Joint Surg 1967;49A:1587-1590.
made to the MP joint to facilitate reduction. Which
of the following structures must be divided to
successfully reduce the dislocation?
A. Volar plate
B. Sagittal band
C. Lumbrical muscle
D. A1 Pulley
E. Superficial transverse metacarpal ligament
Figure 1 Figure 2
Preferred Response: D

2010 Self-Assessment Examination | 19


38. Which of the following anatomic structures is not References:
involved in the spiral cord in Dupuytren’s disease? 1. Terkonda SP, Perdikis G. Non-melanotic skin tumors of the
upper extremity. Hand Clin 2004;20:293-301.
A. Pretendinous band
2. Zabar RIS, Cottel WI. Skin Tumors 1: non-melanoma skin
B. Spiral band tumors. Sel Read Plast Surg 2000;9:2-19.
C. Cleland’s ligaments
D. Grayson’s ligament
E. Lateral digital sheath
Preferred Response: C
Discussion: The spiral cord seen in PIP joint contractures in
Dupuytren’s disease has four origins:
1. Pretendinous band
2. Spiral band
3. Lateral digital sheath
4. Grayson’s ligament
Cleland’s ligament is not involved in the makeup of the spiral
cord. The cord in the palm usually affects the ring and small
finger and is superficial to the neurovascular bundle. Distal
to the metacarpophalangeal joint, it passes deep to the
bundle. It runs lateral to the neurovascular bundle, involving
the lateral digital sheath; then, when Grayson’s ligament is
involved, the cord is again superficial to the bundle.
References:
1. Umlas ME, Bischoff RJ, Gelberman RH. Predictors of
neurovascular displacement in hands with Dupuytren’s
contracture. J Hand Surg 1994;19B:664-666. Figure 1
2. McFarlane R. The anatomy of Dupuytren’s disease. In: Hueston
JT, Tubiana R, eds. Dupuytren’s Disease. 2nd ed. New York:
Churchill Livingstone, 1985:54-71.
40. What associated joint deformity typically occurs
with PIP flexion contractures in patients with
psoriatic arthritis?
39. A 75 year-old rancher presents with a rapidly growing
lesion on the dorsum of his dominant hand seen in A. Boutonniere deformity
Figure 1. It is nonpainful and has been present for one B. Swan neck deformity
month. Which of the following is the correct diagnosis?
C. DIP flexion contracture
A. Squamous cell carcinoma D. MCP extension contracture
B. Basal cell carcinoma E. MCP flexion contracture
C. Keratoacanthoma
Preferred Response: D
D. Pyogenic granuloma
Discussion: The PIP flexion contracture that characteristically
E. Seborrheic keratosis
occurs with psoriatic arthritis is often accompanied by an
Preferred Response: C MCP joint extension contracture. However, the DIP joint does
not typically settle into a posture of hyperextension, as in
Discussion: Keratoacanthomas are rapidly growing skin tumors
a Boutonniere deformity. The MCP extension contractures
that are typically seen on sun-exposed skin. They pathologically
are in contrast to the more typical MCP flexion contractures
resemble squamous cell carcinoma. Keratoacanthoma is
seen in Rheumatoid arthritis.
characterized by rapid growth over a few weeks to months,
followed by spontaneous resolution over 4-6 months in most References:
cases. Squamous cell carcinoma has a more insidious history 1. Feldon, Terrono, Nalebuff, Millender. Rheumatoid Arthritis.
and rarely causes pain. Basal cell carcinomas are the most In: Green DP, Hotchkiss RN, Pederson WC, Wolfe SW, eds.
common form of skin cancer. They are slow-growing with skin Green’s Operative Hand Surgery. 5th ed. Philadelphia: Churchill
Livingstone/Elsevier, 2005:2053.
atrophy, and pink-to-red skin discoloration. Ultimately, they
2. Rose JH, Belsky MR. Psoriatic arthritis in the hand. Hand Clin
ulcerate with elevated pearly borders. Seborrheic keratosis are
1989;5:137-144.
noncancerous (benign), wart-like growths on the surface of the
skin. A biopsy is necessary prior to definitive surgical treatment.

20 | American Society for Surgery of the Hand


41. The diagnosis of acute gout is best confirmed by: Discussion: Patients with long-standing nerve injury (two
years from injury) are better served by tendon transfers for
A. A nucleated cell count above 50,000 on joint aspiration
radial nerve palsy than nerve repair, as motor-end plate
B. The presence of chondrocalcinosis on plain radiographs damage tends to occur 9-12 months after the injury. This
C. Elevated sedimentation rate patient already has atrophy of his nondominant forearm
D. Elevated blood uric acid and would be well served with tendon transfers to restore
hand position for activities of daily living. Pronator teres to
E. Negative birefringence of crystals in joint fluid aspirate
ECRB, FCR to EDC, and palmaris longus to EPL tendon
Preferred Response: E transfers accomplish acceptable hand position for activities
of daily living.
Discussion: Negative birefringence of urate crystals found
on joint aspiration is a clinically accurate way to confirm the If intervention is performed prior to nine months after injury,
diagnosis of gout. The presence of negatively birefringent Mackinnon’s group has recently described nerve transfers
urate crystals does not preclude concurrent infection or from selected expendable branches of the median or ulnar
other inflammatory arthropathy (such as rheumatoid arthritis). nerve to the posterior interosseous nerve and ECRB or
Gout will often present with a slight fever, leucocytosis, ECRL for restoration of long extensor function.
elevated ESR and CRP. Blood uric acid level cannot be relied
References:
on to confirm or exclude gout. A synovial fluid analysis or
1. Bishop J, Ring D. Management of radial nerve palsy associated
tissue specimen should be examined under polarized light
with humeral shaft fracture: a decision analysis model. J Hand
microscopy for the presence of urate crystals, which should Surg 2009;34A:991-6.
be negatively birefringent (yellow when aligned parallel to 2. Bumbasirević M, Lesić A, Bumbasirević V, Cobeljić G,
axis of the red compensator inserted between the polarizer Milosević I, Atkinson HD. The management of humeral shaft
and the analyzer within the microscope). fractures with associated radial nerve palsy: a review of 117
cases. Arch Orthop Trauma Surg 2010;4:519-22.
References:
3. Mackinnon SE, Roque B, Tung TH. Median to radial nerve
1. Snaith ML. ABC of Rheumatology: Gout, Hyperuricemia, and transfer for treatment of radial nerve palsy. Case report.
Crystal Arthritis. Br Med J 1995;310:521-524. J Neurosurg 2007;107:666-71.
2. Kann SE, Jacquemin J, Stern PJ. Simulators of hand infections. 4. Lowe JB 3rd, Tung TR, Mackinnon SE. New surgical option for
AAOS Instr Course Lect 1997;46:69-82. radial nerve paralysis. Plast Reconstr Surg 2002;110:836-43.

42. A 45 year-old patient presents with the radiograph


shown in Figure 1 and a chief complaint of
weakness after sustaining a non-dominant humeral
shaft fracture. He states that he has not felt the back
of his hand, extended his fingers, or extended his
wrist since his initial injury 2 years ago. On exam, he
has extrinsic atrophy and no radial nerve function
distal to the triceps. What surgical plan is most
appropriate to offer him?
A. External neurolysis of the radial nerve and collagen
conduit treatment
B. Radial nerve grafting at the level of the fracture
C. FCR and palmaris nerve branches to posterior
interosseous nerve transfer
D. FCU to ECRB, FDS IV to EDC, and abductor digiti
minimi opponens tendon transfers Figure 1
E. Pronator teres to ECRB, FCR to EDC, and palmaris
longus to EPL tendon transfers
Preferred Response: E

2010 Self-Assessment Examination | 21


43. Interfascicular nerve repair has been shown to: While documented hand strength and function may improve
with growth in a child with this chronic neuromuscular
A. Poorly reapproximate nerve fascicle orientation
disorder, the ever-present weakness from motor axon loss
B. Decrease tension at the repair site can be demonstrated in patients with CMT type 1A using
C. Increase scarring at the repair site straightforward dynamometry. In addition, grip strength has
D. Preserve perineural blood flow been correlated with CMAP amplitude.
E. Be significantly superior to epineural repair References:
1. Burns J, Bray P, Cross LA, North KN, Ryan MM, Ouvrier RA.
Preferred Response: C
Hand involvement in children with Charcot-Marie-Tooth disease
Discussion: Levinthal et al. compared different types of type 1A. Neuromuscul Disord 2008;18(12):970-3.
nerve repair in dogs and found increased scarring and 2. Videler AJ, van Dijk JP, Beelen A, de Visser M, Nollet F,
decreased axonal ratio (presence of axons compared to van Schaik IN. Motor axon loss is associated with hand
dysfunction in Charcot-Marie-Tooth disease 1a. Neurology
connective tissue) in the interfascicular repair group with 2008;71(16):1254-60.
no significant improvement in axonal recovery, compared
to epineural repair. There was improved axonal recovery
in the perineural, fascicular repair group. Cabaud et al
45. A spiral cord in Dupuytren’s contracture will
and Kline et al found no significant differences in recovery
displace the neurovascular bundle:
results, comparing fascicular versus epineural repairs
in cats and monkeys, respectively. No clinical study has A. Distally, dorsally, and away from the midline
effectively demonstrated the superiority of one type of B. Proximally, volarly, and towards the midline
nerve coaptation (fascicular, interfascicular, epineural) over
C. Distally, volarly, and towards the midline
another in a controlled trial in humans.
D. Proximally, dorsally, and away from the midline
References:
E. Distally, volarly, and away from the midline
1. Levinthal R, Brown WJ, Rand RW. Comparison of fascicular,
interfascicular and epineural suture techniques in the repair of Preferred Response: B
simple nerve lacerations. J Neurosurg 1977;47(5):744-50.
2. Lundborg G. A 25-year perspective of peripheral nerve surgery: Discussion: One of the more troublesome anatomic
evolving neuroscientific concepts and clinical significance. variants of Dupuytren’s contracture is the spiral cord. The
J Hand Surg 2000;25A(3):391-414. spiral cord will extend from the palm away from the midline,
3. Kline DG, Hudson AR, Bratton BR. Experimental study of pushing the bundle toward the midline (Figure 1). The spiral
fascicular nerve repair with and without epineural closure. cord will also extend deep to the neurovascular bundle,
J Neurosurg 1981;54(4):513-20. pushing it volarly or superficially. Lastly, the spiral cord will
4. Orgel MG. Epineural versus perineural repair of peripheral cause a flexion contracture of the finger, pushing the bundle
nerves. Clin Plast Surg 1984;11(1):101-4.
more proximally.
5. Cabaud HE, Rodkey WG, McCarroll HR Jr, Mutz SB, Niebauer
JJ. Epineural and perineural fascicular nerve repairs: a critical References:
comparison. J Hand Surg 1976;1A(2):131-7. 1. McGrouther. Dupuytren’s Contracture. In: Green DP, Hotchkiss
RN, Pederson WC, Wolfe SW, eds. Green’s Operative Hand
Surgery. 5th ed. Philadelphia: Churchill Livingstone/Elsevier,
2005:2053.
44. Children with Charcot-Marie-Tooth disease most
2. McFarlane RM. Patterns of the diseased fascia in the fingers of
frequently present with:
Dupuytren’s contracture. Plast Reconstr Surg 1974;54:31-44.
A. Intrinsic hand weakness and poor manual dexterity
B. PIP joint extension contractures
C. Intrinsic spasticity
D. Extrinsic spasticity
E. Characteristic nail changes and collagen related
joint laxity
Preferred Response: A
Discussion: Charcot-Marie-Tooth disease is a
demyelinating neuropathy with progressive muscle
weakness and atrophy. While the foot and the leg are most
characteristically affected, many patients (children and
adults) describe hand weakness and difficulty with manual
dexterity. Intrinsic muscle atrophy is the typical etiology.
Figure 1

22 | American Society for Surgery of the Hand


46. A 62 year-old man fell four days ago. The post- 47. The best treatment of the injury shown in Figure 1 is:
reduction x-ray is shown. The best treatment for
A. Casting in 30º flexion
the injury is:
B. Casting in 90º flexion
A. Open reduction internal fixation
C. Tension banding
B. Closed reduction percutaneous pinning
D. Plating
C. Static splint
E. Excision
D. Dorsal blocking splint
Preferred Response: D
E. Extension block pinning
Discussion: Displaced olecranon fractures require internal
Preferred Response: A
fixation. Simple, stable fractures may be considered for
Discussion: Joint alignment and stability has been shown tension-banding techniques. Unstable fractures must
to be more important in determining overall outcome with be fixed with more rigid constructs, such as plating.
these fractures than joint congruity. Proximal fracture Comminuted fractures are examples of unstable fractures.
dislocations involving the volar base of the middle phalanx Although the fracture pattern in the radiograph is relatively
may be treated with dorsal block splinting if the joint can simple, the obliquity passes distal to the center of the
reduce with flexion. The radiograph illustrates a failed semilunar (or trochlear) notch and is considered unstable
reduction attempt as the joint is hinging open (“wedge and unsuitable for tension banding techniques.
sign”), rather than becoming concentric. In these cases,
References:
nonoperative treatment is not an option. Operative
1. Villanueva P, Osorio F, Commessatti M, Sanchez-Sotelo J.
intervention in the form of ORIF is the most appropriate
Tension-band wiring for olecranon fractures: analysis of risk
answer. If repair is not possible because the fragments are factors for failure. J Shoulder Elbow Surg 2006;15(3):351-356.
too comminuted, hinged external fixation or reconstruction 2. Murphy DF, Greene WB, Gilbert JA, Dameron TB Jr. Displaced
with hamate autograft is reasonable. olecranon fractures in adults. Biomechanical analysis of fixation
methods. Clin Orthop Relat Res 1987;(224):210-214.
References:
3. Veillette C, Steinmann S. Olecranon Fractures. Orthop Clin
1. Shibata T, O’Flanagan SJ, Ip FK, Chow SP. Articular fractures of North Am 2008;39(2):229-236.
the digits: A prospective study. J Hand Surg 1993;18B:225-229.
2. Stern PJ, Roman RJ, Kiefhaber TR, McDonough JJ. Pilon
fractures of the proximal interphalangeal joint. J Hand Surg
1991;16A:844-850.
3. Dobyns JH, McElfresh EC. Extension block splinting. Hand Clin
1994;10(2):229-37.

Figure 1

Figure 1

2010 Self-Assessment Examination | 23


48. A 38 year-old male presents with a contracture of 49. When performing tendon transfers for complete
the elbow following radial head ORIF one year prior. radial nerve palsy, in the absence of a palmaris
Radiographs are shown in Figures 1 and 2. An elbow longus, which procedure is the most appropriate?
contracture release was performed. After excising
A. Extensor pollicis longus tenodesis
the anterior capsule, extension improved from 70º to
40º. The next most appropriate step is: B. Flexor capri ulnaris to extensor pollicis longus
C. Brachioradialis to extensor pollicis longus
A. Wound closure and static progressive splinting
D. Flexor digitorum superficialis to extensor pollicis longus
B. Posterior debridement
E. Interphalangeal arthrodesis
C. Fenestration of distal humerus
D. Release one or both collateral ligaments Preferred Response: D
E. Removal of hardware Discussion: In the absence of palmaris longus, several
alternatives have been described. Incorporating EPL into
Preferred Response: B the FCU to EDC transfer greatly diminishes the abduction
component of the transfer’s effect on the thumb. The
Discussion: Functional range of motion for the elbow is
brachioradialis would not be functional with radial nerve
between 30º and 120º, so it would be preferable not to
palsy, but is a reasonable transfer if the palsy was isolated
accept the degree of extension following anterior capsule
to the posterior interosseous nerve. Extensor pollicis longus
alone. Deepening the olecranon fossa or partial olecranon
tenodesis may be possible, but is not as effective as a FDS
excision should be considered after releasing the anterior
III or IV transfer.
tether. An Outerbridge-Kashiwagi (OK) procedure involves
creating a fenestration between the olecranon and References:
coronoid fossa and is indicated for arthritic conditions 1. Bevin AG. Early tendon transfer for radial nerve transection.
involving significant bony abutment. The radiographs do Hand 1976;8:134-136.
not depict osteophytes significant enough to warrant the 2. Goldner JL, Kelley JM. Radial nerve injuries. South Med J
OK procedure. OK is indicated if insufficient extension is 1958;51:873-883.
obtained with posterior debridement. Releasing collateral 3. Tsuge K, Adachi N. Tendon transfer for extensor palsy of
ligaments should be a last resort, and is usually reserved for forearm. Hiroshima J Med Sci 1969;18:219-232.
cases involving heterotopic bone.
References:
50. One advantage a four-flap Z-plasty has over
1. Martin BD, Johansen JA, Edwards SG. Complications related to
a simple 60° Z-plasty for a thumb index web
simple dislocations of the elbow. Hand Clin 2008;24(1):9-25.
contracture is:
2. Kraushaar BS, Nirschl RP, Cox W. A modified lateral approach
for release of posttraumatic elbow flexion contracture. A. A greater increase in length
J Shoulder Elbow Surg 1999;8(5):476-80.
B. A lower rate of contracture recurrence
3. Morrey BF, Askew LJ, Chao EY. A biomechanical study
of normal functional elbow motion. J Bone Joint Surg C. A smoother contour
1981;63A:872-7. D. Preservation of sensation
4. Forster MC, Clark DI, Lunn PG. Elbow osteoarthritis: prognostic
indicators in ulnohumeral debridement—the Outerbridge- E. No skin graft is required
Kashiwagi procedure. J Shoulder Elbow Surg 2001;10(6):557-60. Preferred Response: C
Discussion: A simple 60° Z-plasty has been reported to
give the greatest increase in length and depth of webspace.
A four flap Z-plasty has an advantage of providing a
smoother contour than the 60° Z-plasty. Preservation of
sensation or rates of recurrence are not different between
the flaps. Neither flap requires skin grafting.
References:
1. Woolf RM, Broadbent TR. The four-flap Z-plasty. Plast Reconstr
Surg 1972;49(1):48-51.
Figure 1 Figure 2 2. Westreich M, Shulman Y. Simple method for 60-degree
Z-plasty. Ann Plast Surg 1981;6(1):24-5.

24 | American Society for Surgery of the Hand


51. The arteriogram shown in the figure below is most 52. The expected outcome of amputation for a
consistent with: nonfunctional finger in a patient with a history of the
complex regional pain syndrome (CRPS) is:
A. Buerger’s disease
B. Lupus A. Symptom-free hand
C. Hypothenar hammer syndrome B. Likely use a prosthetic digit
D. Cardiac emboli C. Low likelihood of CRPS recurrence
E. Polyarteritis nodosa D. Persistent pain
E. Wound dehiscence
Preferred Response: A
Preferred Response: D
Discussion: The arteriogram demonstrates diffuse
narrowing of the vessels, particularly the digital ones. This Discussion: Patients with CRPS may undergo amputation
arteriogram is most consistent with Buerger’s disease, also for several reasons, including a chance to ease intractable
called thromboangiitis obliterans. Characteristics consistent pain, cease recurrent infection, or regain function. Results
with Buerger’s disease include (1) small and medium vessel of amputation in these patients have been less then
involvement, (2) segmental occlusive disease, (3) distal gratifying. Although patients report some improvement
disease greater than proximal, (4) collateralization bypassing in their symptoms, the vast majority of patients have
occlusive segments, and (5) normal proximal (e.g., brachial) persistence of some pain at their stump site. Because of
vessels. Thrombectomy and vascular reconstruction may this pain, compliance with prostheses is rare. Psychosocial
be indicated in select cases of Berger’s disease; however, abnormalities that are present prior to amputation persist
the diffuse nature of these lesions preclude these options despite amputation. CRPS has not been associated with
in most patients with this disease. Buerger’s disease is increases of wound dehiscence.
often difficult to differentiate from scleroderma and collagen
References:
vascular disease. The presence of lymphocytic and
1. Dielissen PW, Claassen ATPM, Veldman PHJM, Goris RJA.
polymorphonuclear leukocytes within the vessel walls would
Amputation for reflex sympathetic dystrophy. J Bone Joint Surg
suggest Buerger’s disease. Polyarteritis shows arteries that 1995;77B:270-273.
are dilated or constricted by blood vessel information. 2. Szeinberg-Arazi D, Heim M, Nadvorna H, et al. A functional and
References: psychosocial assessment of patients with post-Sudeck atrophy
amputation. Arch Phys Med Rehabil 1993;74:416-418.
1. Lambeth JT, Yong NK. Arteriographic findings in
thromboangiitis obliterans. Roentgenology 1970;109:553-561.
2. Papa MZ, Adar R. A critical look at thromboangiitis obliterans
(Buerger’s disease). Vasc Surg 1992;5:1-18.
3. Buerger L. Thromboangiitis obliterans: A study of the vascular
lesions leading to presenile spontaneous gangrene. Am J Med
Sci 1980;136:567-580.

Figure 1

2010 Self-Assessment Examination | 25


53. A 13 year-old boy presents in a long arm cast 54. When harvesting a vascularized proximal fibular
6 weeks after closed reduction of a distal one- physeal graft, the transfer must be based on
third fracture of the radius and ulna. X-rays show which artery:
callus formation at the fractures with an angular
A. Anterior tibialis
alignment 12° apex volar. On palpation along the
forearm, there is no tenderness. Grip in the affected B. Posterior tibialis
limb is diminished, but without forearm pain. C. Popliteal
Recommended initial treatment is: D. Deep femoral
A. Revision closed reduction and casting for 6 weeks E. Peroneal
B. Continued long arm casting with the forearm in Preferred Response: A
pronation for 3 weeks
Discussion: While the typical diaphyseal vascularized
C. Immediate open osteotomy and internal fixation
fibular transfer is based on the peroneal vascular system,
D. Munster cast immobilization with the forearm in work by Innocenti et al has shown that the nutrient vascular
neutral for 3 weeks supply to the proximal fibular apophysis is derived from
E. Immediate range of motion with reassessment in the anterior tibialis artery. This requires an anterolateral
3 weeks approach to the proximal fibula.
Preferred Response: E References:
1. Innocenti M, Delcroix L, et al. Vascularized proximal fibular
Discussion: Cadaveric studies by Matthews, Kaufer, et al,
apophyseal transfer for distal radial reconstruction. J Bone
have shown that angular deformities involving midshaft Joint Surg 2004;86A:1504-1511.
fractures of the forearm begin to impact forearm rotation 2. Ghert M, Coulter John N, Manfrini M. Use of free vascularized
with deformities of 10° and become functionally important fibular grafts in skeletal reconstruction for bone tumors in
beyond 20°. Tarr et al, felt significant loss of forearm rotation children. J Am Acad Orthop Surg 2007;15:577-587.
began with midshaft fractures angulated as little at 15°, but
found that fractures of the distal third of the forearm sustained
similar losses in pronation, and less in supination compared 55. A 23 year-old female presents to the emergency
to midshaft fracture angulation. On the basis of clinical and room with vomiting and increasingly severe pain
radiographic examination, the fractures are healed; therefore, in her right hand. Forty-eight hours earlier, she
additional immobilization would not be appropriate. Given that had popped a small pimple over the dorsal third
the present fracture alignment will most likely affect pronation web of the right hand with a pin. She has fusiform
(which can be compensated for by shoulder abduction), the swelling of her right hand to the midforearm with
appropriate treatment would be to initiate immediate range scattered serous blistering about the distal forearm.
of motion and strengthening and to reassess the functional There are hemorrhagic blisters of the dorsal third
impact of the deformity, as opposed to proceeding with webspace and extending into the middle and ring
immediate corrective osteotomy and internal fixation. fingers, which are swollen and cyanotic. The patient
References: demonstrates significant discomfort with attempts
1. Matthews SL, Kaufer H, et al. The affect on supination/ at active and passive range of motion of her fingers.
pronation of angular malalignment of the fracture of both bones Recommended treatment is:
of the forearm. J Bone Joint Surg 1982;64A:14-17.
A. Admission for IV hydration and antibiotics
2. Tarr RR, Garfield AL, Sarmiento A. The affects of angular and
rotational deformities of both bones of the forearm. J Bone B. Right upper extremity arteriogram to evaluate the
Joint Surg 1984;66A:65-70. vascular status of the extremity
3. Ring D. Forearm fractures and reconstruction. In: Trumble C. Compartment pressures and possible forearm
TE, Budoff JE, eds. Hand Surgery Update IV. Rosemont, IL: fasciotomies
American Society for Surgery of the Hand, 2007.
D. Transfer for hyperbaric oxygen treatment
E. Immediate surgical debridement of the right hand
and forearm
Preferred Response: E
Discussion: The patient’s history and presenting clinical
findings are consistent with a patient with a necrotizing
fasciitis. Necrotizing fasciitis is a rapidly progressive
soft tissue infection, which is frequently life-threatening.
Necrotizing fasciitis is sometimes mistakenly diagnosed as
cellulitis. The distinguishing features of necrotizing fasciitis

26 | American Society for Surgery of the Hand


are a rapid, progressive, high fever, significant swelling with exact role of surgical intervention in the treatment of distal
blister formation, without the erythema and induration biceps tendinosis is not known, literature does support
seen with cellulitis. While admission for IV hydration and good results for surgical treatment of this condition when it
antibiotics is appropriate, the only treatment shown to change is recalcitrant to conservative measures and the passage of
the prognosis of necrotizing fasciitis is immediate surgical time.
debridement and removal of necrotic tissue. A consistent
References:
finding with necrotizing fasciitis is pain out of proportion to the
1. Rineer CA, Ruch DS. Elbow tendinopathy and tendon ruptures:
appearance of the infection. The destruction of subcutaneous
Epicondylitis, biceps and triceps rupture. J Hand Surg
tissues, including vascular structures, can create symptoms 2009;34A:566-576.
of vascular compromise and ischemia. As this destruction 2. Hobbs MC, Koch J, Bamberger HB. Distal biceps tendinosis:
is extrafascial, there is no compromise of the major arterial Evidence based review. J Hand Surg 2009;34A:1124-1126.
structures perfusing the extremity nor are there issues with
increased compartmental pressures.
References: 57. The amount of key pinch strength in a patient with a
1. Vellapianta JM, Ljunquist K, et al. Necrotizing fasciitis. J Am C6 level tetraplegia needed to improve activities of
ACAD Orthop Surg 2009;17:174-84. daily living is:
2. Sunderland IR, Friedrich JB. Predictors of mortality and limb
A. 2 kilograms
loss in necrotizing soft tissue infections of the upper extremity.
J Hand Surg 2009;34A:1900-1901. B. 5 kilograms
3. Sarani, et al. Necrotizing fascitis: Current concepts and review. C. 7 kilograms
J Am Coll Surg 2009;208(2):279-288.
D. 10 kilograms
E. 12 kilograms
56. A 45 year-old male who is an avid weightlifter presents Preferred Response: A
with a 6-month history of right anterior elbow pain. His
Discussion: Achieving 2-kilogram pinch strength has
pain has not improved with cessation of weight lifting.
been shown to enhance the performance of many
He has an MRI demonstrating diffuse tendinosis of the
activities of daily living. Hamou et al recently conducted a
distal biceps insertion. Recommended treatment is:
systematic review of the literature to evaluate the results
A. Six weeks of immobilization in a long arm cast of restoration of pinch and elbow extension in people with
B. Surgical debridement and reattachment of distal tetraplegia. Average restoration of pinch strength using the
biceps tendon brachioradialis transfer was 2 kilograms, whereas pinch
strength restored using a tenodesis procedure was 1 kilogram.
C. A series of two steroid injections to the biceps sheath
three weeks apart References:
D. Referral to physical therapy for iontophoresis 1. Smabyn, Johanson ME, et al. Identification of Key Pinch Forces
Required to Complete Functional Tasks. J Rehabil Res Dev
E. Percutaneous biceps tenotomy 2004;41:215-224.
Preferred Response: B 2. Hamou C, Shah NR, et al. Pinch and elbow extension
restoration in people with tetraplegia: A systematic review of
Discussion: A recent review of the literature on distal the literature. J Hand Surg 2009;34A:692-699.
biceps tendinosis by Hobbs, Koch, and Bamberger
reported that not only is the natural history of this condition
poorly understood, but there are few studies that have
investigated the results of conservative management. All
studies cited in the review demonstrated poor results with
conservative care, and most patients undergoing surgery
showed good results. Given the similarities between distal
biceps tendinosis and other enthesopathy, it is reasonable
to consider initial conservative treatment in the form of
avoidance of provocative activities and the passage of
time to see if symptoms abate. There is no evidence
in the literature to support the role of immobilization or
iontophoresis for treatment of this condition. Studies of
steroid injections for similar conditions, such as elbow
epicondylitis, show they do not change the natural history
of the disease. While a percutaneous biceps tenotomy
might alleviate the pain, this is not a recognized treatment,
nor would it be accepted by an avid weightlifter. While the

2010 Self-Assessment Examination | 27


58. A 35 year-old female presents with a 6-week history Discussion: The expendability and length of the palmaris
of bilateral wrist pain and dorsal hand swelling. longus has historically made this the tendon donor of choice
Clinical examination indicates 4th dorsal compartment for stage flexor reconstruction. The palmaris longus is an
tenosynovitis. Laboratory studies include an example of extrasynovial tendon, while the flexor digitorum
elevated erythrocyte sedimentation rate and positive superficialis is an intrasynovial tendon. Experimental studies
rheumatoid factor. The treatment that will have the have shown that these two types of tendons, when used as
greatest impact on the patient’s long term outcome is: grafts, heal by different methods. The extrasynovial tendon
serves as a scaffold and must be repopulated by tenocytes.
A. Wrist splints and nonsteroidal anti-inflammatory drugs
An intrasynovial graft heals intrinsically, maintaining the
B. Steroid injection 4th dorsal compartment viability of the tenocyte. As such, it possesses greater
C. Surgical tenosynovectomy strength during the healing process. The intrasynovial graft
D. Referral to a rheumatologist within 6 weeks also has a gliding surface that it retains. Such a surface is
not found with an extrasynovial graft.
E. Observation for 6 months
References:
Preferred Response: D
1. Seiler JG, Chu CR, et al. The Marshall R. Ruist Young
Discussion: Wrist splinting, nonsteroidal anti-inflammatory Investigator Award. Autogenous flexor tendon grafts. Biologic
medication, and steroid injections are all conservative mechanisms for incorporation. Clin Orthop 1997;(345):239-247.
measures for tenosynovitis. In this case, however, the 2. Nishida J, Amadio PC, et al. Excursion properties of tendon
clinical picture and laboratory findings are specific for graft sources: Interaction between tendon and A2 pulley.
J Hand Surg 1998;23A:274-284.
rheumatoid arthritis. Randomized controlled clinical studies
have shown that initiating disease-modifying antirheumatic
drugs within 3 months of diagnosis leads to a significant
decrease in the morbidity of rheumatoid arthritis. The 60. On performing an anterior approach to a ruptured
newest generation of biological response modifiers, such as distal biceps tendon, an arterial structure is noted
anti-TNF alpha drugs or recombinant IL-1 antagonist, used passing obliquely within the wound toward the
in combination with methotrexate is particularly effective brachioradialis muscle. To facilitate exposure
in modifying the inflammatory response and destruction division of this arterial structure is desirable. The
seen with rheumatoid arthritis. With the advances in the arterial structure to be ligated and divided is:
pharmacological modification of rheumatoid arthritis, A. Radial
early surgical intervention would be inappropriate. While
B. Recurrent radial
observation for six months prior to considering surgery is
reasonable, the initiation of medical management early in C. Radial collateral
the disease will have the most significant impact on long- D. Anterior interosseous
term prognosis. E. Recurrent branch of the posterior interosseous
References: Preferred Response: B
1. Doan T, Massarotti E. Rheumatoid arthritis: An overview of new
and emerging therapies. J Clin Pharm 2005;45(7):751-762. Discussion: In the absence of vascular anomalies, the
2. Little KJ, Stern PK. Rheumatoid arthritis reconstruction. brachial artery divides into the radial and ulnar arteries,
In: Trumble TE, Budoff JE, eds. Hand Surgery Update IV. just distal to the elbow joint. The first branch of the radial
Rosemont, IL: American Society for Surgery of the Hand, 2007. artery is the radial recurrent artery, which passes radially
and proximally from the radial artery, just distal to the
biceps tendon. The radial collateral artery is a branch of the
59. The reason a flexor digitorum superficialis tendon is profunda brachii, and eventually, an anastomosis with the
a superior graft over a palmaris longus tendon for a radial recurrent artery. The interosseous artery is a proximal
staged flexor tendon reconstruction is: branch of the ulnar artery and will divide into anterior
and posterior interosseous branches. The interosseous
A. The flexor digitorum superficialis graft is longer recurrent artery usually arises from the posterior
B. The flexor digitorum superficialis graft has a interosseous branch.
wider diameter
References:
C. The palmaris longus is sometimes absent
1. Morrey BF. Anatomy of the elbow joint. In: Morrey BF, ed. The
D. An extrasynovial graft has greater strength and elbow and its disorders. Philadelphia: WB Saunders Company,
gliding capacity 1985:24-27.

E. An intrasynovial graft heals intrinsically with 2. Henry AK. Extensile exposure. 2nd ed. Edinburgh: Churchill
Livingstone, 1973: 94-100.
preservation of the gliding surface
Preferred Response: E

28 | American Society for Surgery of the Hand


61. A 23 year-old male presents six months after a 62. The most appropriate treatment for the injury shown
motorcycle accident with a closed left brachial in Figure 1 is:
plexus injury. The patient has intact sensation and
A. Immobilization of the PIP joint in 60° of flexion for
motor function below the elbow and diffuse muscle
3 weeks
atrophy about the shoulder. He is able to abduct
30° with 3/5 strength. He can flex his elbow with B. Early active range of motion in a dorsal blocking splint
3+/5 motor strength. He has recently undergone set at 30°
an EMG of his left upper extremity showing the C. A dynamic distraction external fixator
following findings: D. Volar plate arthroplasty
Emg Summary Table E. Hemi hamate graft (osteochondral autograft)
Insertional Positive Recruitment Preferred Response: C
Fibrillation
Activity Sharp Waves Pattern
Deltoid +1 +1 +3 Absent Discussion: Pilon fractures of the articular base of the middle
Biceps 1- None +2 Discrete phalanx are, by definition, comminuted and not amenable
Triceps Normal None None Normal
to internal fixation. Immobilization of these injuries typically
results in considerable loss of motion. Due to the comminution
Supraspinatus Normal None +1 Normal
of the articular surface, there is no stable articular base that
can be positioned against the articular surface of the head
The patient would like to enhance his strength of
of the proximal phalanx for early active motion. Dorsal block
shoulder abduction. For treatment, you recommend:
splinting is most appropriate for volar rim fractures involving
A. Spinal accessory nerve transfer to the less than 30% of the joint surface. While an Eaton volar plate
suprascapular nerve arthroplasty and a hemi hamate graft can be used in articular
B. Intercostal nerve transfer to the musculocutaneous nerve based fractures of the middle phalanx of increasing severity,
C. Radial nerve branch to the long head of the triceps both rely on a portion of the dorsal cortex of the articular
transfer to the axillary nerve face to have remained intact. This is not the case in a pilon
fracture. Several studies have documented the effectiveness of
D. Latissimus dorsi transfer to the rotator cuff
dynamic distraction external fixation (Figure 2) in the treatment
E. C5-C6 sural nerve grafting to the upper trunk of the of pilon fractures and have shown it to be the most effective
brachial plexus means of preserving range of motion in these fractures.
Preferred Response: C References:
Discussion: The EMG shows denervation primarily in 1. Stern PJ, Roman RJ, et al. Pilon fractures of the proximal
the deltoid and biceps. There is evidence of injury to the interphalangeal joint. J Hand Surg 1991;16A(5):844-850.
supraspinatus; however, this appears to have recovered, 2. Calfee RP, Sommer Kamp, TG. Fracture dislocations about the
finger joints. J Hand Surg 2009;34A:1140-1147.
and is providing abduction and external rotation to the left
shoulder. Given the supraspinatus muscle is functioning 3. Rutland RT, Hogan CJ, et al. Use of dynamic distraction
external fixation for unstable fracture dislocations of the
both clinically and electrodiagnostically, there would be proximal interphalangeal joint. J Hand Surg 2008;33A:19-25.
no reason to perform a transfer of the spinal accessory
nerve to the suprascapular nerve. For this same reason, it
would not be appropriate to perform a latissimus transfer
to the rotator cuff. While there are several methods
of reinnervating the biceps, such as intercostal nerve
transfer to the musculocutaneous, this would not aid in
strengthening shoulder abduction. However, it would help
with strengthening elbow flexion and forearm supination. This
could also be accomplished by a partial ulnar nerve transfer
to the musculocutaneous. To restore strength of abduction
of the shoulder, it is necessary to reinnervate the deltoid. Figure 1
Perhaps the closest intact functioning nerve branch would
be the radial nerve branch to the long head of the triceps.
References:
1. Leechavengvongs S, Witoonchart K, Uerpairojkit C,
Thuvasethakul P. Nerve transfer to deltoid muscle using the
nerve to the long head of the triceps, part II: a report of 7
cases. J Hand Surg 2003;28A:633-8.
2. Bertelli JA, Ghizoni MF. Nerve transfer in C5 and C6 avulsions.
J Hand Surg 2004;29A:131-139.
Figure 2
2010 Self-Assessment Examination | 29
63. The first step in skin graft survival is: 65. A patient with rheumatoid arthritis has a type II swan
neck deformity where the PIP joint has full flexion,
A. Adherence
but flexion is limited with the MP joint in extension.
B. Imbibition Silastic MP arthroplasties are planned. What
C. Inosculation procedure should also be performed?
D. Primary contracture A. DIP fusion
E. Secondary contracture B. Spiral oblique retinacular ligament reconstruction
Preferred Response: A C. PIP joint fusion
Discussion: Adherence occurs in the first 8 hours after D. Lateral band mobilization
skin graft placement. It is followed by imbibition during E. Intrinsic release
the initial 48 hours when nutrient transfer occurs through
Preferred Response: E
diffusion from the surrounding tissues. Inosculation occurs
next when vascular networks from the underlying bed and Discussion: Type II swan neck deformity develops from MP
the graft make connections. Capillary ingrowth results joint synovitis and subluxation. Sagittal band attenuation
in revascularization of the graft. Primary contracture of weakens the extensor force and intrinsic contracture
the skin graft occurs following harvesting and before results. Procedures that limit PIP hyperextension are not
graft placement. Secondary contracture is a late wound- sufficient. An intrinsic release, possibly combined with MP
healing property. arthroplasties, must be performed also.
References: Lateral band mobilization is indicated when passive PIP
1. Smith M, Munster A, Spence R. Burns of the hand and upper motion is limited by a contracted extensor mechanism (type
limb—a review. Burns 1998;24:493-505. III swan neck).
2. Sood, ed. Achauer and Sood’s Burn Surgery: Reconstruction
and Rehabilitation, 2006. References:
1. Keifhaber TR, Strickland JW. Soft tissue reconstruction for
rheumatoid swan neck and boutonniere deformities: long term
results. J Hand Surg 1993;18A:984-989.
64. The most common site of metastasis for a malignant 2. Rizio L, Belsky MR. Finger deformities in rheumatoid arthritis.
fibrous histiocytoma of the upper extremity is: Hand Clin 1996;12:531-540.

A. Brain
B. Lung
66. A patient with chronic upper extremity ischemia has
C. Liver Raynaud’s phenomenon, ulcers, pain and gangrene.
D. Dermal satellites Initial treatment should include avoidance of cold and:
E. Regional lymph nodes A. Intra-arterial reserpine
Preferred Response: B B. Topical nitroglycerine
Discussion: Malignant fibrous histiocytoma is among the C. Digital sympathectomy
most common soft tissue sarcomas in adults presenting in D. Avoidance of nicotine products
the sixth to eighth decade. Although common in the upper E. Calcium channel blockers
extremity, hand involvement is less common. Following
adequate wide excision and radiation therapy, recurrence Preferred Response: D
rates range from 10-30%. The metastatic rate is 35% Discussion: In patients with chronic ischemia of the upper
with the lung most commonly involved. Survival rates are extremity, initial treatment includes avoidance of cold
dependent on grade. Lymph node metastasis is uncommon. exposure and cessation of nicotine products. Morecraft
References: used an ultrasonic Doppler velocimeter to show smoking
1. Gustafson P, Amer M. Soft tissue sarcoma of the upper caused increased vascular resistance and decrease in
extremity: descriptive data and outcome in a population based volumetric blood flow and tissue perfusion.
series of 108 adult patients. J Hand Surg 1999;24A:668-674.
References:
2. McPhee M, McGrath BE, Zhong P, et al. Soft tissue sarcoma of
the hand. J Hand Surg 1999;24A:1001-1007. 1. Jones NF. Acute and chronic ischemia of the hand:
Pathophysiology, treatment and prognosis. J Hand Surg
1991;16A:1074-1083.
2. Morecraft R, Blair WF, Brown TD, et al. Acute effects of
smoking on digital artery blood flow in humans. J Hand Surg
1994;19A:1-7.

30 | American Society for Surgery of the Hand


67. Fitting for myoelectrically controlled, electrically- 69. A college rower develops pain and swelling 4
powered artificial hands for a child with bilateral centimeters proximal to the wrist joint. This is
transverse deficiencies at the upper forearm level is tenosynovitis of which dorsal compartment?
best done at what age?
A. First
A. 1 month old B. Second
B. 4 months old C. Third
C. 18 months old D. Fourth
D. 3 years old E. Fifth
E. 12 years old
Preferred Response: B
Preferred Response: D
Discussion: Frequently seen in athletes in sports such as
Discussion: For transverse deficiencies at the upper rowing and weight lifting, intersection syndrome presents
forearm level, early prosthetic fitting begins at age 4 to 6 with pain, swelling, and crepitus 4 cm proximal to the
months with a simple nonfunctional forearm and hand. A wrist joint. Muscle bellies and tendons of the first dorsal
body-powered split hook is introduced at 15 to 18 months. compartment cross the tendons of the second dorsal
Myoelectrically controlled artificial hands are best fitted at 2 compartment. Grundberg and Reagan demonstrated
to 3 years of age. intersection syndrome to be tenosynovitis of the
second dorsal compartment. Initial treatment is usually
References:
nonoperative, with most patients improving and remaining
1. Lamb DW, Law HT. Upper Limb Deficiencies in Children:
asymptomatic without surgery.
Prosthetic, Orthotic, and Surgical Management. Boston: Little
Brown, 1987. References:
2. Sorbye R. Myoelectric hand prostheses in children. Int J 1. Grundberg AB, Reagan DS. Pathologic anatomy of the forearm:
Rehabil Res 1977;1:15. Intersection syndrome. J Hand Surg 1985;10A:299-302.
2. Wood MB, Dobyns JH. Sports related extraarticular wrist
syndromes. Clin Orthop 1986;202:93-102.
68. Medical grade leeches were used to relieve venous
congestion following a distal digital replantation.
A subsequent wound infection is likely due to 70. At 14-year follow up, what percentage of silastic
which organism? metacarpophalangeal joint replacement implants
A. Staphylococcus aureus remain without fracture?

B. Methicillin resistant Staphylococcus aureus A. 15


C. Pseudomonas aeruginosa B. 35
D. Aeromonas hydrophila C. 55
E. Mycobacterium marinum D. 75

Preferred Response: D E. 95

Discussion: Medical grade leeches are used when venous Preferred Response: B
congestion threatens survival of a replanted part. Infection Discussion: Goldfarb evaluated 36 patients at 14 years
can occur with Aeromonas hydrophila, a gram negative and found fractures occurred in all, except 37% of MP
anaerobic rod. This bacterium is endosymbiotic with the silastic implants. Of these patients without fracture, another
leech. It inhibits growth of other bacteria and produces 22% had severely deformed implants. Trail retrospectively
enzymes essential for the breakdown of red blood cells reviewed 1336 implants in 381 patients and found 34%
and hemoglobin. Treatment usually involves debridement remained without fracture at 17 years. Crossed intrinsic
and appropriate antibiotics, which include trimethoprim- transfers improved implant survival to 90% at 15 years.
sulfamethoxazole, ciprofloxacin, and second generation
cephalosporin. Prophylactic antibiotics are recommended References:
when using leeches. 1. Goldfarb CA, Stern PJ. Metacarpophalangeal joint arthroplasty
in rheumatoid arthritis: a long-term assessment. J Bone Joint
References: Surg 2003;85:1869-1878.
1. Brody GA, Maloney WJ, Henz VR. Digit replantation applying 2. Trail IA, Martin JA. Seventeen year survivorship analysis of
the leech Hirudo medicinalis. Clin Orthop 1989;245:133-137. silastic metacarpophalangeal joint replacement. J Bone Joint
2. Lowen RM, Rodgers CM, Ketch ll, et al. Aeromonas hydrophila Surg 2004;86B:1002-1006.
infection complicating digital replantation and revascularization.
J Hand Surg 1989;14A:714-718.

2010 Self-Assessment Examination | 31


71. The infant whose hands are shown in Figures 1 and 2
is expected to have a mutation in which gene?
A. FGF receptor 2 (FGFR2)
B. Homeobox D13 (HOXD13)
C. Holt-Oram syndrome 1 (HOS1)
D. Human zinc factor protein gene 3 (GLI3)
E. Brachyury T-box 5 (TBX5)
Preferred Response: A
Discussion: This is an Apert child. All Apert children
display one of two point mutations in the gene encoding Figure 1
FGF receptor 2 (FGFR2). These two-point mutations are
Ser252Trp and Pro253Arg. Patients with Apert syndrome
have inversely proportional deformity with respect to
cranial and hand involvement. The Ser252Trp mutation is
associated with the “good hands, bad head” phenotype,
and the Pro253Arg mutation is associated with the “bad
hands, good head” phenotype. Apert syndrome has
autosomal dominant inheritance.
HOXD13 is associated with central polysyndactyly, HOS1
with Holt-Oram syndrome, GLI3 with Greig syndrome, and
TBX5 with hand-heart syndrome. These four conditions are
also inherited in an autosomal dominant manner.
Hand-heart syndrome includes that group of patients
with radial-sided skeletal abnormalities and cardiac
abnormalities (similar to Holt-Oram syndrome), but Figure 2
who also have cardiac conduction anomalies. Greig (or
cephalopolysyndactyly) syndrome has craniofacial and
limb abnormalities, including postaxial polydactyly. GLI3 72. Mechanical shear stress with negative pressure
regulates sonic hedgehog (SHH) and is important in wound therapy (V.A.C.) stimulates granulation
determining digit number and digit identity. tissue. This has been shown to result from increased
References: expression of extracellular regulated kinase (ERK)
1. Borud LJ, Upton J. Embryology of the upper limb. In: Mathes which causes:
SJ, Hentz VR, eds. Plastic Surgery. 2nd ed. Philadelphia: A. Intracellular production of collagen
Saunders/Elsevier, 2006;8:20-21.
2. Muragai Y, Mundlos S, Upton J, Olsen BR. Altered growth in B. Increased cellular mitosis
branching patterns in synpolydactyly caused by mutations in C. Increased cross-linking between collagen fibers
HOXD13. Science 1996;272-548.
D. Stimulation of phagocytosis by macrophages
3. Kalff-Suske M, Wild A, Topp J, et al. Point mutations
throughout the GLI3 gene cause Greig cephalopolysyndactyly E. Production of integrins
syndrome. Human Mole Genet 1999;8:1769.
Preferred Response: B
4. Bonnet D, Laplet A, Legeai-Mallet L, et al. A gene for Holt-
Oram syndrome mapped to the distal long arm of chromosome Discussion: Fibroblasts are deformed and stretched by
12. Nat Genet 1994;6:405. negative pressure wound therapy, in the same way as occurs
5. Basson C, Solomon SD, Weissman B, et al. Genetic heterogeneity with tissue expansion and distraction osteogenesis. Kinases
of heart-hand syndrome. Circulation 1995;91:1326. are responsible for phosphorylation. Stretching fibroblasts
6. Li QY, Newbury-Ecob RA, Jerrett JA, et al. Holt-Oram results in increase in expression of extracellular regulated
syndrome is caused by mutations in TBX5, a member of the
kinase (ERK) and Jun N-terminal protein kinase (JNK), which
Brachyury (T) gene family. Nat Genet 1997;15:21.
in turn are associated with cell division. There is an increase
in the mitotic rate of the stretched cells. Applied mechanical
shear stresses activate the vascular endothelial cell growth
factor (VEGF) pathway. ERK and JNK are activated in the
VEGF pathway. These applied mechanical forces deform
tissues, which result in deformation of cells; then, this is
followed by stimulation of growth factor pathways, resulting
in increased mitosis and production of new tissue.

32 | American Society for Surgery of the Hand


References: 74. The abnormality depicted in Figures 1 and 2
1. Morykwas MJ, Simpson J, Punger K, et al. Vacuum-assisted occurred during embryogenesis between weeks:
closure: state of basic research and physiologic foundation.
Plast Reconstr Surg 2006;117:121. A. 1 and 2
2. Chen KD, Li YS, Kim M, et al. Mechanotransduction in B. 3 and 4
response to shear stress: Roles of receptor tyrosine kinases, C. 5 and 7
integrins, and Shc. J Biol Chem 1999;274:183-193.
D. 8 and 10
E. 11 and 13
73. Which of the following dorsal retinacular distal radius Preferred Response: C
pedicle bone grafts has the longest vascular pedicle?
Discussion: Embryogenesis of the upper extremity begins
A. 1,2 intercompartmental supraretinacular artery at 4 weeks and is complete by 8 weeks after fertilization.
(1,2 ICSRA) At 5 weeks, the hand segment is present in the form
B. 2,3 intercompartmental supraretinacular artery of a paddle with no separation of the digits. The apical
(2,3 ICSRA) ectodermal ridge regulates separation of the digits, creating
C. 3rd extensor compartmental (EC) branch of 2,3 ICSRA longitudinal separation of the digits via apoptosis during
weeks 6 and 7.
D. 2nd extensor compartmental (EC) branch of 1,2 ICSRA
E. 4th extensor compartmental (EC) branch of the 5thECA References:
1. Kozin SH. Syndactyly. J Am Soc Surg Hand 2001;1:1-13.
Preferred Response: E
2. Tonkin MA. Failure of differentiation part I: Syndactyly. Hand
Discussion: Unlike the other three dorsal vessels Clin 2009;25:171-193.
(1,2ICSRA, 2,3ICSRA, 4thECA), the 5thECA seldom
provides direct nutrient branches to the radius. However, it
is often used as a retrograde flow conduit to the 4thECA.
This retrograde flow comes from the dorsal intercarpal arch.
It provides the longest vascular pedicle of all the described
dorsal distal radius bone grafts and can easily reach up to
the metacarpals (Figures A and B).

Figure 1

Figure: A) Graft based on 1,2 ICSRA and its branch to the 2nd extensor
compartment. B) Graft based on the 4th ECA with retrograde flow through
the 5th ECA from the dorsal intercarpal arch. (From Shin AJ, Bishop AT. J
Am Soc Surg Hand 2002; 2:186).

References:
1. Shin AY, Bishop AT. Vascularized bone grafts from the distal
radius for disorders of the carpus. J Am Soc Surg Hand
2002;2:181. Figure 2
2. Sheetz KK, Bishop AT, Berger RA. The arterial blood supply
of the distal radius and its potential use in vascularized bone
grafts. J Hand Surg 1995;20A:902.

2010 Self-Assessment Examination | 33


75. What is the predominant collagen type in a obtain intraoperative specimen radiography or send a
granulating wound of three weeks duration? specimen for a CT scan to show the nidus.
A. I Ultrasound localization has been used for certain
B. II soft tissue tumors and lymph nodes, but not for small
intraosseous lesions.
C. III
D. IV References:
1. DeSmet L. Osteoid osteoma of the wrist and hand. J Am Soc
E. V
Surg Hand 2001;1:267.
Preferred Response: A 2. Swee R, McLeod R, Beabout J. Osteoid osteoma, detection,
diagnosis, and localization. Radiology 1979;130:117.
Discussion: In the first 4-5 days, the predominant collagen
3. Bednar M, Weiland A, Light T. Osteoid osteoma. Hand Clin
type is collagen type III. The collagen in uninjured skin 1995;11:211.
is 80-90% type I and 10-20% type III. The appearance
of collagen type III in fresh wounds coincides with the
presence of fibronectin. In granulation tissue, type III
collagen comprises 30% and in the mature scar is down
to 10%. Collagen I, except in the earliest phases of wound
healing, is the predominant collagen in a granulating wound.
References:
1. Witte M, Barbul A. General principles of wound healing. Surg
Clin North Am 1997;77:509.
2. Broughton G, Janis J, Attinger CE. The basic science of wound
healing. Plast Reconstr Surg 2006;117:12.

Figure 1
76. A 45 year-old male patient complains of persistent
deep wrist pain. The plain wrist radiographs were
normal. Computed tomography of the wrist showed
the lesion seen in Figure 1. Which of the following is 77. A 36 year-old healthy female patient, with no known
the best way to localize the lesion intraoperatively? drug allergies, is to undergo a 2.5 hour elective
soft tissue surgery for release of thumb adduction
A. Technetium bone scan
contracture and flap reconstruction. Based on the
B. Administration of tetracycline best evidence-based medical practice, which of the
C. Intravenous methylene blue following is the first line prophylactic antibiotic choice?
D. Fluorescein dye A. Methicillin
E. Intraoperative ultrasonography B. Clindamycin
Preferred Response: B C. Cefazolin

Discussion: This is a classic example of an osteoid D. Vancomycin


osteoma with a relatively radiolucent “nidus” in a sclerotic E. Penicillin
area. Plain radiographs may not show the lesion. Computed
Preferred Response: C
tomography may be valuable to show the tumor. They will
nearly always be “hot” on technetium bone scan. Discussion: Prophylactic antibiotics reduce the rate of
postoperative infection after clean orthopedic procedures
While radio-frequency ablation has been the suggestion for
lasting longer than 2 hours without prosthetic implantation.
treatment, especially for osteoid osteomas involving larger
Complex reconstructive and microvascular procedures have
bones and joints, en bloc excision of the nidus and some
a decreased incidence of infection with preoperative antibiotic
surrounding sclerotic bone is necessary to ensure complete
use. One of the absolute indications for the administration of
resection and pain relief.
prophylactic antibiotics in hand surgery includes clean elective
Exact nidus localization has been determined preoperatively procedures > 2 hours duration. To minimize the risk of surgical
by placement of a needle under CT guidance. The site infection, the selected prophylactic antibiotic should cover
fluorescent localization by preoperative administration of the most common infecting pathogens, but need not cover all
oral tetracycline for bone labeling and use of a Woods UV potential contaminating organisms. For class I/clean surgeries,
lamp intraoperatively enables localization. Radioisotope the CDC recommends Cefazolin as the first line agent for
labeling has also been successfully used. One can also antimicrobial prophylaxis and recommends against routine
use of Vancomycin.

34 | American Society for Surgery of the Hand


References: References:
1. Mangram AJ, Horan TC, Pearson ML, et al. Guideline for 1. Kjaer-Petersen K, Langhof O, Andersen K. Bennett’s fracture.
prevention of surgical site infection, 1999. Hospital Infection J Hand Surg 1990;15B:58.
Control Practices Advisory Committee. Infect Control Hosp 2. Oosterbos CJ, deBoer HH. Nonoperative treatment of Bennett’s
Epidemiol 1999;20:250. fracture: A 13-year follow-up. J Orthop Trauma 1995;9:23.
2. Annlan PS, Andenaes K, Samdal F, et al. A prospective, 3. Carlsen BT, Moran SL. Thumb trauma: Bennnett fractures,
double-blind, placebo-controlled trial of a single dose of Rolando fractures, and ulnar collateral ligament injuries:
Azithromycin on postoperative wound infections in plastic Current concepts. J Hand Surg 2009;34A:945.
surgery. Plast Reconstr Surg 1995;96:1378.
4. Timmenga EJ, Blokhius TJ, Mas M, et al. Long-term evaluation
3. Hoffman RD, Adams BD. The role of antibiotics in the of Bennett’s fracture. A comparison of open and closed
management of elective and post-traumatic hand surgery. reduction. J Hand Surg 1994;19B:373.
Hand Clin 1998;14:657.
4. Rizbi N, Bille B, Holtom P, et al. The role of prophylactic
antibiotics in elective hand surgery. J Hand Surg 2008;33A:413.
5. Verma MK, Shah AS, Jebson PJ. Cephalosporins in hand
surgery. J Hand Surg 2009;34A:755.
6. Elward AM, McAndrews JM, Young DL. Methicillin-sensitive and
Methicillin-resistant Staphylococcus aureus: Preventing surgical site
infections following plastic surgery. Aesth Surg J 2009;29:232.

78. A 26 year-old man has the injury shown in the


x-ray. In order to have minimum long-term
postoperative pain and the best possible function,
the recommended treatment should include:
A. Open reduction and plate fixation
B. Open or closed reduction and metacarpal fixation
C. Closed reduction by thumb pronation and thumb Figure 1
spica cast application
D. Excision of the fragment and reconstruction of the
volar beak ligament
79. Which of the following nerves provides innervation
E. Arthroscopically assisted reduction of the fracture to the gracilis muscle?
Preferred Response: B A. Genitofemoral
Discussion: This is a Bennett fracture-dislocation. From B. Obturator
available literature, the following treatment guidelines are C. Saphenous
important: 1) reduction of the thumb metacarpal at the
D. Sciatic
CMC joint must be maintained by surgical fixation, whether
using open or percutaneous techniques; 2) fracture reduction E. Femoral
techniques should include thumb pronation to aid in anatomic Preferred Response: B
reduction of the metacarpal to the ulnar fragment; 3) resort to
open reduction when greater than 2 mm (some recommend Discussion: Innervation is through the obturator nerve. The
> than 1 mm) step-off, despite closed reduction attempt. genitofemoral nerve supplies the cremaster muscle. The
saphenous nerve is the largest branch of the femoral nerve.
The metacarpal shaft subluxes in a dorsal, proximal, and radial It supplies branches to the rectus femoris, vastus lateralis,
direction, due to the unbalanced pull of APL, EPL, EPB, and and vastus medialis muscles. The sciatic nerve supplies
adductor pollicis. The small volar-ulnar fragment is stable and the hamstring muscles before dividing into the tibial and
does not necessarily require fixation. Proper fracture reduction peroneal nerves. The femoral nerve supplies the pectineus
requires axial traction, palmar abduction, and pronation, and sartorius muscles.
while applying external pressure over the metacarpal base.
Generally, there are reports of poor outcomes with casting References:
alone, as this is an unstable fracture pattern. 1. Kay SBJ, Lees VC. Free tissue transfer in children. In: Gupta
A, Kay SBJ, Scheker LR, eds. The Growing Hand: Diagnosis
All methods of fixation (intermetacarpal, to the trapezium, or and Management of the Upper Extremity in Children. London:
to the volar-ulnar fragment) have been shown to be effective. Mosby, 2000:982-983.
Clinical studies document improved outcomes with anatomic 2. Mathes SJ, Nahai F. Transplantation of muscle and
reduction and maintenance of minimal articular step-off. musculocutaneous flaps. In: Mathes SJ, Nahai F, eds. Clinical
Applications for Muscle and Musculocutaneous Flaps.
Arthroscopic assistance has not yet been shown to result in St. Louis: Mosby, 1982:663-667.
a better outcome than radiographic evaluation.
2010 Self-Assessment Examination | 35
80. A 67 year-old male patient has spastic hemiplegia Discussion: The patient illustrated has a clinodactyly with
following a stroke. He has a clenched fist a trapezoidal, as opposed to a triangular, middle phalanx.
deformity. A superficialis-to-profundus (STP) Physiolysis is a procedure that involves resection of the
tendon transfer is recommended, rather than finger physeal bracket with interposition of a fat pad. The best
flexor fractional lengthening, based on which of the results are seen in children that are younger than six
following observations? years old, with greater deformity and a trapezoidal middle
phalanx. Lumbrical division, distraction lengthening,
A. Absence of volitional movement
and tendon transfer are not procedures for clinodactyly.
B. Skin maceration in the flexion creases Because of the risk of further shortening of the phalanx and
C. Demonstrable dynamic extrinsic finger flexion function extensive dissection required with osteotomy, physiolysis
D. PIP joints flex while the DIP joints remain extended is a better option for this young patient with a trapezoidal
distal phalanx.
E. Inability to passively extend the fingers
References:
Preferred Response: A
1. Ty JM, James MA. Failure of differentiation: Part II
Discussion: Muscles that contribute to the clenched fist (Arthrogryposis, camptodactyly, clinodactyly, Madelung
deformity in stroke victim patients include FDS and FDP. If deformity, trigger finger and trigger thumb). Hand Clin
2009;25:195-213.
PIP joints flex while DIP joints remain extended, spasticity
of FDS (rather than FDP) may be suspected. FDS muscles 2. Caoutte-Laberge L, Laberge C, Egerszegi EP. Physiolysis
for correction of clinodactyly in children. J Hand Surg
often exhibit a marked degree of spasticity, whereas FDP 2002;27A:659-665.
muscles may be normal or minimally spastic.
When the overriding problem is in the FDS, or when
volitional control is demonstrated in the extrinsic finger
flexor muscles (by having the patient grasp on a finger
placed in the palm, or by dynamic EMG), fractional
lengthening is indicated. Inability to accomplish passive
finger extension and skin maceration does not differentiate
which of the procedures is best. In a hand with skin
maceration and malodor from a clenched fist, in which
no volitional movement is detected, more significant
lengthening of flexor tendons is required and STP tendon
transfer is performed.
References:
1. Braun RM, Vise GT. Sublimus-to-profundus tendon transfers
in the hemiplegic upper extremity. J Bone Joint Surg Figure 1
1973;55A:873.
2. Hisey MS, Keenan MAE. Orthopedic management of upper
extremity dysfunction following stroke or brain injury. In: Green
DP, Hotchkiss RN, Pederson WC, eds. Green’s Operative Hand
Surgery. 4th ed. New York: Churchill Livingstone, 1999:315-318.

81. A three year-old patient with otherwise normal


growth and development presents with his parents
for evaluation of his small fingers. There is no
antecedent trauma and the deformity seen in
Figures 1 and 2 is present bilaterally. The most
appropriate treatment is:
A. Tendon transfer
B. Physiolysis Figure 2
C. Distraction lengthening
D. Surgical division of abnormal lumbrical
E. Observation
Preferred Response: B

36 | American Society for Surgery of the Hand


82. What is the theoretical length gain when performing References:
a 60º Z-plasty? 1. DiDonna ML, Kiefhaber TR, Stern PJ. Proximal row
carpectomy: Study with a minimum of ten years of follow-up.
A. 15% J Bone Joint Surg 2004;86A:2359-65.
B. 25% 2. Jebson PJ, Hayes EP, Engber WD. Proximal row carpectomy:
C. 50% a minimum 10-year follow-up study. J Hand Surg
2003;28A:561-569.
D. 75%
E. 95%
Preferred Response: D
Discussion: Z-plasties are commonly used for scar
contractures and can be fashioned at many angles. There
is commonly a balance between maximizing length and
ease of transposition. Flaps cut at 30º, 45º, and 60º yield
theoretical increases of length of 25%, 50%, and 75%
respectively.
References:
1. Smith P. Lister’s The Hand. 4th ed. London: Harcourt, 2002: 157.
2. Pederson WC, Hotchkiss RN, Wolfe SW. Skin Flaps. In: Green
DP, Hotchkiss RN, Pederson WC, Wolfe SW, eds. Green’s
Operative Hand Surgery. 5th ed. Philadelphia: Churchill
Livingstone/Elsevier, 2005:1654-1655.

83. A 48 year-old male has failed conservative treatment


of the pathology seen on the radiograph of his right Figure 1
wrist. He asks about surgical treatment of his wrist.
When discussing a proximal row carpectomy, the
patient is advised that studies with ten-year follow-
84. Compared with the radial forearm flap, an
up indicate the following outcome:
advantage of the ulnar artery forearm flap is:
A. Patients typically do not return to the same occupation
A. A donor site that can often be closed primarily
B. Long-term range of motion will be more than 80% of
B. A lower incidence of neurologic compromise
the uninjured side
C. A more hirsute donor site
C. There is a low rate of patient satisfaction with the
surgical outcome D. No sacrifice of a major artery
D. Development of radiographic arthritis does occur and E. Ability to cover defects about the elbow as well as
correlates with symptoms the hand
E. Grip strength following the procedure increases to Preferred Response: A
80% of the uninjured side
Discussion: Both the radial and ulnar artery forearm flaps
Preferred Response: E can be based proximally and distally. Both require the
sacrifice of the radial and ulnar arteries, respectively. The
Discussion: Multiple studies have assessed the durability
ulnar artery runs in close proximity to the ulnar nerve, and
of proximal row carpectomy results. At ten-year follow-
therefore, has a higher incidence of nerve injury—often
up, the motion is approximately 60% of the uninjured side
transient. The donor site for the ulnar artery forearm flap
and strength is 80%, compared with the uninjured side,
can often be closed primarily in older individuals and bears
not corrected for hand dominance. Patients are typically
less hair than the radial forearm flap.
pleased with the results and return to previous occupations.
Radiocapitate degenerative changes have been reported Reference:
(almost universally in some studies), but do not typically 1. Guimberteau JC, Goin JL, Panconi B, Schuhmacher B. The
correspond with symptoms. reverse ulnar artery forearm island flap in hand surgery: 54
cases. Plast Reconstr Surg 1988;81:925-32.
2. Glasson DW, Lovie MJ. The ulnar island flap in hand and
forearm reconstruction. Br J Plast Surg 1988;41:349-53.

2010 Self-Assessment Examination | 37


85. After performing the procedure depicted, there is an Discussion: Gout is an inflammatory arthritis that is caused
ischemic event noted in the forearm. What is the most by crystallized monosodium urate (MSU) in or around the
likely muscle group to sustain permanent damage? joints. Arthroscopic findings of gout include diffuse synovitis
with crystalline deposits, often in the intercarpal ligaments.
A. Deep finger flexors
The crystals dissolve in formalin and therefore should be
B. Superficial finger flexors biopsied and sent in alcohol. Steroid can also be seen as
C. Finger extensors intraarticular crystals and therefore biopsy for diagnosis of
D. Pronator Teres gout is important.
E. Wrist flexors References:
1. Fitzgerald BT, Setty A, Mudgal CS. Gout affecting the hand and
Preferred Response: A
wrist. JAAOS 2007;15:625-635.
Discussion: FDP and FPL are the most vulnerable muscle 2. Wilczynski MC, Gelberman RH, Adams A, Goldfarb CA.
group in ischemic contracture. In the mildest form, these Arthroscopic findings in gout of the wrist. J Hand Surg
muscles are the only affected, with the ring and middle 2009;34A:244-250.
being affected first. FDS, Pronator teres and wrist flexors
are all involved with more severe or prolonged ischemia.
Only in the most severe cases are the extensors affected.
Reference:
1. Stevanovic M, Sharpe F. Management of established
Volkmann’s contracture of the forearm in children. Hand Clin
2006;22:99-111.
2. Hovius SE, Ultee J. Volkmann’s ischemic contracture.
Prevention and treatment. Hand Clin 2000;16:647-57.

Figure 1

87. What is the mechanism of action of bone


morphogenic protein (BMP) in the treatment of a
fracture nonunion?
A. Increase the cross linking of collagen in callus
B. Modulate the activity of mesenchymal stem cells
C. Provide a structural scaffolding for new bone formation
D. Decrease bone resorption via osteoclast apoptosis
E. Inhibit formation of type II collagen
Preferred Response: B
Discussion: BMP’s are proteins that are members of the
Figure 1 Figure 2 larger family of proteins called TGF-beta [transforming
growth factors] that act specifically on bone. While the
precise mechanism of induction of bone by BMP’s is under
investigation, it has been determined that these proteins act
86. A 57 year-old male with wrist pain undergoes wrist by modulating the activity of mesenchymal stem cells. They
arthroscopy at an outpatient center. Arthroscopic are not believed to increase the cross linking of collagen
findings are shown in Figure 1. In order to confirm or decrease the production of type II collagen, which is
the diagnosis, the optimal solution the synovial extruded in the matrix by chondrocytes early in bone
biopsy specimen should be sent in is: healing. Apoptosis, or programmed cell death, is an action
A. Sterile water of certain BMP’s in adipogenic and myogenic cell lines, not
osteoclasts. BMP’s had no structural properties and are
B. Saline
often added to structural materials in clinical applications.
C. Formalin
References:
D. Hydrogen peroxide
1. Einhorn TA, Lee CA. Bone regeneration: New findings and
E. Alcohol potential clinical applications. JAAOS 2001;9:157-165
Preferred Response: E 2. Geissler WB. Bone graft substitutes in the upper extremity.
Hand Clin 2006;22:329-339.

38 | American Society for Surgery of the Hand


88. The preferred study design for determining the
reliability of a new test is:
A. Randomized controlled trial
B. Retrospective cohort trial
C. Prospective cohort trial
D. Case-control study
E. Expert opinion
Preferred Response: C
Discussion: A randomized, controlled trial controls the
timing and the amount of intervention. A prospective cohort
study looks at individuals with a positive test, and then
applies a gold standard; while a retrospective study looks
back at individuals with a positive gold standard for a history
of prior positive test. In a case-control study, patients with a
disease are compared against controls without the disease Figure 1
for presence of a positive test. Expert opinion is weak
evidence compared with the other options available.
References: 90. A 38 year-old factory worker sustained a laceration
1. Spindler KP, Kuhn JE, Dunn W, Matthews CE, Harrell FE Jr, from a piece of sheet metal resulting in numbness
Dittus RS. Reading and reviewing the orthopaedic literature: a
in the following distribution image. This most likely
systematic, evidence-based medicine approach. J Am Acad
Orthop Surg 2005;13:220-9. represents a laceration of which nerve:
2. Bryant DM, Willits K, Hanson BP. Principles of designing a A. Ulnar
cohort study in orthopaedics. J Bone Joint Surg Am 2009;91
Suppl 3:10-4. B. Lateral Antebrachial Cutaneous
C. Medial Antebrachial Cutaneous
D. Superficial Radial
89. What is the preferred treatment for the lesions E. Posterior Interosseous
seen below?
Preferred Response: D
A. Observation
Discussion: The area of numbness closely corresponds to
B. Prednisone
the superficial sensory branch of the radial nerve. This nerve
C. Radiation emerges from beneath the brachioradialis approximately 9
D. Amputation cm proximal to the radial styloid. The cutaneous innervation
E. Chemotherapy of the lateral antebrachial cutaneous nerve is more proximal.

Preferred Response: A References:


1. Netter FH. Atlas of Human Anatomy, Summit, NJ: CIBA-GEIGY,
Discussion: The radiographs demonstrate several fairly 1991: 151.
symmetric foci of sclerotic lesions centered in the bones 2. Green D. In: Green DP, Hotchkiss RN, Pederson WC, Wolfe SW,
without cortical disruption or periosteal reaction. These eds. Green’s Operative Hand Surgery. 5th ed. Philadelphia:
do not extend beyond the margins of the bone and are Churchill Livingstone/Elsevier, 2005:1033-141.
consistent with the findings of multiple benign bone islands,
osteopoikilosis. Differential diagnosis includes osteoblastic
metastases, principally prostate in men and breast in
women. Multiple myeloma rarely results in sclerotic lesions.
Osteopoikilosis is a benign and nonprogressive disorder of
uncertain etiology and any concern for metastatic disease
can be addressed with the use of a bone scan which will
not demonstrate increased uptake in osteopoikilosis.
References:
1. McLennan M. Radiology Rounds: Osteopoikilosis Can Fam
Physician 1999;45:2318-20.
2. Greenspan A. Bone island (enostosis): current concept—
a review. Skeletal Radiol Feb 1995;24(2):111-5.
Figure 1

2010 Self-Assessment Examination | 39


91. A 43 year-old woman presents to the emergency 92. A 58 year-old laborer presents with a tumor on
room with a distal radius fracture after a fall on her the dorsum of the hand. He is referred by the
outstretched hand. She has a past medical history dermatologist who has done a biopsy which yields
significant for Crohn’s disease and is on long-term the diagnosis of invasive squamous cell carcinoma.
corticosteroids. Open reduction and internal fixation The lesion is 2 centimeters in diameter and lies in
of her distal radius fracture is indicated. Which a bed of sun-damaged skin. A specialist trained
vitamin can help reverse the detrimental effect of in Moh’s micrographic surgery is not available
corticosteroids on wound healing? to participate in this patient’s care. Which of the
following treatments is most appropriate?
A. A
B. C A. Cryosurgery with liquid nitrogen
C. D B. Photodynamic therapy
D. E C. Radiation therapy
E. K D. Surgical excision
E. Topical 5-fluorouracil application
Preferred Response: A
Preferred Response: D
Discussion: Glucocorticoids (corticosteroids) cause dehiscence
of surgical incisions, increased risk of wound infection, and Discussion: The key to successful use of cryosurgery for
delayed healing of open wounds. They produce these effects squamous cell carcinoma is appropriate tumor selection,
by interfering with inflammation, fibroblast proliferation, collagen with the fewest recurrences occurring with original lesions
synthesis and degradation, deposition of connective tissue that are small (<=1 cm), well-defined, well-differentiated, and
ground substances, angiogenesis, wound contraction, and re- either in situ or only superficially invasive on biopsy. This
epithelialization. These actions are mediated by the antagonism lesion, as presented, is too advanced to treat in this manner.
of various growth factors and cytokines. Vitamin A restores
The proper treatment for this lesion is surgical excision
the inflammatory response and promotes epithelialization
with an adequate margin. Current practice is based on the
and the synthesis of collagen and ground substances.
pivotal study from Brodland and Zitelli, which demonstrated
However, vitamin A does not reverse the detrimental effects
that 6 mm margins are required for high risk squamous
of glucocorticoids on wound contraction and infection. Oral
cell carcinoma (in this study, high-risk lesions were defined
ingestion of 25,000 I.U. per day or topical application of 200,000
as those with size >=2 cm, poor differentiation, invasion
I.U. ointments every 8 hours is effective in most cases.
into subcutaneous fat, or location on high-risk areas
Vitamin C – ascorbic acid, is an essential cofactor in the including the hands) to achieve 95% clearance. Using these
synthesis of collagen, and as such, is critically important for guidelines for margins, the 5-year cure rate of primary
wound healing. It does not, however, reverse the detrimental squamous cell carcinoma is 92%.
effects of corticosteroids. Vitamin D is critical to bone
Photodynamic therapy is frequently used to treat actinic
healing, mineral homeostasis and bone integrity, but also
keratoses. Photodynamic therapy involves the application
for numerous physiologic functions including regulation
of a topical photosensitizer with subsequent exposure of
of growth and differentiation in a broad variety of normal
the lesion to visible light, resulting in damage to neoplastic
and malignant tissues. Vitamin E has been used to control
cells. Based on current evidence, photodynamic therapy is
various problems in wound overhealing and large doses
recommended only for squamous cell carcinoma in situ or
of Vitamin E can inhibit healing. The mechanism of this
in nonsurgical candidates. It would not be appropriate in
action appears to have something to do with its membrane
this case of an invasive squamous cell cancer.
stabilizing properties. Vitamin E may improve wound healing
in the setting of irradiated wounds, but not in the setting Radiation therapy is an important alternative to surgical
of long-term corticosteroids. Vitamin K is useful in the treatment of squamous cell carcinoma, especially for older
coagulation cascade and affects bleeding, and clotting. It patients, patients with multiple comorbidities, and for treatment
may also play some role in bone loss and bone turnover but of large lesions that are not amenable to surgical excision.
the effect is not related to corticosteroids. The 5-year cure rate for primary squamous cell carcinoma is
approximately 90%. Radiation therapy lacks margin control,
References:
has a prolonged course of treatment, and increases the risk
1. Anstead GM. Steroids, retinoids, and wound healing. Adv
of future squamous cell carcinoma within the radiation field.
Wound Care 1998;11(6):277-85.
2. Reichrath J, Lehmann B, Carlberg C, Varani J, Zouboulis CC. Several studies have shown success in treating squamous
Vitamins as hormones. Horm Metab Res 2007;39(2):71-84. cell carcinoma in situ with topical 5-fluorouracil however, it
3. Cheung AM, Tile L, Lee Y, Tomlinson G, Hawker G, Scher is currently only Food and Drug Administration–approved to
J, Hu H, Vieth R, Thompson L, Jamal S, Josse R. Vitamin K treat actinic keratosis and superficial basal cell carcinoma.
supplementation in postmenopausal women with osteopenia
(ECKO trial): a randomized controlled trial. PLoS Med 2008 Oct Moh’s micrographic surgery is an appropriate option when
14;5(10):e196. Erratum in: PLoS Med 2008 Dec;5(12):e247. available to the patient.

40 | American Society for Surgery of the Hand


References: 94. A 23 year-old woman presents with clinical findings
1. Hepper DM, Hepper CT, Anadkat M. Treatment options for of DeQuervain’s tenosynovitis in her right wrist. She
squamous cell carcinoma of the dorsal hand including Mohs recently delivered her first child and is currently
micrographic surgery. J Hand Surg 2009;34A:1337-9. breast-feeding. She is asking about treatment
2. Lane JE, Kent DE. Surgical margins in the treatment of options; both surgical and non-surgical. The option
nonmelanoma skin cancer and mohs micrographic surgery. with the best chance of improving her symptoms at
Curr Sur 2005;62(5):518-26.
this time is:
A. Cortisone injection
93. A 35 year-old professional boxer complains of pain B. Iontophoresis
and fullness over the 2nd and 3rd carpometacarpal C. Occupational therapy
joints of the right hand. X-rays demonstrate chronic
D. Splinting
CMC subluxation of these joints with traumatic
carpal bossing. The recommended treatment is: E. Cold laser therapy

A. Arthrodesis of the affected joints Preferred Response: A


B. Extensor tendon realignment Discussion: DeQuervain’s tenosynovitis, which involves
C. Interposition arthroplasty the abductor pollicis longus and extensor pollicis brevis
tendons, is much more common in women than men.
D. Wedge excision of the carpal boss
Housewives and persons involved in manual occupations
E. Wrist splint using the hands and wrists account for most cases in
Preferred Response: A previous series. In one series, six of 24 female patients
(25%) were pregnant or postpartum at the time of onset.
Discussion: Traumatic carpal bossing is a disabling In five of the six, activities of infant care aggravated
and crippling injury in professional boxing if not treated symptoms. Both pregnancy, per se, and mechanical factors
appropriately. Arthrodesis of the destabilized carpometacarpal appear to play a role in causing this condition.
joints has been the preferred method of treatment. Precisely-
executed operative treatment of both injuries has resulted DeQuervain’s disease of pregnancy and lactation is usually
in a favorable outcome, as in the vast majority of cases self-limited and responds well to nonsurgical treatment.
the boxers have experienced relief of pain, restoration of In one study, one group had a cortisone injection into the
function, and an unrestricted return to competition. tendon sheath and the other group used thumb spica
splints. All 9 patients with injections had complete pain relief
Wedge excision is for carpal bossing without subluxation. with only one late recurrence. None of the patients with
It would not be appropriate in this patient since there is splints had complete pain relief; however, at the end of the
subluxation and would likely lead to significant instability lactation period, 8 had spontaneous resolution of symptoms
leading to further pain and instability. The central rigid and 1 received a cortisone injection.
pillar of the hand is made up of the index and middle
metacarpals, which are firmly joined through their irregularly References:
shaped carpometacarpal articulations and intermetacarpal 1. Avci S, Yilmaz C, Sayli U. Comparison of nonsurgical treatment
ligaments to the relatively immobile distal carpal row. measures for de Quervain’s disease of pregnancy and lactation.
J Hand Surg 2002;27A:322-4.
Extensor tendon realignment can be useful in cases of 2. Peters-Veluthamaningal C, van der Windt DA, Winters JC,
tendon attrition from synovitis, such as in a case of an Meyboom-de Jong B. Corticosteroid injection for de Quervain’s
os styloideum, but in this setting there is no mention of tenosynovitis. Cochrane Database Syst Rev 2009
Jul 8;(3):CD005616. Review.
tendinopathy or synovitis. Interposition arthroplasty is not
described for this condition and likely would not be strong 3. Peters-Veluthamaningal C, Winters JC, Groenier KH,
Meyboom-DeJong B. Randomised controlled trial of local
enough in a boxer to allow function. corticosteroid injections for de Quervain’s tenosynovitis in
References: general practice. BMC Musculoskelet Disord 2009;10:131.

1. Melone CP Jr, Polatsch DB, Beldner S. Disabling hand injuries 4. Schned ES. DeQuervain’s tenosynovitis in pregnant and
in boxing: boxer’s knuckle and traumatic carpal boss. Clin postpartum women. Obstet Gynecol 1986;68(3):411-4.
Sports Med 2009;28(4):609-621.
2. Citteur JME, Ritt MJPF, Bos KE. Disabling hand injuries in
boxing: boxer’s knuckle and traumatic carpal boss.
J Hand Surg 1998;23B:76-78.
3. Flatt AE. The care of minor hand injuries, 3rd ed. St. Louis: C V
Mosby Company, 1972: 3.
4. Joseph RB, Linscheid RL, Dobyns JH, Bryan RS. Chronic sprains
of the carpometacarpal joints. J Hand Surg 1981;6:172-180.

2010 Self-Assessment Examination | 41


95. A 65 year-old woman with rheumatoid arthritis Discussion: A Venous-Flow-Through-Flap (VFTF) is a
presents on referral from her rheumatologist because novel technique of microsurgical free tissue transfer that
she has lost the ability to extend the fingers of her right is appropriate for use in certain cases requiring soft-tissue
hand. On clinical examination, she has full passive coverage. VFTF are unusual, but are gaining acceptance in
metacarpophalangeal motion with intact tenodesis. certain kinds of hand and finger wounds. The ideal site for
What is the most likely cause of her disability? coverage with a VFTF is a long narrow defect needing thin soft
tissue. VFTF typically get congested in the first week, and then
A. Atlanto-axial subluxation
decongest over the following two weeks as they revascularize
B. Cubital tunnel syndrome from the wound bed. VFTF cannot reliably transfer composite
C. Extensor tendon attritional ruptures tissue such as bone and tendon or cover a wide defect such
D. Intrinsic tightness as an entire palm. Because they do not bring in as good
vascularization to the wound bed as a classical flap, they are
E. Posterior interosseous nerve entrapment
not indicated in radiated or potentially infected wound beds.
Preferred Response: E A small defect such as the 2 cm2 defect needing coverage
during a finger replant is the ideal candidate for this flap.
Discussion: Posterior interosseous nerve (PIN) entrapment
is a rare complication of rheumatoid arthritis (RA) References:
which, together with extensor tendon rupture and 1. Brooks, D. Invited Discussion: Arterialized venous free flaps.
metacarpophangeal joint dislocation, should be considered J Reconstr Microsurg 2002 Oct;18(7):575-7.
in the differential diagnosis of inability to extend the fingers. 2. De Lorenzi F, van der Hulst RR, den Dunnen WF, Vranckx JJ,
The inability to extend the thumb in PIN entrapment is a Vandenhof B, Francois C, Boeckx WD. Arterialized venous free
useful distinguishing clue on physical examination, and flaps for soft-tissue reconstruction of digits: a 40-case series.
J Reconstr Microsurg 2002;18(7):569-74.
nerve conduction studies confirm the diagnosis. Elbow
3. Titley OG, Chester DL, Park AJ. A-a type, arterialized, venous,
joint synovitis and compression of the PIN at the arcade
flow-through, free flap for simultaneous digital revascularization
of Frohse are the main reasons for PIN entrapment in RA. and soft tissue reconstruction-revisited. Ann Plast Surg
Intraarticular steroid injections and surgical intervention 2004;53(2):185-91.
resolve symptoms of PIN entrapment in RA. 4. Rose EH. Small flap coverage of hand and digit defects. Clin
Plast Surg 1989;16(3):427-42.
Atlanto-axial subluxation of the cervical spine can occur with
RA and is a life-threatening condition. It does not present as
an isolated peripheral nerve injury. Cubital tunnel syndrome
causes ulnar nerve weakness, and would not result in 97. Which of the following studies provides early indication
inability to extend the fingers. The extensor tendons are likely of eventual demarcation in severe frostbite injuries?
not ruptured in this case because the caput ulnae is mild and A. Angiography
the tenodesis effect is intact, suggesting that the fingers can
B. Doppler laser
be passively extended if the wrist is flexed.
C. Technetium 99 bone scan
References:
D. Thermography
1. Fernandez AM, Tiku ML. Posterior interosseous nerve
entrapment in rheumatoid arthritis. Semin Arthritis Rheum 1994 E. Ultrasound
Aug;24(1):57-60.
Preferred Response: C
2. White SH, Goodfellow JW, Mowat A. Posterior interosseous
nerve palsy in rheumatoid arthritis. J Bone Joint Surg Discussion: Several diagnostic tests have been used to
1988;70B:468-71. help predict the severity and prognosis of the frostbite
3. Westkaemper JG, Varitimidis SE, Sotereanos DG. Posterior injury including plain radiographs, infrared thermography,
interosseous nerve palsy in a patient with rheumatoid synovitis angiography, double and triple phase bone scanning, laser
of the elbow: a case report and review of the literature. J Hand
Surg 1999;24A:727-31. Doppler studies, digital plethysmography and magnetic
resonance imaging/magnetic resonance angiography
(MRI/MRA). The most promising studies have used three
phase bone scanning and MRI/MRA. Bone scanning (using
96. Which of the following would be an appropriate use 99
technetium) has now become the standard study used
of a Venous-Flow-Through-Flap?
within the first few days after injury. This technique assesses
A. Composite defects of bone, tendon and skin tissue viability in an effort to allow early debridement of soft
B. Closure of a large skin cancer defect tissue and early coverage of ischemic bony structures.

C. Resurfacing a dorsal hand wound in a radiated bed Scintigraphic methods using technetium 99m pertechnetate
D. Closure of a small wound during a finger replant seem to be the most successful to delineate eventual
amputation necessity and level. The results of one study
E. Coverage of a palmar degloving injury
show that an initial bone scan (as early as day 3) has excellent
Preferred Response: D specificity in evaluating the severity of frostbite injury.

42 | American Society for Surgery of the Hand


There was a direct correlation between the demarcation 2. Beaton DE, Katz JN, Fossel AH, Wright JG, Tarasuk V,
zone of uptake in the phalanges and the eventual level of Bombardier C. Measuring the whole or the parts? Validity,
reliability, and responsiveness of the Disabilities of the Arm,
amputation (positive predictive value, 0.84). A second scan on
Shoulder and Hand outcome measure in different regions of the
approximately day 7 was even more sensitive and informative. upper extremity. J Hand Ther 2001;14(2):128-46.
A strong correlation existed between positive uptake and 3. Gummesson C, Atroshi I, Ekdahl C. The disabilities of the arm,
eventual healing (negative predictive value, 0.99). This study shoulder and hand (DASH) outcome questionnaire: longitudinal
showed that 99mTc bone scanning in the first few days after construct validity and measuring self-rated health change after
frostbite injury indicates the level of amputation in severe surgery. BMC Musculoskelet Disord 2003 Jun 16;4:11.
frostbite in more than 84% of cases.
References:
99. Which of the following factors related to open distal
1. Cauchy E, Marsigny B, Allamel G, Verhellen R, Chetaille E. The
value of technetium 99 scintigraphy in the prognosis of amputation radius fractures has the strongest correlation with
in severe frostbite injuries of the extremities: A retrospective study the risk of post-op infection?
of 92 severe frostbite injuries. J Hand Surg 2000;25A:969-78.
A. Degree of contamination
2. Imray C, Grieve A, Dhillon S. Caudwell Xtreme Everest Research
Group. Cold damage to the extremities: frostbite and non- B. Gustillo classification
freezing cold injuries. Postgrad Med J 2009;85(1007):481-8. C. Time to initial debridement
D. Systemic illness
E. Method of fixation
98. Which outcome instrument is best for evaluating
patient disorders that involve multiple joints of the Preferred Response: A
upper limb?
Discussion: Of all the above possible factors relating to
A. Brigham and Women’s carpal tunnel questionnaire risk of post-operative infection, the strongest correlation
B. DASH score (disability of arm, shoulder and hand is with significant contamination. Contamination type
questionnaire) is usually categorized by the CDC classification: clean,
clean-contaminated, contaminated and dirty/infected.
C. Gartland and Werley score
Significant contamination corresponds to contaminated and
D. PRWE score (patient-rated wrist evaluation dirty/infected wounds and is found to have the strongest
questionnaire) correlation with risk of post-op infection.
E. SPADI (shoulder pain and disability index)
The Gustillo and Anderson fracture classification has been
Preferred Response: B found to have some correlation with open fracture infection
rates, but this is not as strong a correlation in the distal
Discussion: The choice of a proper validated instrument is
radius as it is in long bones of the lower extremity.
critical when trying to formulate and conduct an outcomes
study. The PRWE score is the most responsive instrument Recent studies by Glueck et al. and Swanson et al. have
for evaluating the outcome in patients with distal radius shown that contamination is a much stronger correlate to
fractures. SPADI is useful for shoulder dysfunction, while the risk of post-operative infection.
DASH score is the best instrument for evaluating patients
The timing of the initial debridement, as long as it was within
with disorders involving multiple joints of the upper limb. The
the first 24 hours of injury, does not significantly affect
Brigham and Women’s score is a disease-specific outcome
infection rate. In contrast to this, there is some evidence
instrument for carpal tunnel syndrome; it has been validated
to suggest that performing multiple serial debridements
and demonstrated to show good responsiveness and reliability
in significantly comtaminated wounds is of benefit in
in evaluating outcome in patients with carpal tunnel release.
preventing post-op infection.
The Gartland and Werley score, although the most commonly
described instrument in the literature for evaluating outcome The presence of systemic illness, such as diabetes mellitus,
after wrist surgery, has not been validated so to date. and the type of fracture fixation, seems to have a modest
correlation to post-op infections in distal radius fracutres,
The Disabilities of the Arm, Shoulder and Hand (DASH)
but the findings are generally not statistically significant.
outcome measure was developed to evaluate disability
and symptoms in single or multiple disorders of the upper References:
limb at one point or at many points in time. DASH has 1. Glueck DA, Charoglu CP, Lawton JN. Factors associated with
demonstrated reliability, validity, and responsiveness in infection following open distal radius fractures. Hand (NY)
a group of diverse patients and compares favorably with 2009;4:330-334.
results obtained with joint-specific measures. 2. Swanson TV, Szabo RM, Anderson DD. Open hand fractures:
prognosis and classification. J Hand Surg 1991;16A:101-107.
References: 3. Rozental TD, Beredjiklian PK, Steinberg DR, Bozentka DJ. Open
1. Changulani M, Okonkwo U, Keswani T, Kalairajah Y. Outcome fractures of the distal radius. J Hand Surg 2002;27A:77-85.
evaluation measures for wrist and hand: which one to choose?
Int Orthop 2008 Feb;32(1):1-6. Epub 2007 May 30. Review.

2010 Self-Assessment Examination | 43


100. The hand finding most commonly associated with References:
the condition depicted in Figure 1 is: 1. Kay SPJ. Hypoplastic and absent digits. In: Green DP, Hotchkiss
RN, Pedersen WC, eds. Green’s Operative Hand Surgery. 4th
A. Normal hand ed. Philadelphia: Churchill Livingstone, 1999:368-369.
B. Hypoplastic hand 2. Flatt AE. The Care of Congenital Hand Anomalies. 2nd ed.
C. Radial club hand St Louis: Quality Medical Publishers, 1994: 337-365.
3. Al-Qattan MM. Classification of hand anomalies in Poland’s
D. Brachysyndactyly syndrome. Br J Plast Surg 2001 Mar;54(2):132-6. Review.
E. Transverse deficiency proximal to the MCP joints

Preferred Response: B
Discussion: Poland’s syndrome is absence of the sternal
head of the pectoralis major muscle. The syndrome is
classified according to the degree of involvement of the
hand and rays. The classic presentation, although not the most
common, is a Type III with the associated brachysyndactyly,
which is occasionally found. The most common finding is a
hypoplastic hand. Treatment is dependent on the degree of
hypoplasia of the hand and digits.

Classification of Hand Anomalies in Poland’s Syndrome


TYPE CLASSIFICATION FINDINGS Figure 1
I Normal hand Isolated pectoral anomaly
II Form fruste deformity Hypoplastic hand
III The “classic” Brachysyndactyly with five
deformity potentially functional rays:
A. Mild to moderate
B. Severe
IV Some functional rays A. A  radial club hand with
still present floating or absent thumb
B. Adactyly of the index
C. Adactyly of the index and
long fingers
D. A  dactyly of the central
rays creating a cleft hand
E. Adactyly of the ulnar rays
V All digits are
functionless or absent
VI Transverse deficiency
proximal to the
metacarpophalangeal
joints
VII Phocomelia-like
deficiency

44 | American Society for Surgery of the Hand


101. Which of the following treatment options is not 102. The scaphoid cortical ring (SCR) sign is abnormal if
effective in arresting the longitudinal overgrowth in present with the wrist in neutral (flexion/extension)
the 9-month-old child with the condition depicted in and what degree of radial or ulnar deviation:
Figures 1 and 2?
A. 10° or greater of radial deviation
A. Epiphysiodesis B. 5 to 9° of radial deviation
B. Epiphysectomy C. 13° or greater of ulnar deviation
C. Digital neurectomy D. 10° or less of ulnar deviation
D. Digital shortening E. Neutral radial / ulnar deviation
E. Debulking
Preferred Response: C
Preferred Response: C
Discussion: There is some confusion surrounding the
Discussion: A retrospective review was performed for 23 clinical significance of the SCR sign. The SCR sign however,
patients who were surgically treated for macrodactyly of is normally observed when the wrist moves into radial
the hand. Although the authors found it difficult to compare deviation from ulnar deviation. It is due to the vertical
treatment regimens, epiphysiodesis/epiphysectomy was posture of the scaphoid as the two poles are superimposed
quite effective in the prevention of longitudinal overgrowth on each other on the PA view. The functional status of
of the digits, but resection of the hypertrophic nerves the scapholunate (S-L) ligament has a direct bearing on
(digital neurectomy) was unsuccessful in preventing the presence of this sign. A substantial portion of the
finger overgrowth. Amputation of the involved digits ligament needs to be torn for the sign to be present. It is,
sometimes has to be considered as a measure to control however, sometimes difficult to determine if the presence
the size. Middle phalangectomy was successful in one of this radiologic sign is abnormal. Moreover, when the
case with a good aesthetic and functional result. Further S-L ligament is completely torn, the scaphoid assumes a
clinical evaluation is obviously required before middle more vertical posture. Partial S-L ligament injuries are still
phalangectomy can be recommended for macrodactyly. a perplexing clinical problem. Therefore, other clinical and
radiographic parameters should be used when evaluating
References:
a patient with a suspected S-L ligament injury. A study
1. Ishida O, Ikuta Y. Long-term results of surgical treatment
was carried out with the wrist in neutral flexion-extension
for macrodactyly of the hand. Plast Reconstr Surg
1998;102(5):1586-90. to determine when the SCR sign should be considered
2. Tan O, et al. Middle phalangectomy: a functional and abnormal. The radiometacarpal angle was used to
aesthetic cure for macrodactyly. Scand J Plast Reconstr Surg determine the radioulnar position of the wrist. This study
2006;40(6):362-5. used the normal wrists of patients to determine when
the SCR sign was absolutely not present based on two
standard deviations from the mean of the wrists evaluated.
Using this rationale, they concluded that if the SCR sign is
observed in 13° of ulnar deviation or greater, it should be
considered abnormal.
References:
1. Pirela-Cruz MA, Hilton ME, Faillace J. Frequency and
characteristics of the scaphoid cortical ring sign. Surg Radiol
Anat 2003;25(5-6):451-4.
2. Abe Y, Doi K, Hattori Y. The clinical significance of the scaphoid
cortical ring sign: a study of normal wrist X-rays. J Hand Surg
2008;33B:126-9.
Figure 1 3. Cautilli GP, Wehbé MA. Scapho-lunate distance and cortical
ring sign. J Hand Surg 1991;16A:501-3.

Figure 2
2010 Self-Assessment Examination | 45
103. A 25 year-old male presents with chronic elbow pain 104. In the patient shown in Figure 1, the minimum
a year after undergoing a radial head implant for recommended time (months) before proceeding to
a comminuted fracture. Intra-operative findings of the second stage reconstruction is:
the radiocapitellar joint are noted in Figure 1. The
A. 2
primary technical reason for this unsatisfactory
outcome is: B. 3
C. 6
A. Implant particulate wear debris
D. 9
B. Implant loosening
E. 12
C. Occult infection
D. Joint “over-stuffing” Preferred Response: B
E. Metallosis Discussion: Scarring of the flexor tendon after repair is
the prime reason for reduced range of motion of the finger
Preferred Response: D and failure. To minimize the problems associated with
scarring, staged flexor tendon reconstruction is performed
Discussion: The surgical treatment for a comminuted
by-passing zone II. A silicone implant (Hunter rod) can
displaced radial head fracture in a young patient is a
help to make the reconstruction successful by creating a
difficult problem. Preservation of the radial head should be
pseudosheath which lays down a more receptive bed for
considered initially. In cases where ORIF is not possible,
the tendon graft. As a general rule, it requires approximately
insertion of a radial head implant should be performed.
three months for an adequate tendon pseudosheath.
The most common primary reason for an unsatisfactory
The second flexor tendon reconstructive stage should be
outcome is “over-stuffing” of the radiocapitellar joint
performed after at least three months, when the scar is soft,
manifested by the wear on the capitellum. Other reasons
mature and the patient has good passive range of motion.
for failure include implant malposition that can lead to an
eccentric wear pattern. Metallosis is a rare cause of implant References:
failure and was not noted in this patient. 1. Freilich AM, Chhabra AB. Secondary flexor tendon
reconstruction, a review. J Hand Surg 2007;32A:1436-42.
References:
2. Vucekovich K, Gloria G, Fiala K. Rehabilitation after flexor
1. Ring D, King G. Radial head arthroplasty with a modular metal tendon repair, reconstruction and tenolysis. Hand Clin
spacer to treat acute traumatic elbow instability. Surgical 2005;21:257-265.
technique. J Bone Joint Surg Am 2008 Mar;90 Suppl 2 Pt 1:63-73.
3. Strickland JW. Delayed treatment of flexor tendon injuries
2. Stuffmann E, Baratz ME. Radial head implant arthroplasty. including grafting. Hand Clin 2005;21:219-43.
J Hand Surg 2009;34A:745-54.

Figure 1

Figure 1

46 | American Society for Surgery of the Hand


105. A 19 year-old male with a 4-week history of a
nondominant elbow dislocation was referred by another
physician after surgical treatment for an elbow injury.
See Figures 1, 2, and 3. The recommended treatment is:
A. Therapy
B. Resection arthroplasty
C. Arthrodesis
D. Open reduction
E. Closed reduction
Preferred Response: D Figure 3

Discussion: This young man and the treating physician have


a complicated problem to address. It is likely that this patient
had an ipsilateral olecranon fracture and an ulnohumeral 106. When elevating the flap in Figure 1, which structure
dislocation. The treatment provided was likely an ORIF of the may tether the pedicle?
olecranon fracture and a closed reduction of the dislocation.
A. Tensor fascia
Despite this treatment, instability of the elbow persists. In a
study by Ruch and Triepel, patients with elbow instability B. Femoral quadratus
six weeks or less were treated with open reduction, hinged C. Inguinal ligament
elbow fixation and early ROM had a reasonable outcome. D. Sartorius muscle
In this study, formal repair or reconstruction of the collateral
E. Abdominal aponeurosis
ligaments was not performed.
References: Preferred Response: D
1. Tashjian RZ, Katarincic JA. Complex elbow instability. J Am
Discussion: Axially-patterned flaps based on single arteries
Acad Orthop Surg 2006;14(5):278-86.
have a high success rate. The groin flap is an axial pattern flap
2. Ring D, Jupiter JB. Compass hinge fixator for acute and chronic
instability of the elbow. Oper Orthop Traumatol 2005;17(2):143-57.
of the cutaneous variety based on the superficial circumflex
iliac artery (SCIA). It provides excellent tissue coverage for
3. Ruch DS, Triepel CR. Hinged elbow fixation for recurrent
instability following fracture dislocation. Injury 2001 Dec;32 the extensor surface of the hand, does not require a split
Suppl 4:SD70-8. thickness skin graft, and is easily tubed to a pedicle. The
SCIA may be tethered by the sartorius muscle in certain
positions. Therefore, careful elevation of the flap is required
to ensure that flow into the groin flap is not compromised.
References:
1. Smith PJ, Foley B, McGregor IA, Jackson IT. The anatomical
basis of the groin flap. Plast Reconstr Surg 1972;49:41-47.
2. Taylor GI. Blood supply of the abdomen revisited, with
emphasis on the superficial inferior epigastric artery:
discussion. Plast Reconstr Surg 1984;74:667-670.

Figure 1

Figure 2 Figure 1

2010 Self-Assessment Examination | 47


107. The flexion contracture of the distal interphalangeal pollicis. Andre-Thomas sign is when the patient attempts to
joint seen in Dupuytren’s disease can be attributed tenodese the extrinsic extensor tendons with wrist flexion,
to the: in an attempt to prevent digit clawing. This maneuver
actually increases the claw deformity.
A. Natatory cord
B. Pretendinous cord References:
1. Omer GE Jr. Ulnar nerve palsy. In: Green DP, ed. Operative hand
C. Retrovascular cord of Thomaine
surgery. New York: Churchill Livingstone, 1988:1535-1554.
D. Distal commissural cord 2. Elhassan B, Steinmann SP. Entrapment neuropathy of the ulnar
E. Vertical cord of Legueu and Juvara nerve. J Am Acad Orthop Surg 2007;15(11):672-81.
3. Kozin SH, Porter S, Clark P, Thoder JJ. The contribution of
Preferred Response: C the intrinsic muscles to grip and pinch strength. J Hand Surg
1999;24A:64-72.
Discussion: The retrovascular fibers of Thomaine are
located dorsal to the neurovascular bundle and thought
to extend to all three phalanges. Although these fibers do
not make up a well developed ligament, they extend distal
to the DIP joint. When diseased, they form a retrovascular
cord-like structure that contributes to DIP joint flexion
deformity. Other digital cords, however, can cause DIP
contracture. The pretendinous cord is involved in MP joint
contracture. The natatory cord causes digital web space
contracture. The distal commissural cord results in thumb
Figure 1
index web space contracture and is an indication of a more
extensive disease process. The vertical cord is the diseased
septum of Legueu and Juvara in the palm that may contribute
to stenosing tenosynovitis in Dupuytren’s disease. 109. A 65 year-old woman presents with a painful round
mass in her palm which she has noticed about 6
References:
weeks ago. Since the time she noticed the mass,
1. McGrouther DA. Dupuytren’s contracture. In: Green DP, she has developed a cord and “tightness” when
Hotchkiss RN, Pedersen WC, eds. Green’s Operative Hand
Surgery. 5th ed. Philadelphia: Churchill Livingstone/Elsevier, she attempts to extend the index and middle
2005:159-185. fingers. There is no history of penetrating trauma or
2. Rayan GM. Palmar facial complex anatomy and pathology in neurologic complaints. She recently has had a PAP
Dupuytren’s disease. Hand Clin 1999;15:73-86. smear after going to her gynecologist for new onset
vaginal bleeding. Her regular x-rays are normal. A
clinical picture of her hand, MRI and gross biopsy
108. The clinical sign depicted in the patient’s left hand specimen are shown (Figures 1-3). Her diagnosis is:
shown in Figure 1 is: A. Ganglion cyst
A. Froment’s sign B. Malignant fibrous hystiocytoma (MFH)
B. Wartenberg’s sign C. Palmar Fasciitis
C. Masse’s sign D. Hemangioma
D. Jeanne’s sign E. Dupuytren’s contracture
E. Andre-Thomas sign
Preferred Response: C
Preferred Response: A
Discussion: Palmar fasciitis and polyarthritis syndrome
Discussion: Figure 1 depicts Froment’s sign, also known as
(PFPAS) is an uncommon paraneoplastic syndrome
Bunnell’s “O” sign. This occurs due to loss of distal thumb
associated with several malignant neoplasms. Murray
tip stability from paralysis of the adductor pollicis and the
and coauthors recently published a series of patients with
flexor pollicis brevis. Wartenberg’s sign is an inability to
this syndrome associated with ovarian cancer. A literature
adduct the extended small finger due to loss of the third
review revealed 10 other cases of this syndrome associated
palmar interossei, which opposes the action of the extensor
with gynecologic cancer. Improvement of the palmar
digiti minimi. Masse’s sign is flattening of the palmar arch
fasciitis and associated arthritis can occur with successful
and loss of ability to elevate the ulnar portion of the hand,
treatment of the carcinoma. A gynecologic examination is
due to paralysis of the opponens digit quinti and decreased
recommended in any woman presenting with the sudden
range of flexion of the small finger metacarpophalangeal
onset of unexplained hand pain, palmar inflammatory
joint. Jeanne’s sign is hyperextension of the thumb MP joint
fasciitis, palmar fibromatosis, and digital contractures.
with gross grip or key pinch due to loss of the adductor

48 | American Society for Surgery of the Hand


References 2. Ellis SJ, Cheng R, Prokopis P, Chetboun A, Wolfe SW,
1. Martorell EA, Murray PM, Peterson JJ, Menke DM, Calamia Athanasian EA, Weiland AJ. Treatment of proximal
KT. Palmar fasciitis and arthritis syndrome associated with interphalangeal dorsal fracture-dislocation injuries with
metastatic ovarian carcinoma: A Report of four cases. J Hand dynamic external fixation: A pins and rubber band system.
Surg 2004;29A:654-660. J Hand Surg 2007;32A:1242-1250.
2. Leslie BM. Palmar fasciitis and polyarthritis associated with a
malignant neoplasm: a paraneoplastic syndrome. Orthopedics
1992;15:1436-1439.

Figure 1

Question 109, Figure 1 Question 109, Figure 2 111. Figure 1 represents the x-rays of the right hand of a
patient brought to the Emergency room after being
struck by a car while riding his motorcycle. The
imaging technique which best defines the injury is:
A. Standard x-rays
B. 30° pronated oblique films
C. 30° supinated oblique films
D. CT scan
Question 109, Figure 3 E. MRI scan
Preferred Response: D

110. The fracture depicted in Figure 1 can be treated with a Discussion: This is an example of a carpometacarpal
number of methods, including dynamic external fixation. (CMC) dislocation (small, ring, long, and index). Veigas has
In all of the methods, using this fixation modality, the investigated the pathomechanics of digital CMC fractures
central concept to achieving a satisfactory result is: and dislocations and has found them to be a complex
injury best imaged by CT scan. While x-rays can often
A. Getting an anatomic reduction with traction reveal considerable information, small fractures and other
B. Maintaining a concentric reduction of the joint information important to treatment can often be missed. The
C. Bone grafting any visible defects on x-rays injuries occur from a combination of axial load and shear and
are common in patients involved in high-force injuries.
D. Keeping the device on for at least 6 weeks
E. Stable fixation of fracture fragments References:
1. Yosida R, Shah M, Veigas SF, Patterson R, Buford M. Anatomy
Preferred Response: B and pathomechanics of ring and small finger carpometacarpal
joint injuries. J Hand Surg 2003;28A:1035-1043.
Discussion: The x-rays depict a pilon fracture of the PIP
2. Nakamura K, Veigas SF, Patterson R. The ligament and skeletal
joint. One of the more useful methods of treatment of this
anatomy of the second through fifth carpometacarpal joints and
injury is use of dynamic traction. There are a number of adjacent structures. J Hand Surg 2001;26A:1016-1029.
variations of this technique, including commercially-available
devices. Central to all techniques is the ability to achieve
and maintain a concentric reduction with the device. Bone
grafting is not essential, or routinely used. Anatomic
reductions, while ideal, may not always be possible. Not
having a perfect reduction does not prevent an acceptable
result. There is not a designated time frame the device must
be left on for it to achieve the desired result.
References:
1. Badia A, Riano F, Ravikoff J, Khouri R, Gonzalez-Hernandez
E, Orbay JL. Dynamic intradigital external fixation for proximal
interphalangeal joint fracture dislocations. J Hand Surg
2005;30A:154-160. Figure 1

2010 Self-Assessment Examination | 49


112. A patient falls off a scaffold onto his right wrist. There 113. For the flap depicted in Figure 1, the modification
is pain, swelling, limited range of motion but no open that will minimize the donor defect is:
wounds or neurologic deficit on exam. Figure 1 is a
A. Vein graft the artery defect at the donor site
radiograph of the injured wrist. Closed reduction is
attempted in the ER, but not successful. After open B. Use a full thickness skin graft at the donor site
reduction is achieved in the operating room, the C. Leave the fascia at the donor site
carpal injury should be treated with: D. Use a split thickness skin graft at the donor site
A. Fusion of the midcarpal joint E. Use a wound VAC for donor site closure
B. Ligament repair of the midcarpal joint with Preferred Response: C
internal fixation
Dicussion: The radial forearm flap is one of the workhorse
C. External fixation of the wrist with no ligament repair
flaps for upper extremity reconstruction. Its main drawback
D. Proximal row carpectomy is the donor site defect, which can sometimes be
E. Cast immobilization problematic. Use of a full thickness vs. split thickness graft
has not been shown to solve the problem of sometimes
Preferred Response: B
poor-take of the graft at the donor site. As long as the
Dicussion: The x-rays reveal a divergent carpal dislocation palmar arch is complete and there is a patent ulnar artery,
with dislocation of the midcarpal joint. Divergent carpal reconstructing the radial artery is not necessary. Raising
dislocations typically result from high force injuries and the flap in a suprafascial plane has shown to be a significant
involve axial load and torsion applied to the carpus. improvement in the donor site by various authors. Wound
While there are no large series of these injuries in the VAC closure of the donor site would not be indicated.
literature, all studies to date advocate open reduction and
References:
ligament repair supplemented by internal fixation. Cast
1. Lutz BS, Wei FC, Sophia CN. Donor site morbidity after
immobilization is insufficient to stabilize these injuries and
suprafascial elevation of the radial forearm flap: a prospective
acute fusions are not indicated. A proximal row carpectomy study in 95 consecutive cases. Plas Recon Surg 1999;
in this situation would not be indicated. External fixation 103:132-137.
without ligament repair has not been reported as a method 2. Bardsley AE, Soutar DS, Elliot D, Batchelor AD. Reducing
of stabilization for this injury pattern. morbidity on the forearm flap donor site. Plas Recon Surg
1990;86:284-287.
References:
1. Herzberg G. Perilunate and axial carpal dislocations and
fracture-dislocations. J Hand Surg 2008;33A:1159-1168.
2. Garcia-Elias M, Dobyns J, Cooney WP III, Linscheid RL.
Traumatic axial dislocations of the carpus. J Hand Surg
1989;14A:446-457.

Figure 1

Figure 1

50 | American Society for Surgery of the Hand


114. The parents of a 12 year-old child are concerned 115. A 55 year-old right hand dominant woman with
about the appearance of the right thumb nail shown advanced bilateral rheumatoid wrist arthritis wishes to
in Figure 1. There is no history of trauma. The child proceed with bilateral staged wrist surgery, after she
does not report any pain, functional loss and other has been unable to obtain relief with oral medication,
than the nail appearance, the physical exam is splinting and injections. On examination, there do not
normal and no other digits are affected. X-rays are appear to be any tendon ruptures of the wrist and finger
normal. The next step in management would be: tendons. There is no active synovitis. She has minimal
pain with forearm rotation. The radial carpal joint is
A. Observation and re-evaluation in 6 months
tender. Wrist motion reproduces her symptoms. Her
B. MRI scan neurologic examination is normal. There is minimal joint
C. IP joint disarticulation involvement and symptoms in the shoulder and elbow.
D. Nail bed excision and full thickness skin graft PA x-rays of the right and left wrist are shown in Figures
1 and 2. She would like to preserve some motion
E. Biopsy
so she can play Guitar Hero with her grandchildren.
Preferred Response: E The surgical plan supported by the current literature
which is consistent with her wishes would be:
Dicussion: With the lesion shown, one would need to
rule out a melanoma, which is very rare in children. It A. Right silicone arthroplasty and left wrist fusion
is important to get a definitive diagnosis in this patient; B. Right wrist fusion and left proximal row carpectomy
therefore, a biopsy would be the best choice. An MRI
C. Right total wrist arthroplasty and left wrist fusion
scan would not give definitive information and observation
alone would be inadequate. The lesion is most likely a D. Right wrist fusion and left silicone arthroplasty
melanonychia striata and proceeding with ablative surgery E. Right proximal row carpectomy and left wrist fusion
would not be the best option.
Preferred Response: C
References:
Dicussion: Rheumatoid arthritis can be a crippling disease
1. Molina D, Sanchez JL. Pigmented longitudinal bands of the nail.
which presently does not have a cure. When patients have
A clinicopathologic study. Am J Dermatopathol 1995;17(6):539-41.
disease progression (despite non-operative measures) and
2. Buka R, Friedman KA, Phelps RG, Silver L, Calero F, Rudikoff
D. Childhood longitudinal melanonychia: case reports and they desire pain-relief and functional improvement, then
review of the literature. Mt Sinai J Med 2001;68(4-5):331-5. surgery is indicated. While wrist fusions have been the “gold
3. Quinn MI, Thompson JE, Crotty K, McCarthy WH. Subungual standard” for end-stage rheumatoid wrist disease, the present
Melanomas of the Hand. J Hand Surg 1996;21A:506-511. generation of wrist replacements appears to have addressed
a number of problems, such as component loosening and
instability (as reported by Adams). Considering the present
literature and patient wishes, the plan of a wrist fusion on
the non-dominant side with arthroplasty on the dominant
side would be appropriate. A proximal row carpectomy and
silicone wrist replacement would be contraindicated.
References:
1. Herzberg G. Management of bilateral advanced rheumatoid
wrist destruction. J Hand Surg 2008;33A:1192-1196.
2. Divelbiss BJ, Sollerman C, Adams BA. Early results of the
universal total wrist arthroplasty in rheumatoid arthritis. J Hand
Surg 2002;27A:195-204.

Figure 1

Figure 1 Figure 2

2010 Self-Assessment Examination | 51


116. A 45 year-old right hand dominant man presents
with a burn contracture of his first web space after
contacting a steam pipe at home. There is 1-2 cm.
ulceration in the mid-web and this is the same hand
which had an open TFC repair one year ago. Pre-
operative angiography shows an incomplete palmar
arch. He wants a single-stage reconstruction. An
appropriate procedure would be:
A. Radial forearm flap
B. Dorsal ulnar artery flap (Becker flap)
Figure 3
C. Double opposing Z-plasty (jumping man flap)
D. 1st dorsal metacarpal artery flap
E. Posterior interosseous flap
117. The cell type associated with the development of a
Preferred Response: B Dupuytren’s contracture is:
Discussion: In the scenario presented, a thumb web A. Myofibroblast
space contracture with unstable skin, a local flap such as
B. Fibroblast
a z-plasty would not give reliable coverage or restore web
span. With an incomplete arch, a radial forearm flap may C. Osteoblast
cause loss of vascularity to all or part of the hand. A 1st D. Dermoblast
dorsal metacarpal artery flap does not provide enough E. Pluripotential stem cell
tissue to resurface the web. A posterior interosseous flap
would be a reasonable choice, but with prior surgery at Preferred Response: A
the ulnar wrist, the vascular connection needed for pedicle Discussion: The Myofibroblast is the cell line which has
transfer may not be present. This leaves the dorsal ulnar been identified as the cell type in a Dupuytren’s contracture.
artery flap as the only viable choice. It is useful to have in
the armamentarium for this problem (Figure 1-3). References:
1. McFarlane RM. The current status of Dupuytren’s disease.
References: J Hand Surg 1983;8:703-708.
1. Page R, Chang J. Reconstruction of hand soft-tissue defects: 2. Hueston JT. Dupuytren’s contracture. In: Flynn JE, ed. Hand
alternatives to the radial forearm fasciocutaneous flap. J Hand surgery. Baltimore: Williams & Wilkins, 1982:797-823.
Surg 2006;31A:847-56.
2. Antonopoulos D, Kang NV, Debono R. Our experience with
the use of the dorsal ulnar artery flap in hand and wrist tissue
cover. J Hand Surg 1997;22B:739-44. 118. A 37 year-old man has been transferred to you in a
posterior splint, one week after undergoing open
reduction and internal fixation of a right elbow
fracture dislocation sustained while break dancing.
The operative report documents anatomic repair of
a 50% vertical height coronoid fracture and a radial
head fracture. The current x-rays show a concentric
reduction. The lateral ulnar collateral ligament (LCL)
was firmly repaired, but the surgeon did not repair the
medial collateral ligament (MCL). Under anesthesia,
the elbow was reported to be stable in an arc of
20° short of full extension to 140° flexion. The most
Figure 1 appropriate program now is:
A. No therapy at present. Continue splinting for 6 weeks
B. No therapy at present. Continue splinting for 12 weeks
C. Return to OR to repair the medial ligaments
D. Begin motion now with the elbow in an arc of -20 to
140 with the forearm pronated
E. Begin motion now with the elbow in an arc of -20 to
140 with the forearm supinated
Preferred Response: E
Figure 2

52 | American Society for Surgery of the Hand


Discussion: The patient has undergone repair for a “terrible References:
triad” injury to his elbow. By definition and the clinical 1. Lozano Calderon S, Paiva A, Ring D. Patient satisfaction after
scenario presented, the medial collateral ligament has open carpal tunnel release correlates with depression. J Hand
been torn. Dr. Forthman has shown these injuries can be Surg 2008;33A:303-307.
managed with early motion, without repairing the medial 2. Pomerance J. Return to work in the setting of upper extremity
collateral ligament. Fracture fixation and lateral collateral illness. J Hand Surg 2009;34A:137-141.
ligament repair allows early motion to commence. Dr.
Matthew and co-authors have reported early motion can
be used in MCL- and LCL-deficient elbows. If the MCL is 120. A 16 year-old male sustains a closed elbow injury
deficient and the LCL has been repaired firmly, keeping the following a dirt bike accident. Radiographs and CT
forearm supinated assists in elbow stabilization. In an LCL- scan of the elbow are obtained (Figures 1 and 2). What
deficient elbow with MCL repair or continuity, the forearm is the recommended treatment for this isolated injury?
should be pronated to aid in stability. Since the LCL has
A. Closed reduction and pinning
been repaired, shoulder abduction should be avoided to
keep stress off the ligament repair. B. Cast immobilization
C. Closed reduction and percutaneous screw fixation
References:
D. Application of hinged brace with early range of motion
1. Forthman C, Henket M, Ring D. Elbow dislocation with
intra-articular fracture: The results of operative treatment E. Open reduction and internal fixation
without repair of the medial collateral ligament. J Hand Surg
2007;32A:1200-1209. Preferred Response: E
2. Matthew P, Athwal G, King GJW. Terrible triad injury of the
elbow: Current Concepts. JAAOS 2009;17:137-151. Discussion: The patient has sustained a fracture of the
lateral column with intra-articular extension. Imaging studies
confirm intra-articular displacement. In order to achieve
119. A 45 year-old part-time cashier is scheduled to anatomic reduction of the joint, an open approach is
undergo a right carpal tunnel release. Her pre- required. Moderate comminution of the fracture including the
operative nerve conduction studies reveal sensory lateral epicondyle is present, requiring plate stabilization.
latencies just above normal values with normal References:
motor studies and a normal EMG. She has had 1. Rizzo M, Nunley JA. Fractures of the elbow’s lateral column
difficulties with co-workers and has been on anti- radial head and capitellum. Hand Clin 2002;18(1):21-42.
depressant medication for over one year. She rates 2. Henley MB, Bone LB, Parker B. Operative management of
her pain as a 10 on a 10 point pain scale. Based on intra-articular fractures of the distal humerus. J Orthop Trauma
current literature, what is her expected outcome? 1987;1:24.

A. It is unclear if her pain complaints and depression will


affect outcomes
B. Placing her on short term disability will resolve her
work issues
C. Her near normal nerve conduction study predicts
consistently good outcomes
D. Factors are present which may negatively affect
surgical outcome
E. While return to work may be delayed, her surgery
outcomes should be equal to other patients with
similar nerve conduction study findings Figure 1
Preferred Response: D
Discussion: The patient presents with multiple factors
which have been shown to negatively affect patient
outcomes and return to work. These would include pain
catastrophizing, depression, and poor coping mechanisms.
Depression and poor coping mechanisms have been shown
to predict patient dissatisfaction after open carpal tunnel
release. Similarly, these same factors, along with pain out of
proportion (pain catastrophizing) prolong return to work.
Figure 2

2010 Self-Assessment Examination | 53


121. A patient undergoes arthroscopic elbow capsular References:
release for a post-traumatic contracture. Post- 1. Szabo RM, MacDermid JC. Evidence-Based Practice. Hand
operatively, the patient reports new onset of altered Clin 2009 Feb;25(1):1-14.
sensation in the ring and small finger. Examination 2. Hatala R, Keitz S, Wyer P, et al. Tips for learners of evidence-
of the operated extremity demonstrates weakness based medicine: 4. Assess heterogeneity of primary studies in
of the ulnar innervated intrinsic muscles and absent systematic reviews and whether to combine their results. CMAJ
2005;172(5):661-5.
sensation in the small finger and ulnar aspect of the
ring finger. The nerve injury most likely occurred
during which portion of the procedure?
123. A 25 year-old male presents 6 weeks following
A. Arthroscopic anterior capsular release traumatic amputation to his dominant hand
B. Establishment of the proximal anteromedial portal (Figure 1). Radiographs of the involved hand are
C. Establishment of the posterocentral portal demonstrated (Figure 2). Which of the following is
the best reconstructive option for the thumb?
D. Arthroscopic release of the posterior band of the
medial collateral ligament A. Distraction lengthening
E. Osteophyte debridement of the posterior B. Radial forearm flap
lateral olecranon C. Web Space deepening
Preferred Response: D D. Toe-to-thumb transfer
Discussion: In cases of severe elbow flexion contracture, E. Osteoplastic thumb reconstruction
scarring of the ulnar nerve may take place between the Preferred Response: D
ulnar nerve and the posterior band of the medial collateral
ligament. Arthroscopic release of the posterior band places Discussion: The patient has sustained an amputation
the ulnar nerve at risk. The radial nerve is at greatest risk through the proximal portion of the thumb metacarpal.
during anterior capsular release. The ulnar nerve is in close Soft tissue loss at this level requires complete thumb
proximity to the posterocentral and proximal anteromedial reconstruction. Pollicization is not possible in this case.
portals. However, nerve injury is avoidable with careful Microvascular reconstruction with great toe or second toe
portal placement. transfer is the best option (Figure 3). In cases of amputation
through the middle third of the thumb: metacarpal
References: lengthening, web space deepening, dorsal rotational flap or
1. Evans PJ, Nandi S, Maschke S, Hoyen HA, Lawton JN. Prevention osteoplastic thumb reconstruction can be considered.
and treatment of elbow stiffness. J Hand Surg 2009;34A:769-778.
2. O’Driscoll SW. Arthroscopic osteocapsular arthroplasty. References:
In: Yamaguchi K, King GJW, McKee MD, O’Driscoll SW, eds. 1. Shin AY, Bishop AT, Berger RA. Microvascular reconstruction of
Advanced reconstruction elbow. Rosemont, IL: American the traumatized thumb. Hand Clin 1999;15:347-371.
Academy of Orthopaedic Surgeons, 2007:59-68. 2. Chung KC, Wei FC. An outcome study of thumb reconstruction
using microvascular toe transfer. J Hand Surg 2000;25A:651-658.

122. Which of the following study designs is consistent


with the highest level of evidence (Level I)?
A. Case Series without controls
B. Meta-analysis of randomized trials
C. Expert opinion
D. Prospective cohort series
E. Meta-analysis of case controlled studies
Preferred Response: B
Discussion: Five levels of quality of evidence have been created
to assess studies. The highest level of evidence (Level I) is
represented by large randomized clinical trials or meta-analysis
of randomized trials with low error risk. Level II evidence comes Question 123, Figure 1
from randomized trials with high error risk or prospective cohort.
Level III is represented from case-controlled studies. Level
IV is represented by case series without controls. Expert
opinion is consistent with level V evidence.

54 | American Society for Surgery of the Hand


References:
1. Braun RM, Vise GT. Sublimus-to-profundus tendon transfers in the
hemiplegic upper extremity. J Bone Joint Surg 1973;55A:873.
2. Hisey MS, Keenan ME. Orthopaedic management of upper
extremity dysfunction following stroke or brain injury. In: Green
DP, Hotchkiss RN, Pederson WC, eds. Green’s Operative
Hand Surgery. 4th ed. Philadelphia: Churchill Livingstone,
1999:316-318.

125. A 45 year-old male sustained an eccentric load to


the arm two days ago. He presents with pain and
ecchymosis within the antecubital fossa of the elbow.
An MRI of the elbow was obtained (Figure 1). The
axial image demonstrates which of the following?
A. Brachialis strain
Question 123, Figure 2
B. Normal appearance of the distal biceps tendon
C. Partial tearing of the distal biceps tendon
D. Complete tearing of the distal biceps tendon
E. Disruption of the lacertus fibrosis
Preferred Response: C
Discussion: MRI imaging for distal biceps tendon injuries
may be useful when clinical exam is equivocal. The axial
image of the radial tuberosity is the most helpful view to
define distal biceps pathology. Figure 1 demonstrates
increased fluid surrounding the distal biceps with partial
loss of the insertion. Studies have demonstrated high
sensitivity and specificity in the diagnosis of partial and
Question 123, Figure 3 complete tears of the distal biceps utilizing MRI. Normal
appearance of the distal biceps insertion is shown in
Figure 2.
124. A 35 year-old patient sustained a traumatic brain
References:
injury 20 months ago and presents with a clenched
1. Williams BD, Schweitzer ME, et al. Partial tears of the distal
fist. Dynamic EMG demonstrates volitional control
biceps tendon: MR appearance and associated clinical
of the extrinsic flexors. There is no evidence of findings. Skeletal Radiol 2001;30(10):560-4.
skin maceration or wrist flexor contracture on 2. Festa A, Mullieri PJ, Spitz DJ, Leslie BM. Effectiveness of
examination. What is the recommended treatment to magnetic resonance imaging in detecting partial and complete
improve the patient’s hand function? distal biceps tendon rupture. J Hand Surg 2010;35A:77-83.

A. Superficialis-to-profundus tendon transfer


B. Fractional lengthening of the flexor digitorum
profundus tendons
C. Flexor pronator slide
D. Dynamic splinting
E. Phenol injection
Preferred Response: B
Discussion: Fractional lengthening of the long flexors is
the surgical treatment of choice, since volitional control Figure 1 Figure 2
was identified on EMG. A more extensive lengthening with
superficialis-to-profundus transfer is indicated in the absence
of volitional motion and with severe skin breakdown of the
palm. Botulinum toxin may be used in the recovery phase to
modulate spasticity. Phenol causes permanent nerve injury
and is not indicated during the recovery phase.

2010 Self-Assessment Examination | 55


126. Following operative repair of the distal biceps with 127. In a patient following ORIF of a both bones forearm
single incision endobutton fixation, the patient fracture, which factor is most associated with
reports painful numbness within the volar and refracture following hardware removal?
radial portion of the distal forearm. The most likely
A. Transverse fracture of the radius
explanation for the patient’s complaints is an injury
of which of the following nerves? B. Hardware removal after one year
C. Use of 4.5 mm plates
A. Superficial radial nerve
D. Use of 3.5 mm plates
B. Posterior interosseous nerve
E. Use of locking plates and screws
C. Anterior interosseous nerve
D. Lateral antebrachial cutaneous nerve Preferred Response: C
E. Palmar cutaneous branch median nerve Discussion: Hardware removal following both bone
forearm fractures should be strongly discouraged. Mih
Preferred Response: D
and colleagues reported an 11% refracture rate following
Discussion: Performing a single incision endobutton forearm hardware removal. The average time for refracture
technique for a distal biceps repair places the lateral was 6 months. In addition, hardware removal did not
antebrachial cutaneous nerve at risk during the approach. The predictably improve symptoms and there was a four
nerve should be identified with its superficial location exiting times higher complication rate in patients who underwent
between the brachialis and biceps tendon (Figure 1). Injury hardware removal, compared to those that did not. Risk
may occur as result of laceration or secondary to excessive factors for fracture following hardware removal include:
retraction. Although the superficial radial nerve is at risk early plate removal (< 1yr), fractures with initial comminution,
with this procedure, the patient describes symptoms within and the use of 4.5mm plates.
the distribution of the lateral antebrachial cutaneous nerve.
References:
References: 1. Mih AD, Cooney WP, Idler RS, et al. Long-term follow-up
1. Greenberg JA, Fernandez JJ, Wang T, Turner C. Endobutton- of forearm bone diaphyseal plating. Clin Orthop Rel Res
assisted repair of distal biceps tendon ruptures. J Shoulder 1994;299:256-8.
Elbow Surg 2003;12(5):484-490. 2. Moss JP, Bynum DK. Diaphyseal fractures of the radius and
2. Cohen MS. Complications of distal biceps tendon repairs. ulna in adults. Hand Clin 2007;23:143-51.
Sports Med Arthrosc 2008;16(3):148-53.

128. Figures 1-5 demonstrate a 45 year-old male who has


been diagnosed with a plasmacytoma. What is the
most appropriate treatment?
A. Curettage, open reduction and internal fixation,
bone grafting, and radiation
B. Curettage and bone grafting following fracture healing
C. Curettage, phenol and polymethylmethacrylate
D. Chemotherapy, above elbow amputation
E. Chemotherapy, radical wide excision

Preferred Response: A
Figure 1
Discussion: Plasmacytoma is a marrow plasma cell tumor
characterized by overproduction of an intact monoclonal
immunoglobulin, or free monoclonal kappa or lambda
chains. The lesion is expansile and has features of
malignancy. The appropriate treatment for the lesion is to
stabilize the fracture with bone grafting or PMMA. Following
fracture fixation and wound healing, these lesions can be
treated with radiation therapy. Lesions with no fracture
may be treated with radiation alone. Two thirds of solitary
plasmacytomas progress to multiple myeloma.

56 | American Society for Surgery of the Hand


References: Discussion: In a large review of elbow arthroscopy
1. Dimopoulos MA, Hamilos G. Solitary bone plasmacytoma and its complications, Kelly et al noted that the factors
and extramedullary plasmacytoma. Curr Treat Options Oncol significantly associated with increased transient nerve
2002;3:255-9. injury were rheumatoid arthritis, a joint contracture and the
2. Ellis PA, Colls BM. Solitary plasmacytoma of bone: Clinical performance of a capsulotomy. The authors also found
features, treatment and survival. Hematol Oncol 1992;10:207-11. that a prolonged tourniquet time was noted in patients with
nerve complications, but it was not statistically significant.
A history of prior nerve transposition or surgery that
distorts the elbow anatomy is considered by many to be a
contraindication to elbow arthroscopy.
References:
1. Kelly EW, Morrey BF, O’Driscoll SW. Complications of elbow
arthroscopy. J Bone Joint Surg 2001;83A:25-34.
2. Dodson CC, Nho SJ, Williams RJ, Altchek DW. Elbow
arthroscopy. J Am Acad Orthop Surg 2008;16:574-85.

Question 128, Figure 1 Question 128, Figure 2 130. A 48 year-old male presents with acute pain in his
nondominant arm following forced elbow flexion.
Clinical exam demonstrates significant pain with
resisted supination, but no obvious deformity. His
elbow MRI is shown in Figures 1 and 2. What is the
most appropriate initial treatment?
A. Injection
B. Immobilization
C. Surgical debridement
D. Surgical release
Question 128, Figure 3 Question 128, Figure 4 E. Surgical repair
Preferred Response: B
Discussion: Figures 1 and 2 show a partial biceps tendon
rupture. While the definitive surgical treatment remains
debated and depends on the level of patient activity and
functional demands, most surgeons agree that a trial of
non-operative treatment is an excellent first choice in
management. Immobilization for 2-3 weeks has been shown
to be effective followed by protected range of motion.
References:
1. Bourne MH, Morrey BF. Partial ruptures of the distal biceps
Question 128, Figure 5
tendon. Clin Orthop Rel Res 1991;271:143-8.
2. Durr HR, Stabler A, Pfahler M, et al. Partial rupture of the distal
biceps tendon. Clin Orthop Rel Res 2000;374:195-200.
3. Ramsey ML. Distal biceps tendon injuries: Diagnosis and
129. Which of the following conditions or considerations management. JAAOS 1999;7:199-207.
is associated with a higher risk of neurovascular
injury in patients undergoing elbow arthroscopy?
A. Obesity with a BMI greater than 40%
B. Tourniquet time of greater than 1 hour
C. History of prior surgery
D. Elbow joint contracture
E. Osteophytosis of the coronoid
Preferred Response: D

Figure 1 Figure 2

2010 Self-Assessment Examination | 57


131. What characteristic differentiates an aneurysm from 133. What is the typical amount of lengthening that can
a pseudoaneurysm? be obtained with this technique shown in Figure 1?
A. The presence of a thrombus A. 10 mm
B. Dilation of the vessel B. 20 mm
C. The presence of a pulsatile mass C. 30 mm
D. Association with acute or repetitive trauma D. 40 mm
E. Weakening of the internal elastic lamina of the artery E. 50 mm
Preferred Response: E Preferred Response: C
Discussion: Aneurysms may be classified into (1) true or (2) false Discussion: Metacarpal lengthening, with and without bone
or pseudoaneurysms. False aneurysms are generally the result grafting for partial thumb amputations, can help afford
of penetrating trauma, are eccentric, and tend to have a lumen improved length and function in patients. They are generally
that is contiguous with the arterial lumen that has no endothelial indicated for amputations through the middle or proximal
layer lining. True aneurysms are more symmetric and tend to one-third of the phalanx. The average amount of length that
occur in response to a repetitive trauma that results in gradual can be afforded is approximately 30 mm.
dilation of the vessel. They are associated with a weakening of
References:
the internal elastic lamina of the artery. Both may present as a
1. Finsen V, Russwurm H. Metacarpal lengthening after traumatic
dilated vessel, pulsatile mass and have an associated thrombus.
amputation of the thumb. J Bone Joint Surg 1996;78B:133-6.
References: 2. Zimmerman R, et al. Functional outcome with special attention
1. Rush D, Koman A, et al. Other disorders of the upper extremity. to the DASH questionnaire following callus distraction and
In: Green DP, Hotchkiss RN, Pederson WC, Wolfe SW, eds. phalangization of the thumb after traumatic amputation in the
Green’s Operative Hand Surgery. 5th ed. Philadelphia: Churchill middle one-third. Arch Orthop Trauma Surg 2003;123:521-6.
Livingstone/Elsevier, 2005:2300-2.
2. Koman LA, Urbaniak JR. Ulnar artery thrombosis. Hand Clin
1985;3:11-25.

132. Examples of osteoinductive bone graft


substitutes include:
A. Bone morphogenic protein-2
B. Hydroxyapatite
C. Collagen
D. Beta-tricalcium phosphate
E. Marine coral
Preferred Response: A
Discussion: Alternative options to bone autograft include
cadaveric allograft and bone graft substitutes. Bone graft
material may possess osteoconductive and/or osteoinductive
properties. Osteoconductive material is thought to provide Figure 1
a scaffold to support new bone formation in a process
of graft incorporation known as creeping substitution.
Osteoinductive material stimulates a biological response to
differentiate primitive mesenchymal cells into osteoblasts.
Bone morphogenic proteins are known osteogenic proteins.
Hydroxyapatite is a ceramic-containing calcium phosphate
and has no known osteoinductive properties. Collagen,
beta-tricalcium phosphate, and coralline substitutes are all
osteoconductive to varying degrees.
References:
1. Khan SN, Fraser JF, Sandhu HS, et al. Use of osteopromotive
growth factor demineralized bone matrix and ceramics to enhance
spinal fusions. J Am Acad Orthop Surg 2005;13:129-137.
2. Finkemeier CG. Bone-grafting and bone-graft substitutes.
J Bone Joint Surg 2002;84A:454-464. Figure 2

58 | American Society for Surgery of the Hand


134. Which of the following agents is used at the site of a Discussion: Dynamic and static splinting were compared
mechlorethamine chemotherapy extravasation? in a prospective study for zone 5 and 6 extensor tendon
injuries by Mowlavi et al. The authors noted improved short-
A. Dexrazoxane administered intravenously at the site
term range-of-motion and strength in the group treated
away from the extravasation site
with dynamic splinting. However, in the long-term, strength
B. Dimethyl sulfoxide applied topically with a cotton and range-of-motion were not statistically significant.
swab or gauze Dynamic splinting does not appear to result in increased
C. Hyaluronidase injected either through the existing IV complications for injuries in these zones.
or subcutaneously at the extravasation site
References:
D. Sodium thiosulfate with sterile water injected at the 1. Mowlavi A, Burns M, Brown RE. Dynamic versus static splinting
extravasation site of simple zone V and zone VI extensor tendon repairs:
E. Glucocorticoid injection into the extravasation site A prospective, randomized, controlled study. Plast Reconstr
Surg 2005;115:482-7.
Preferred Response: D 2. Chow JA, Dovelle S, Thomes LJ, Ho PK, Saldana J.
A comparison of results of extensor tendon repair followed by
Discussion: Chemotherapy extravasation can lead to a early controlled mobilization versus static immobilization.
significant amount of morbidity, depending on the agent J Hand Surg 1989;14B:18-20.
and the amount infiltrated. Antidotes for some common
chemotherapeutic agents include: (1) Sodium thiosulfate for
mechlorethamine infiltration (and optionally for cisplatin), (2)
136. The optimal treatment for a patient who has a remote
Hyaluronidase injection for plant alkaloids (such as vincristine
history of mallet finger now with a long finger swan
and vinblastine), and (3) topical Dimethyl sulfoxide (DMSO)
neck deformity with 45° hyperextension of the PIP
and intravenous dexrazoxane for anthracyclines.
joint that is passively correctable but cannot be
Vesicant Extravasation Antidotes and Treatment
overcome with active finger flexion is:
Agent Antidote/Treatment A. Arthrodesis of the PIP joint
Mechlorethamine Sodium Thiosulfate B. Reconstruction of the oblique retinacular ligament
Anthacyclines Dimethyl sulfoxide (DMSO) C. Superficialis hemitenodesis of the PIP joint
Dexrazoxane
D. Percutaneous pinning of the DIP joint in flexion
Plant Alkaloids Hyaluronidase
E. Percutaneous pinning of the PIP and DIP joint in extension
References: Preferred Response: B
1. Schulmeister L. Extravasation management. Semin Oncol Nurs
Discussion: The single procedure among the options that
2007;23:184-90.
addresses both the PIP and DIP joints is reconstruction
2. Goolsby TV, Lombardo FA. Extravasation of chemotherapeutic
agents: Prevention and treatment. Semin Oncol 2006;33:139-43. of the oblique retinacular ligament. It will help correct
the hyperextension deformity of the PIP and the flexion
3. Dorr RT. Antidotes to vesicant chemotherapy extravasations.
Blood Rev 1990;4:41-60. deformity of the DIP joint. All of the other treatment options
are either incomplete or incorrect.
References:
135. Postoperative therapy options following the repair 1. Thompson JS, Littler JW, Upton J. The spiral oblique retinacular
of a zone 6 extensor tendon laceration include ligament (SORL). J Hand Surg 1978;3:482-7.
dynamic or static splinting. The literature comparing 2. Kleinman WB, Petersen DP. Oblique retinacular ligament
the outcomes of dynamic versus static splinting reconstruction for chronic mallet finger deformity. J Hand Surg
following the repair of a zone 6 extensor tendon 1984;9:399-404.
laceration reports that dynamic splinting yields:
A. A higher incidence of tendon re-ruptures
B. Better long-term (greater than one year) range-of-motion
C. Better short-term (8 weeks) strength
D. Better long-term (greater than one year) strength
E. Worse long-term (greater than one year) strength
Preferred Response: C

2010 Self-Assessment Examination | 59


137. Which of the following hand conditions or 139. The most likely anatomical explanation for a
deformities is associated with systemic lupus posturally abducted small finger in a patient with a
erythematosis (SLE)? neurotmetic injury to the ulnar nerve 2 cm proximal
to the medial humeral epicondyle is:
A. Ankylosis of the PIP joint
B. Boutonniere deformity of the PIP joint A. Loss of function of the fourth dorsal interosseous muscle
C. Hyperlaxity of the finger joints B. Preservation of flexor digitorum superficialis function
in the small finger
D. Thumb adduction contracture
C. Unopposed pull of the abductor digiti minimi muscle
E. Fixed subluxation and dislocation of the MP joints
D. Preservation of radial nerve function
Preferred Response: C
E. Intact function of the third palmar interosseous muscle
Discussion: Ligamentous laxity is the primary characteristic
Preferred Response: D
of the fingers in patients with SLE. It often results in
hyperlaxity and incompetence of the fingers and MP joint Discussion: The abducted small finger in high (proximal)
of the thumb. Generally speaking these patients are not ulnar nerve palsy (Wartenberg’s sign) occurs in
likely to ankylose and develop contractures. Swan-neck approximately 50% of patients with this injury. It is thought
deformity, rather than boutonniere deformities, are more to be caused by the preservation of function in the radially-
associated with SLE. innervated extensor digiti minimi muscle, which may have
an attachment to the abductor tubercle of the proximal
References:
phalanx of the small finger or an unbalanced ulnar slip
1. Bleifeld CJ, Inglis AE. The hand in systemic lupus
of the extensor digiti minimi tendon. The loss of the third
erythematosis. J Bone Joint Surg 1974;56:1207-15.
palmar interosseous and lumbrical to the small finger
2. Dray GJ. The hand in systemic lupus erythematosis. Hand Clin
1989;5:145-55. allows unopposed abduction and extension to occur by the
extensor digiti minimi tendon. The abductor digiti minimi
muscle is denervated in ulnar nerve palsy. The fourth dorsal
interosseous, which is also denervated, is an abductor of
138. The effect of physical training on the structure and
the ring finger.
function of muscle tissue is?
References:
A. Potential force of muscular contraction is directly
1. Gonzalez MH, et al. The extensor tendons to the little finger:
proportional to the increase in muscle cross sectional An anatomic study. J Hand Surg 1995;20A:844-847.
area produced by training
2. Chung MS, et al. Extensor indicis proprius transfer for the
B. The velocity of muscular contraction is inversely abducted small finger. J Hand Surg 2008;33A:392-397.
proportional to the change in muscle fiber length
produced by training
C. Physical training results in the production of additional 140. A 35 year-old carpenter presents with wrist pain,
muscle fibers which leads to increased force of cold intolerance, ring and small finger numbness,
muscular contraction and tingling of insidious onset. Figures 1-3
D. Physical training produces a change in muscle fiber demonstrate his arteriogram. His intraoperative photos
type within a single muscle (pre and post-reverse vein grafting) are shown.
Which factor is associated with improved long-term
E. The force of muscular contraction increases with
patency of the graft?
training as the velocity of contraction increases
A. Symptoms less than 2 years in duration
Preferred Response: A
B. Non-dominant extremity involvement
Discussion: The force of muscular contraction is directly
C. Male gender
proportional to the cross sectional area and inversely
proportional to the velocity of contraction. Longer muscular D. The use of a lower extremity vein graft
fibers are able to contract more rapidly than shorter ones. E. Life-long use of Persantine
Physical training results in hypertrophy of muscle fibers, but
Preferred Response: D
not production of additional ones, and does not lead to a
change in fiber type within a given muscle. Discussion: The patient shown has ulnar artery thrombosis
and underwent reverse saphenous vein grafting. Melhoff
Referrences:
and Wood demonstrated that factors associated with
1. Huard J, et al. Muscle injuries and repair: Current trends in
improved prognosis regarding long-term patency of the
research. J Bone Joint Surg 2002;84A:822-832.
reconstruction included: the use of a lower extremity graft,
2. Mariet E. Essentials of human anatomy and physiology. 9th ed.
Addison Wesley Publishing, 2008: 312-320. duration of symptoms for less than five months, obstruction

60 | American Society for Surgery of the Hand


secondary to a single traumatic event, and cessation of 141. A 24 year-old man was involved in a motorcycle
smoking. Most of these conditions involve males and racing accident. He hit a brick embankment at
affect the dominant extremity. While the definitive surgical approximately 60 mph. His only musculoskeletal
treatment of ulnar artery thrombosis is somewhat debated, injury is a fracture of the trapezial body. The
successful arterial reconstruction has been shown to fracture is associated with subluxation of the thumb
improve function, symptoms, and patient related outcomes. metacarpal. There is no associated fracture of
the metacarpal. He is treated with open reduction
References:
and pin fixation of the fracture-dislocation and
1. Melhoff TL, Wood MB. Ulnar artery thrombosis and the role
anatomical alignment is restored. The pins
of interposition vein grafting: Patency with microsurgical
technique. J Hand Surg 1991;16:174-8. are removed in 6 weeks and joint alignment is
2. Chloros GD, et al. Post-traumatic ulnar artery thrombosis: maintained. Two years after the injury he is most
Outcome of arterial reconstruction using reverse interpositional likely to complain of:
vein grafting at 2 years minimum follow-up. J Hand Surg
2008;33:932-40. A. Loss of pinch strength
B. Pain
C. Difficulty writing
D. Decreased range of motion of his thumb
E. Inability to work
Preferred Response: B
Discussion: In the largest single series published about this
injury, the most common problem was residual pain. Many
patients also complained of loss of grip strength, however,
objective measures demonstrated no difference in grip or
pinch strength compared to the contralateral uninjured side.
Surprisingly, loss of pinch strength and functions requiring
pinch were rare subjective complaints. Thumb and wrist
range-of-motion were unaffected by the injury.
Question 140, Figure 1
References:
1. McGuigan FX, Culp RW. Surgical treatment of intra-articular
fractures of the trapezium. J Hand Surg 2002;27A:697-703.
2. Colman M, Mass DP, Draganich LF. Effects of deep anterior
oblique and dorsoradial ligaments on trapeziometacarpal joint
stability. J Hand Surg 2007;32A:310-317.

Question 140, Figure 2

Question 140, Figure 3

2010 Self-Assessment Examination | 61


142. A 50 year-old woman complains of swelling in her 143. For the past six months, a retired 84 year-old
dominant right long finger and pain in her small man has had pain in his dominant left hand and
finger. She relates no history of trauma. She has wrist. He awakens several times every night with
had occasional low-grade fever, a dry cough pain and numbness in his thumb, index and long
but no other constitutional symptoms. Physical fingers. He has no thumb pain. Night splints do not
examination reveals flexor tenosynovitis without help. Pressure over the carpal tunnel reproduces
erythema in the long finger. There is a raised, his painful symptoms. Electrophysiologic studies
erythematous nodule at the dorsal base of the demonstrate sensory latency of 8.5msec and
small finger. There are no other hand deformities. motor latency of 6.5msec. Fibrillations are found in
Sensibility is intact. She also has several small, the abductor pollicis brevis muscle. A hand x-ray
raised tender nodules on her nose and cheeks. demonstrates stage III trapeziometacarpal joint
A hand x-ray demonstrates a lytic, moth eaten osteoarthritis. Recommended treatment for long-
appearance of the proximal phalangeal base. No term improvement of function is:
fractures are seen. A biopsy of the nodule and
A. Continued conservative treatment
tenosynovium is performed revealing non-caseating
granulomas without acid-fast bacilli. There is no B. Carpal tunnel release
endarteritis. Tissue cultures are negative. In addition C. Trapezial resection arthroplasty
to chest x-ray and blood tests, the most appropriate D. Trapezial resection arthroplasty combined with carpal
treatment for this patient is: tunnel release
A. Place a PPD test to rule out miliary tuberculosis E. Opponensplasty
B. Triple antibiotic therapy for tuberculosis Preferred Response: B
C. Antibiotic therapy for tertiary syphilis
Discussion: Carpal tunnel release surgery for severe
D. Synovectomy, debridement of the bone lesion, median nerve compression in elderly patients has been
followed by antibiotic therapy for atypical demonstrated to be effective in relieving painful symptoms.
mycobacterial infection In a prospective clinical outcome study that examined
E. Oral corticosteroid therapy 75 elderly patients with severe carpal tunnel syndrome,
there was statistically improved function in activities of
Preferred Response: E
daily living, work performance, overall hand function, and
Discussion: This patient presents with features typical for satisfaction with hand function. Sensory recovery is rarely
sarcoidosis. It occurs in the United States approximately complete, and motor recovery is not expected. Painful
ten times more frequently in women than men, and about symptoms, especially nocturnal paresthesias, usually
five times more commonly in African American women improve and 90% of patients are satisfied with the outcome.
than white women. The etiology is unknown. Bilateral hilar
References:
lymphadenopathy is present in 90% and accounts for the
1. Weber RA, Rude MJ. Clinical outcomes of carpal tunnel release
cough. Patients with sarcoidosis and miliary tuberculosis
in patients 65 and older. J Hand Surg 2005;30A:75-80.
are anergic, so a PPD will be negative in both. Absence of
2. Leit ME, et al. Patient-reported outcome after carpal
acid fast bacilli suggests a mycobacterial infection is less tunnel release for advanced disease: a prospective and
likely. Most patients with tertiary syphilis have neurological longitudinal assessment in patients older than 70. J Hand Surg
abnormalities and biopsies demonstrate endarteritis. 2004;29A:379-383.
Operative excision of the sarcoid tissue frequently results
in a recurrence, and oral corticosteroids are the hallmark
of treatment.
References:
1. Wang HT, Sunil TM, Kleinert HE. Multiple unusual complications
after extensive chronic sarcoid tenosynovitis of the hand: A
case report. J Hand Surg 2005;30A:610-614. Although this is a
case report, it is a good review of the literature and has a good
reference list.
2. Gonzalez del Pino J, et al. Sarcoidosis of the hand and wrist.
J Hand Surg 1997;22A:942-945.

62 | American Society for Surgery of the Hand


144. The most common problem related to transfer of 145. When treating a poorly-differentiated squamous cell
the posterior third of the deltoid to the triceps, in the carcinoma of the dorsal hand in a 50 year-old male
restoration of elbow extension in patients with a C5 laborer, the highest cure rates are achieved with
functional level spinal cord injury is: which treatment modality:
A. Rupture or stretching of the repair A. Surgical excision incorporating a 4 mm tumor
B. Injury to the axillary nerve free margin
C. Excursion provided by the deltoid is inadequate to B. Surgical debulking (intralesional) followed by
extend the elbow cryosurgery with liquid nitrogen
D. Heterotopic ossification of the triceps C. Mohs micrographic surgery
E. Ankle contracture after harvesting the tibialis D. Electrodessication and curettage
anterior tendon E. Photodynamic therapy
Preferred Response: A Preferred Response: C
Discussion: The most common complication is stretching Discussion: Although all of the answers have been used
or rupture of the repair. Post-operative immobilization of the for squamous cell carcinoma of the skin on the dorsum of
elbow in 10 degrees of flexion can help avoid this potential the hand, Mohs micrographic surgery has the highest cure
problem. The 3 cm of deltoid excursion is adequate for rate. It has the highest cure-rate for all non-melanoma skin
elbow extension, and the potential complication of deltoid cancers. Surgical excision of aggressive lesions, defined
muscle denervation can be minimized by avoiding deep as poorly differentiated or greater than 2.5 cm, require
muscle dissection within 5 cm of the lateral edge of the a 6 mm tumor free margin. The dorsum of the hand is
acromion. Post-operative ankle contracture is best avoided considered a high-risk lesion. Radiation therapy is indicated
with casting for 4-6 weeks. Heterotopic ossification is a rare as an adjuvant to surgery in cases of recurrent tumor, large
complication. lesions (>2 cm), deep lesions (>4 mm), as well as those
with perineural invasion or regional nodal metastasis.
References:
Consultation with a dermatologist should be considered in
1. Hamou C, et al. Pinch and elbow extension restoration in
all patients with squamous cell carcinoma.
people with tetraplegia: a systematic review of the literature.
J Hand Surg 2009;34A:692-699. References:
2. Mulcahey MJ, et al. Prospective evaluation of biceps to triceps 1. Hepper DM, et al. Treatment options for squamous cell
and deltoid to triceps for elbow extension in tetraplegia. J Hand carcinoma of the dorsal hand including Mohs micrographic
Surg 2003;28A:964-971. surgery. J Hand Surg 2009;34A:1337-1339.
2. Scholtens REM, et al. Treatment of Recurrent Squamous Cell
Carcinoma of the Hand in Immunosuppressed Patients. J Hand
Surg 1995;20A:73-76.

2010 Self-Assessment Examination | 63


146. A 36 year-old computer programmer injured his
dominant right wrist in a motorcycle accident. His
physical exam shows marked swelling in the wrist
with pain diffusely to palpation. The x-ray is shown in
Figure 1. The best treatment for this injury would be:
A. Closed reduction of radius and scaphoid and thumb
spica splint
B. Closed reduction percutaneous pin fixation of radius
fracture and thumb spica splint
C. Closed reduction of radius with external fixation and
thumb spica splint
D. Closed reduction of radius with percutaneous fixation
of scaphoid and long arm splint
Figure 1
E. Open reduction and internal fixation of all injuries
Preferred Response: E
Discussion: Combined distal radius and scaphoid injuries
are uncommon, generally high-energy injuries which occur
in patients between the 3rd and 5th decade. Several limited
series have reviewed treatment for these injuries. More
recent studies advocate internal fixation of both fractures in
order to initiate early rehabilitation. The distal radial fracture
is the prime determinant of outcome, and the radius fracture
treatment is the most important treatment consideration.
While closed treatment may be done in nondisplaced radial
fractures, displaced radial fractures are recommended for
Figure 2 Figure 3
internal fixation, with open or arthroscopic reduction. Screw
fixation of the scaphoid is recommended, considering bone
grafting if the scaphoid injury is comminuted or widely
displaced. Although there is concern about distraction 147. Figure 1 shows a mass found in a 38 year-old female
forces, either from reduction maneuvers or external with symptoms of carpal tunnel syndrome and nerve
fixation causing displacement and healing problems in conduction studies showing advanced median
the scaphoid, the literature is divided over whether this is nerve compression. The mass is a:
actually a clinical problem. A. Schwannoma
The ulnar styloid fracture was fixed to stabilize a distal radial B. Neurofibroma
ulnar joint, which was unstable after reduction and fixation C. Intraneural lipofibroma
of the radius. D. Giant Cell tumor
References: E. Synovial sarcoma
1. Rutgers M, Mudgal CS, Shin R. Combined fractures of the
distal radius and scaphoid. J Hand Surg 2008;33A:478-83. Preferred Response: C
2. Slade JF, Sudeep T, Safanda J. Combined fracture of scaphoid Discussion: Figure 1 shows an example of an intraneural
and distal radius fracture; a revised treatment rationale lipoma, or lipofibroma, of the median nerve. This sometimes
using percutaneous and arthroscopic techniques. Hand Clin
2005;21:427-41.
occurs with macrodactyly. The median nerve is most
commonly involved, although multiple other peripheral
3. Trumble TE, Benirschke SK, Vedder NB. Ipsilateral fractures of
the scaphoid and radius. J Hand Surg 1993;18A:8-14. nerves (including the ulnar nerve) have been reported.
Treatment is aimed primarily at decompression of the nerve
involved, although sometimes, debulking or resection is
indicated for neurologic deterioration.
References:
1. Camilleri IG, Milner RH. Intraneural lipofibroma of the median
nerve. J Hand Surg 1998;23B:120-22.
2. Houpt P, Storm van Leeuwen JB, van den Bergen HA.
Intraneural lipofibroma of the median nerve. J Hand Surg
1989;14A:706-9.

64 | American Society for Surgery of the Hand


References:
1. Supan TJ. Active functional prostheses. Hand Clin 2003;19:185-191.
2. Lake C, Dodson R. Progressive upper limb prosthetics.
Phys Med Rehabil Clin N Am 2006;17:49-72.
3. Raichle KA, Hanley MA, Molton I, Kadel NJ, Campbell K, Phelps
MA, Ehde D, Smith DG. Prosthesis use in persons with lower and
upper limb amputations. J Rehabil Res Dev 2008;45(7):961-72.

149. A 21 year-old female presents with a history of blunt


trauma to the proximal forearm, two years prior
to exam. Since the injury, she has been unable to
extend any of her digits. She extends the wrist with
5/5 strength. Her neurological exam is otherwise
normal. The best choice for tendon transfers to
Question 147, Figure 1 restore function to her hand would be:
A. PT to EDC, PL to EPL
148. A 63 year-old diabetic male with renal failure on dialysis B. FCU to EDC, FDS IV to APL
presents with complaints of increasingly severe pain in C. FCR to EDC, PL to EPL
the hand. He has had multiple digital amputations for D. ECU to EDC, PL to EPL
nonhealing ulcers. The hand appears cyanotic to the
E. ECRB to EDC, FDS IV to APL
midpalm. The best level for amputation would be:
Preferred Response: C
A. Metacarpal
B. Distal third forearm Discussion: The patient has an injury to the posterior
interosseous nerve (PIN), and has intact radial wrist
C. Mid forearm
extensor function. The pronator teres is used to restore
D. Proximal third forearm wrist extension, in radial nerve palsy; but in this example,
E. Elbow disarticulation the patient does not need this restored.
Preferred Response: B The FCU is often used to motor the digital extensors in radial
nerve palsy, and is the strongest motor with better excursion
Discussion: Because of the relative rarity of upper extremity
than the FCR. In PIN palsy, the radial wrist extensors still
amputation, most hand surgeons do not gain a great deal
function so that using the ulnar-sided wrist flexor in transfer
of experience with this problem. Preserving limb length
weakens the ulnar side, and leads to radial deviation of the
is generally the best approach when selecting an upper
wrist, especially with wrist extension. The ECU muscle is
extremity amputation level.
denervated in PIN palsy, and is of no use for transfer.
In the forearm, preserving length will preserve forearm
Although the ECRB can be transferred to give finger extension,
rotation. Longer forearm levels better distribute forces
it is not optimal, as the strength and excursion are less than
across the limb during daily activities, and may help with
the EDC. The thumb extension can be restored by transfer of
residual limb pain, which limits prosthesis use. Studies have
the PL to the EPL. This involves less donor deficit than taking
shown that persons with distal upper extremity amputation
a superficialis tendon from the ring or long finger, which some
wore their prostheses more days per month than those
authors feel should be avoided in patients such as computer
with a proximal amputation; also, that appearance of the
workers (who require maximal autonomy of finger flexor function).
prosthesis is comparable in importance to its function for
those with upper limb loss. The FCR to EDC and PL to EPL will avoid radial deviation in
wrist extension. The FCR can be transferred either around the
While wrist disarticulation retains more forearm rotation,
radial side of the wrist, or through the interosseous membrane.
it creates problems due to extra limb length. This may
The PL will motor the rerouted EPL. This is the preferred
restrict the type of components which can be attached as a
transfer in PIN palsy. An option for patients with absent PL
terminal device. In most cases, use the “three finger widths
can be the FCR, split to motor both the fingers and thumb.
rule” to determine the level from the wrist joint crease for a
transradial amputation. References:
Elbow disarticulation gives better rotational control than 1. Ropars M, Deano T Piret P, Belot N, Langlais F. Long term
results of tendon transfers in radial and posterior interosseous
a more proximal transhumeral level, and it optimizes nerve paralysis. J Hand Surg 2006;31B(5):502-506.
suspension. However, elbow function, in terms of weight
2. Chidgey LK, Szabo RM. Radial nerve palsy. In: Chapman MW,
bearing and force distribution are not as good as an intact ed. Chapman’s Orthopaedic Surgery. 3rd ed. Philadelphia:
elbow joint, so forearm levels are preferred if possible. Lippincott Williams & Wilkins, 2001: 1633-45.

2010 Self-Assessment Examination | 65


150. A 52 year-old female complains of scissoring of the 151. The 33 year-old contractor pictured in Figure 1 had
fingers following a proximal phalangeal fracture an amputation of the thumb tip. The most common
5 months prior to presentation. Her deformity is complication as a result of the treatment shown in
shown in the clinical photo. The maximum expected the photo is:
correction a rotational osteotomy at the metacarpal
A. Loss of length
level can achieve is approximately how many degrees?
B. Flap failure
A. 10
C. Cold intolerance
B. 20
D. Unstable skin coverage
C. 30
E. Joint stiffness
D. 40
E. 50 Preferred Response: E
Preferred Response: B Discussion: The photo shows a cross-finger flap from the
index proximal phalanx dorsum to the thumb tip. This is a
Discussion: The treatment of phalangeal malunions is
reliable flap with low-risk of flap failure (30 of 31 successful
difficult and may be compromised by stiffness, particularly
in one report). The flap can resurface moderate size defects
around the PIP joint. Many surgeons prefer to treat purely
with good quality soft tissue coverage, and limit the loss
rotational malunion with osteotomy at the metacarpal level
of length. Although cold intolerance occurs, the more
to avoid tendon adhesion and PIP stiffness. If the nonunion
common complication is related to loss of motion in the MP
is less than 10 weeks old, osteotomy through the fracture
and IP joints.
site will reestablish the anatomy, and may be the best choice.
Other coverage options to maintain sensibility also have
For a longer duration malunion of the metacarpal or phalanx,
limitations. Palmar advancement is effective with pulp
metacarpal derotation osteotomy (either transverse or step cut)
defects up to 1.5cm, but may limit IP motion. Radial and
has been show to improve alignment with little risk of motion
neurovascular island flaps have problems with donor site
loss or joint stiffness, and it has demonstrated reliable healing.
neuromas, and cortical misrepresentation. The authors
In cadaver studies, Gross and Gelberman have shown the below showed good recovery of protective sensation with
deep metacarpal ligament limits the maximal rotation of simple cross finger flap, and concluded that in comparison
the digit to 19° in the index, long and ring, and 30° in the to other, more complex innervated flaps, that it remains
little finger. uncertain if a greater surgical effort consistently delivers a
superior surgical outcome.
References:
1. Jawa A, Zucchini M, Lauri G, Jupiter J. Modified step-cut References:
osteotomy for metacarpal and phalangeal rotational deformity. 1. Woon CY, Lee JY, Teoh LC. Resurfacing hemipulp losses of
J Hand Surg 2009;34A:335-40. the thumb: the cross finger flap revisited: indications, technical
2. Freeland AE, Lindley SG. Malunions of the finger metacarpals refinements, outcomes and long term neurosensory recovery.
and phalanges. Hand Clin 2006;22:341-55. Ann Plast Surg 2008;61(4):385-91.
3. Ring D. Malunion and nonunion of the metacarpals and 2. Hynes DE. Neurovascular pedicle and advancement flaps for
phalanges. J Bone Joint Surg 2005;87A:1380-88. palmar thumb defects. Hand Clin 1997;13(2):207-16.
4. Gross MS, Gelberman RH. Metacarpal rotational osteotomy.
J Hand Surg 1985;10A:105-108.

Figure 1

Figure 1

66 | American Society for Surgery of the Hand


152. The 11 year-old female in Figure 1 has a contracture 153. The injury shown in Figures 1 and 2, occurred while
of her small finger, which arose from a laceration at bench pressing. There is no pain at the elbow.
age 3, after getting it caught in an exercise machine. Appropriate management of this closed fracture
The limitation of passive extension is 25° at the PIP should include ORIF of the radius. After ORIF of
and 15° at the DIP. The best reconstructive choice to the radius what other important aspect should be
correct this contracture would be: assessed intraoperatively?
A. Z-plasty A. Wrist flexion and extension
B. Split thickness skin graft B. Elbow varus and valgus stability
C. Full thickness skin graft C. Elbow flexion and extension
D. Kite flap D. Stability of DRUJ
E. Cross finger flap E. ROM of DRUJ

Preferred Response: A Preferred Response: D


Discussion: This mild contracture is best served by the simplest Discussion: The x-ray shows a comminuted fracture of the
level of the reconstructive ladder. Z-plasties are transposition radial shaft and a fracture at the base of the ulnar styloid.
flaps which require no distant donor site, no prolonged Accurate restoration of radial length and stable fixation
immobilization, and they heal reliably. They will provide up is the essential first step in treatment. Once the radius is
to a 75 percent increase in length with 60° angles, which will fixed, intraoperative assessment of the stability of the DRUJ
adequately restore full extension in this case (Figure 2). is essential. If unstable, consideration should be given to
Skin grafting after scar release, with either full or partial fixation of the ulnar styloid fracture. This can be done with
thickness, would be a better choice for more advanced skin percutaneous K-wires, cannulated screws, or tension band
contracture. A neurovascular rotation flap, such as a kite flap, technique. If no fracture is present, then repair of the TFCC
is primarily indicated if there is tissue loss in the thumb—where should be considered if the DRUJ is unstable. ROM of
sensibility is desired. A cross-finger flap would be used only for the wrist, elbow, and DRUJ should also be assessed, but
contractures so extensive that correction would expose deeper are unlikely to be abnormal in the absence of elbow pain.
structures, such as tendons and nerves that need coverage. Elbow stability should also be checked, but is unlikely to be
a problem.
References:
1. Lehman TP, Petersen SL, Rayan GM. Soft tissue coverage of References:
the hand. In: Trumble TE, Budoff JE, eds. Hand Surgery Update 1. Rettig ME, Raskin KB. Galeazzi fracture-dislocation:
IV. Rosemont, IL: American Society for Surgery of the Hand, a new treatment-oriented classification. J Hand Surg
2007:593-617. 2001;26A:228-35.
2. Trumble TE, Vedder NB. Tissue Transfer: pedicle and free 2. Giannoulis FS, Sotereanos DG. Galeazzi fractures and
tissue flaps. In: Trumble TE, ed. Principles of Hand Surgery and dislocations. Hand Clin 2007;23(2):153-63.
Therapy. 1st ed. Philadelphia: WB Saunders, 2001:499-528.

Figure 1

Figure 1 Figure 2

Figure 2
2010 Self-Assessment Examination | 67
154. Surgical anterior approach to expose the fracture Careful exam is helpful in determining the contributing
in Figures 1 and 2 in question 153 should be between factors causing contracture. Equal active and passive
what two structures? ROM of a joint suggests that capsular tightness is the main
cause. Relatively good PIP motion and MP motion that is
A. FCR and Median nerve
unchanged by wrist position suggests that extrinsic flexor
B. Radial artery and Brachioradialis or extensor tendons are not a significant factor in this MP
C. Brachioradialis and ECRL motion loss although the limited wrist motion must be
D. ECRL and APL taken into account. Oblique retinacular ligament tightness
would tend to cause DIP extension and a flexed PIP joint.
E. FPL and FCR
Tightness of this ligament is suspected when the DIP joint
Preferred Response: B can be flexed only with PIP joint flexion. With the limited MP
Discussion: Anterior or volar approach to the distal half of motion, the classic Bunnell intrinsic muscle tightness test
the radius is through a volar incision over the FCR tendon. cannot be performed; thus, intrinsic tightness cannot be
Deeper dissection is between the Brachioradialis and the entirely ruled out. However, the good PIP motion suggests
Radial artery. Dissection between the FCR and Median that intrinsic tightness is not a major cause.
nerve in the distal forearm places the median nerve and References:
palmar cutaneous branch at risk. Dissection between the 1. Shin AY, Amadio PC. Stiff finger joints. In: Green DP, Hotchkiss
Brachioradialis and ECRL places the radial sensory nerve RN, Pederson WC, Wolfe SW, eds. Green’s Operative Hand
at risk and is not an internervous plane. This is true for Surgery. 5th ed. Philadelphia: Churchill Livingstone/Elsevier,
the ECRL and APL interval as well. The FPL is deep to the 2005:424.
FCR and needs to be reflected ulnarly to expose the radial 2. Field PL, Hueston JT. Articular cartilage loss in long-standing
shaft. The insertion of the Pronator Teres will also need to immobilization of the interphalangeal joints. Br J Plast Surg
1970;23:186-191.
be reflected ulnarly. Exposure more proximal in the forearm
requires dissection between the brachioradialis and FCR,
as well as release of the supinator muscle.
156. Cold intolerance following a finger amputation will:
References:
1. Hoppenfeld S, deBoer P. Surgical Exposures in Orthopaedics. A. Resolve with time
2nd ed. Philadelphia: Lippincott Williams & Wilkins, 1994: B. Occur in one third of the patients
120-123.
C. Be less likely with flap coverage
2. Agur AMR, Lee MJ. Grant’s Atlas of Anatomy. 10th ed.
Philadelphia: Lippincott Williams & Wilkins, 1999: 476-479. D. Be less likely if the digit is replanted
E. Be unaffected by surgical treatment
Preferred Response: E
155. A 45 year-old presents 4 months after cast
treatment for a distal radius fracture complaining Discussion: Cold intolerance is a common sequela to
of an inability to make a fist. Clinical exam reveals any digital amputation. Multiple studies have reported
40° of wrist flexion and extension. MP joint motion incidences of 80 percent and higher. This problem is likely
reveals full extension but only a 20° arc of both more common in colder climates. Although the symptoms
active and passive flexion. Wrist position has no may moderate with time, several studies looking at cold
effect on MP motion. The PIP joints lack only a few intolerance from amputation or nerve injury do not show
degrees of extension with 90° of active flexion. The such resolution.
most likely cause of the MP joint contracture is?
There seems to be little difference in the incidence or
A. Extrinsic extensor tendon tightness severity of cold intolerance, whether a digital amputation is
B. Extrinsic flexor tendon tightness simply revised, or the digit is replanted. The cold intolerance
is felt to be the result of the initial trauma, rather than the
C. Intrinsic muscle tightness
subsequent surgical treatment. The pathophysiology of cold
D. MP capsular tightness sensitivity is not well understood, and is usually ascribed
E. Oblique retinacular ligament tightness to the sympathetic nervous system. It does not seem to
correlate with vessel patency or flow at the capillary level.
Preferred Response: D
References:
Discussion: Contracture of joints can develop after many
1. Lithel M, Backman C, Nystrom A. Cold intolerance is not
different types of trauma and determining its cause can be more common or disabling after digital replantation than after
difficult. In this case, no direct trauma to the hand is noted; other treatment of compound digital injuries. Ann Plast Surg
but immobilization, post-traumatic swelling, or reluctant 1998;40(3):256-9.
ROM of the fingers has led to an MP joint contracture.

68 | American Society for Surgery of the Hand


2. Nystrom A, Backman C, Bertheim U, Karlson L, Carlsson A.
Digital amputation, replantation and cold intolerance.
J Reconstr Microsurg 1991;7(3):175-8.
3. Klein-Welgel P, Pavelka M, Dabernig J, Rein P, Kronenberg
F, Fraedich G, Plza Katzer H. Macro and microcirculatory
assessment of cold sensitivity after traumatic finger amputation
and microsurgical replantation. Arch Orthop Trauma Surg
2007;127(5):355-60.

157. A 65 year-old female complains of achy pain in her


right thumb with most activities. She has sharp
pains daily when trying to pinch objects. She denies
numbness and tingling. Clinical exam reveals pain
with thumb grind test at the CMC joint and marked
laxity of the ulnar collateral ligament of the MP joint Figure 1
with pain and hyperextension. There is no pain
with IP joint flexion. An x-ray is shown in Figure 1.
After discussing the treatment options the patient 158. The spastic clenched fist deformity is commonly
requests trapezial excision and tendon interposition seen in patients after a stroke involving the upper
arthroplasty. An adjunctive procedure that is extremity. This pattern is caused by?
recommended to be performed at the same time is?
A. Unmasking of the primitive grasp reflex
A. Open CTR
B. Ischemic contracture of the forearm flexor
B. Thumb IP joint arthrodesis compartment
C. Thumb MP joint arthrodesis C. Ischemic contracture of the hand intrinsic muscles
D. EPB tenotomy D. Lack of extension splinting during the acute stroke
E. Opponensplasty of thumb E. Failure to maintain passive ROM during recovery
Preferred Response: C Preferred Response: A
Discussion: Basilar joint arthritis of the thumb is a common Discussion: Distinct clinical syndromes occur following a
problem and the underlying pathology can be addressed stroke depending on which area of the brain is involved.
with a tendon interposition arthroplasty, CMC arthrodesis, Motor impairment and recovery follows a typical pattern.
or other type of arthroplasty. The presence of chronic Initial flaccid paralysis occurs, followed by gradual motor
MP joint ulnar collateral ligament laxity should also be recovery from proximal to distal, starting a few days after
addressed at the same time. While repair or reconstruction the initial insult. This increasing muscle tone can lead to
of the UCL can be considered, in this age group, a fusion eventual spasticity. The main cause of residual clenched
is more likely to provide stability and predictable pain hand posture is due to the unmasking of the primitive grasp
relief. While about 20% of patients with osteoarthritis of reflex. Peripheral ischemia or compartment syndromes are
the thumb CMC joint will have concurrent CTS, this patient not common after a stroke, unless the affected individual
did not complain of numbness. Any flexed posture of the is alone and left in one position for a prolonged period of
thumb IP joint usually corrects if the MP and CMC joints time. While extension splinting is helpful during the flaccid
are corrected. Fusion of the IP joint is not usually needed paralysis phase, the increasing spasticity that can develop
unless painful. EPB tenotomy or transfer to the metacarpal after severe strokes can make continued splinting difficult,
has been proposed to help address MP hyperextension in painful, or cause skin problems. Trying to maintain passive
the absence of arthritis or pain. This will not help correct motion as the spasticity increases can become a full time
ulnar MP laxity. Opponensplasty would only be needed in pursuit and is difficult to continue.
the presence of neuropathy or other nerve compression
syndrome. References:
1. Keenan MAE. Upper extremity dysfunction after stroke or brain
References: injury. In: Green DP, Hotchkiss RN, Pederson WC, Wolfe SW,
1. Trumble TE, ed. Hand Surgery Update 3, Hand, Elbow, and eds. Green’s Operative Hand Surgery. 5th ed. Philadelphia:
Shoulder. Rosemont, IL: American Society for Surgery of the Churchill Livingstone/Elsevier, 2005:1262.
Hand, 2003: 510. 2. Pomerance JF, Keenan MA. Correction of severe spastic
2. Tomaino MM, Pellegrini VD, Burton RI. Arthroplasty of the flexion contractures in the nonfunctional hand. J Hand Surg
basal joint of the thumb, long-term follow-up after ligament 1996;21A:828-833.
reconstruction with tendon interposition. J Bone Joint Surg
1995;77A:346-355.

2010 Self-Assessment Examination | 69


159. A 35 year-old mechanic presents complaining of 160. The best adjunctive therapy agent to decrease
pain, limited motion, and swelling of his middle xanthine production is:
finger after a puncture wound 3 days ago. Exam
A. Heparin
reveals a flexed resting posture of the finger, pain
with passive stretching, and marked tenderness B. Dextran
along the flexor tendon sheath. Appropriate C. Allopurinol
management should be? D. Aspirin
A. Outpatient management with oral antibiotics E. T-PA
B. Needle aspiration of flexor tendon sheath and Preferred Response: C
oral antibiotics
Discussion: Establishment of blood flow after microvascular
C. Inpatient management with intravenous antibiotics
anastomosis may lead to reperfusion injury. One
D. Operative drainage contributing factor may be due to ischemia-induced
E. Continuous irrigation with tendon sheath catheter hypoxanthine conversion to xanthine. Blockage of this
reaction has been shown to decrease post-operative pain,
Preferred Response: D
swelling, and infection rate.
Discussion: Infection of the finger flexor tendon sheath
A well-performed randomized study by Waikakul
is usually a result of a direct puncture wound. However,
compared outcomes in replantations with and without the
extension from a felon or other deep space infection is
perioperative addition of allopurinol. The allopurinol group
possible. Diagnosis of pyogenic flexor tenosynovitis is made
demonstrated a lower infection rate, less perioperative
clinically. The typical hallmarks of infection include pain
pain, and less swelling. Nerve recovery may also have
with passive extension, tender volar flexor tendon sheath,
improved. Reperfusion after ischemia results in the
fusiform finger swelling, and a flexed finger posture different
activation of neutrophils which release cytotoxic oxidants.
from adjacent fingers. A high index of suspicion maybe
These oxidants initiate a cascade of capillary permeability
needed in unusual infections or immunocompromised
and tissue damage with the reestablishment of blood flow.
hosts, where the typical signs may not be present. The
This study suggests that the xanthine oxidase inhibitory
preferred treatment for infections that are treated within 3-5
effects of allopurinol may decrease some of the damage
days of onset is small proximal and distal incisions into the
brought about by prolonged ischemia. The other adjunctive
tendon sheath with careful intra-operative irrigation. Care
pharmacologic agents have a role in replantation. Dextran,
should be taken to avoid tendon sheath distention possibly
with its volume enhancing effects is used routinely in many
causing finger ischemia. Lille has shown that placement
replant centers. Aspirin is also used for its platelet inhibition.
of a tendon sheath catheter for continuous irrigation is
Heparin and t-PA use is less common.
unnecessary. Larger exposure and debridement of fibrotic
tenosynovium associated with chronic or unusual infections References:
maybe needed 1. Trumble TE, ed. Hand Surgery Update 3, Hand, Elbow, and
Shoulder. Rosemont, IL: American Society for Surgery of the
References: Hand, 2003: 475.
1. Trumble TE, ed. Hand Surgery Update 3, Hand, Elbow, and 2. Waikakul S, et al. The role of allopurinol in replantation. J Hand
Shoulder. Rosemont, IL: American Society for Surgery of the Surg 1999;24B(3):325-7.
Hand, 2003: 443.
3. Conrad MH, Adams WP. Pharmacologic optimization of
2. Lille S, Hayakawa T, Neumeister MW, Brown RE, Zook EG, microsurgery in the new millennium. Plast Reconstr Surg
Murray K. Continuous postoperative catheter irrigation is not 2001;108(7):2088-96.
necessary for the treatment of suppurative flexor tenosynovitis.
J Hand Surg 2000;25B:304-307.

161. In distal biceps repair, the advantage of 2-incision


versus 1-incision technique yields is:
A. Improved supination strength
B. Improved elbow motion
C. No clear advantage
D. Lower incidence of nerve injury
E. Lower risk of heterotopic ossification
Preferred Response: C

70 | American Society for Surgery of the Hand


Discussion: The repair of distal biceps tendon ruptures has CMC joint have been associated with unacceptably high
undergone intensive recent study. Without repair, about complication rates. Arthrodesis does provide good pain
70% return of flexion strength and 50% return of supination relief and stability, but the loss of motion and inability to
torque can be expected by one year after injury. This may flatten out the hand can be problematic.
not be acceptable to young active workers that require
It is of equal importance to assess the MP joint, as this
manual labor. The classic two-incision repair through bone
is crucial to long term success of the LRTI or other CMC
tunnels, described by Boyd and Anderson, was utilized
procedures. MP hyperextension of greater than 20° or 30°
for years. This provided anatomic repair of the tendon, but
can result in continued collapse of the thumb with pinch
carried a significant risk of heterotopic ossification leading
activities. Restoring MP stability with a volar capsulodesis
to radio-ulnar synostosis. Recently, single-incision anterior
or arthrodesis is needed to provide the best outcomes. If
repairs have been performed using several repair methods.
no arthritis is present, then capsulodesis or arthrodesis
Endo-Button repair seems to provide a superior repair that
can be performed. Patients with arthritic changes, pain,
allows early mobilization. Henry’s study has shown that,
or limited flexion arcs are treated with arthrodesis. Implant
while the one anterior incision does not place the tendon
arthroplasties of the thumb MP joint require intact collateral
repair in anatomic position, there is no clinically-significant
ligaments and are unpredictable in outcome.
difference in supination torque and flexion force compared
to a two-incision technique. References:
1. Trumble TE, ed. Hand Surgery Update 3, Hand, Elbow, and
References:
Shoulder. Rosemont, IL: American Society for Surgery of the
1. Morrison KD, Hunt TR. Comparing and contrasting methods Hand, 2003: 510.
for tenodesis of the ruptured distal biceps tendon. Hand Clin
2. Trumble TE, Budoff JE, eds. Hand Surgery Update IV.
2002;18(1):169-78.
Rosemont, IL: American Society for Surgery of the Hand, 2007:
2. Greenberg JA. Endobutton repair of distal biceps tendon 639-641.
ruptures. J Hand Surg 2009;34A:1541-8.
3. Tomanio MM, Pellegrini VD, Burton RI. Arthroplasty of the
3. Henry J, et al. Biomechanical analysis of distal biceps tendon basal joint of the thumb, Long term follow-up after ligament
repair methods. Am J Sports Med 2007;35(11):1950-4. reconstruction with tendon interposition. J Bone Joint Surg
4. Greenberg JA, et al. Endobutton assisted repair of distal biceps 1995;77A:346-355.
tendon ruptures. J Shoulder Elbow Surg 2003;12(5):484-90.

163. Which of the following treatment modalities is


162. A 57 year-old female complains of worsening pain in contraindicated in the treatment of snakebites?
her non-dominant thumb. Pain is mostly with activity
A. Antibiotics
limiting her ability to pinch and grasp objects. She
has some achiness at night, but denies morning B. Tetanus prophylaxis
numbness, tingling, or nocturnal awakening. Clinical C. Systemic corticosteroids
exam reveals a positive thumb grind test and 60° of D. Application of ice
MP hyperextension with 60° of MP flexion. X-rays
E. Application of a “lymphatic” tourniquet
reveal pantrapezial arthritis. MP joint space is well
preserved. The preferred treatment is? Preferred Response: D
A. CMC and MP arthrodesis Discussion: Treatment of poisonous snakebites continues
B. CMC arthrodesis and MP volar capsulodesis to evolve. While some treatment aspects remain
controversial, others are more universally accepted. The
C. CMC implant arthroplasty and MP arthrodesis
use of a broad spectrum antibiotic and tetanus prophylaxis
D. CMC LRTI and MP volar capsulodesis is appropriate. The application of a lymphatic tourniquet
E. CMC and MP implant arthroplasties is helpful, but this should be applied loosely and moved
proximally as swelling advances. Steroid administration
Preferred Response: D
is advocated by some authors, but has no benefit in
Discussion: This clinical picture is one of CMC basilar preventing tissue damage. It is universally accepted that
joint arthritis and MP hyperextension without arthritis. ice should not be used after snakebites. Cooling has been
This problem requires treatment of both joints for long shown to potentiate the necrotic effect of the snake venom.
term success. The CMC arthritis can be treated with
References:
resection of the trapezium alone, tendon interposition,
ligament reconstruction and tendon interposition, implant 1. Deichmann WB, Radomski JL, Farrell JJ, et al. Acute toxicity
and treatment of intoxication due to Crotalus adamanteus
arthroplasty, or arthrodesis. Each procedure has their (rattlesnake venom). Am J Med Sci 1958;236:204-207.
advocates, but of the choices offered above, the LRTI is
2. Kerrigan KR. Bacteriology of snakebite abscess. Trop Doctor
the most appropriate. LRTI has been in widespread clinical 1992;22:158-160.
use since 1986 and has consistently provided pain relief 3. Ya PM, Guzman T, Perry JF. Treatment of bites of North
and preservation of motion. Implant arthroplasties of the American pit vipers. South Med J 1961;54:134-136.

2010 Self-Assessment Examination | 71


164. A 55 year-old male sustains an essentially non- 165. A 34 year-old man presents 2 months after
displaced fracture of his distal radius. He was penetrating injury to the hand. Examination suggests
placed in a cast and followed every two weeks. radial digital nerve injury to his index finger. At
X-rays remained unchanged as to fracture position. surgery, after trimming of the ends, the resulting gap
Just before he is to return for his 6-week check, he is 10 mm. The best reconstructive choice is:
calls complaining of a sudden inability to extend
A. Direct repair under tension after nerve mobilization
his thumb. He reports no new injury or pain.
Recommended treatment is: B. Direct repair by keeping the finger in full flexion
C. Nerve grafting with sural nerve donor
A. Application of a thumb spica cast
D. Nerve grafting with lateral antebrachial cutaneous
B. Primary tendon repair
nerve donor
C. Thumb IP joint fusion
E. Spanning the gap with a conduit
D. ORIF of fracture
Preferred Response: E
E. EIP to EPL tendon transfer
Discussion: Nerves should be repaired without tension.
Preferred Response: E
When a significant gap is present, nerve grafting or conduit
Discussion: Impacted and minimally-displaced fractures reconstruction is required. Polyglycolic acid conduit
of the distal radius are routinely treated in a cast or splint. reconstruction produces results that are equivalent to
The EPL has a relative avascular area at the level of those of grafting in large gaps (greater than 8 mm) while
Lister’s tubercle. Here, the tendon can undergo pressure eliminating donor-site morbidity associated with harvest.
necrosis, due to increased pressure in the third extensor
References:
compartment from post-fracture swelling or hematoma.
1. Weber RA, Breidenbach WC, Brown RE, Jabaley ME, Mass DP.
This can cause delayed rupture of the tendon, leading to
A randomized prospective study of polyglycolic acid conduits
loss of thumb function. Operative exploration is indicated for digital nerve reconstruction in humans. Plast Reconstr Surg
to explore the EPL tendon. Treatment options include 2000;106:2036.
primary repair, repair with a tendon graft, or tendon transfer. 2. Mackinnon SE, Dellon AL. Clinical nerve reconstruction with
Repair is often difficult (or impossible even) with tendon a bioabsorbable polyglycolic acid tube. Plast Reconstr Surg
transposition, due to the atrophic nature of the tendon 1990;85:419.
rupture. Repair with an intercalated tendon graft is possible,
but requires a donor tendon, and healing of two tendon
repair sites. EIP to EPL transfer is the preferred treatment. 166. A 32 year-old man presents with a small puncture
While this procedure is not urgent, it should be performed wound in the pulp of his left index finger which
to prevent loss of thumb extension. occurred when the paint gun he was cleaning
with paint thinner misfired. The most appropriate
Application of a thumb spica cast will not help, as the
treatment is:
tendon ends are not in close proximity. Sudden onset of
radial nerve palsy is unlikely and would usually affect finger A. Administration of systemic steroids
extension as well. ORIF is contraindicated at this time, B. Administration of antibiotics
unless fracture displacement has occurred.
C. Injection of a neutralizing agent
References: D. Local surgical debridement electively
1. Trumble TE, ed. Hand Surgery Update 3, Hand, Elbow and
Shoulder. Rosemont, IL: American Society for Surgery of the E. Wide surgical debridement emergently
Hand, 2003: 544. Preferred Response: E
2. Bonatz E, Kramer TD, Masear VR. Rupture of the EPL tendon.
Am J Orthop 1996;25:118-122. Discussion: High-pressure injection injuries can result in
3. Skoff HD. Post fracture extensor pollicis longus tenosynovitis devastating sequelae, with an amputation rate of 30%. The
and tendon rupture: a scientific study and personal series. risk of amputation is highest with organic solvent (paint,
Am J Orthop 2003;32:245-247. paint thinner, gasoline, oil, jet fuel) injection into the fingers.
Emergent surgical debridement within six hours reduces the
risk of amputation.
References:
1. Hogan CJ, Ruland RT. High-pressure injection injuries to the
upper extremity: a review of the literature. J Orthop Trauma
2006;20:503.
2. Gonzalez R, Kasdan ML. High pressure injection injuries of the
hand. Clin Occup Environ Med 2006;5:407.

72 | American Society for Surgery of the Hand


167. Patients with which of the following causes of ischemia
will benefit most from periarterial sympathectomy?
A. Smoking
B. Embolic
C. Auto-immune
D. Post-traumatic Figure 3
E. Atherosclerosis
Preferred Response: C
Discussion: Periarterial sympathectomy can result in complete
healing in patients with autoimmune digital ischemia from
vasospasm (e.g. Raynaud’s disease, CREST syndrome).
However, this procedure has little benefit in patients with
Figure 1 Figure 4
chronic ischemia due to atherosclerotic disease.
References:
1. Hartzell TL, Makhni EC, Sampson C. Long-term results of
periarterial sympathectomy. J Hand Surg 2009;34A:1454.
2. el-Gammal TA, Blair WF. Digital periarterial sympathetectomy for
ischemic digital pain and ulcers. J Hand Surg 1991;16B:382.
Figure 2 Figure 5

168. A 34 year-old female presents with a wrist mass for


several years. Radiographs, CT images, and biopsy
169. A 6-month-old male presents with upper limb
pathology slides are shown. The best treatment
deformities. Examination reveals radial deviation of
option is:
the hands with normal thumbs. Which laboratory
A. Intralesional curettage with bone autograft abnormality is expected?
B. Marginal resection with bone autograft A. Microcytic anemia
C. Wide resection and reconstruction with B. Iron deficient anemia
matched allograft
C. Hemolytic anemia
D. Wide resection with free vascularized graft and
D. Thrombocytopenia
wrist fusion
E. T-cell lymphopenia
E. Wide resection and prosthetic wrist reconstruction
Preferred Response: D
Preferred Response: D
Discussion: Radial longitudinal defect can be associated
Discussion: Giant cell tumors of bone are generally
with a variety of disorders. However, the presence of normal
treated with intralesional curettage, if there is no significant
thumbs suggests TAR (thrombocytopenia with absent
cortical break thru. With significant bone destruction, joint
radii) syndrome. TAR syndrome (autosomal recessive) is
reconstruction may be possible—using fibular head or
characterized by hypomegakaryocytic thrombocytopenia
matched allograft if the tumor has not invaded the radial
and bilateral radial aplasia.
carpal joint. In this case, with severe bone and joint space
destruction, the best option for a stable wrist is wide References:
resection including the proximal carpus and fusion with a 1. Bonsi L, Marchionni C, Alviano F, Lanzoni G, Franchina M,
vascularized fibular graft. Costa R, Grossi A, Bagnara GP. Thrombocytopenia with
absent radii (TAR) syndrome: from hemopoietic progenitor to
References: mesenchymal stromal cell disease. Exp Hematol 2009;37:1.
1. Kawamura K, Yajima H, Kobata Y, Murata K, Shigematsu K, 2. Ballmaier M, Schulze H, Strauss G, Cherkaoui K, Wittner N,
Maegawa N, Takakura Y. Wrist arthrodesis with vascularized Lynen S, Wolters S, Bogenberger J, Welte K. Thrombopoietin
fibular grafting. J Reconstr Microsurg 2009;25:501. in patients with congenital thrombocytopenia and absent
2. Ono H, Yajima H, Mizumoto S, Miyauchi Y, Mii Y, Tamai S. radii: elevated serum levels, normal receptor expression, but
Vascularized fibular graft for reconstruction of the wrist after defective reactivity to thrombopoietin. Blood. 1997;90:612.
excision of giant cell tumor. Plast Reconstr Surg 1997;99:1086.

2010 Self-Assessment Examination | 73


170. A 49 year-old female breast cancer survivor 171. A 62 year-old patient presents with the ring and small
presents for evaluation one year after axillary lymph finger deformity seen in Figure 1. He also has numbness
node dissection with painful numbness in her hand. in the thumb, index, long, and half the ring finger. Carpal
Physical examination reveals no lymphedema. tunnel compression test reproduces the symptoms.
The clinical and electrodiagnostic findings indicate Which is the most appropriate course of treatment?
median nerve compression at the ipsilateral
A. Observation
wrist. She has failed trials of splinting and steroid
injection. Appropriate treatment should include: B. Carpal tunnel release
C. Partial palmar fasciectomy
A. Continued splinting with advice against surgery
D. Staged carpal tunnel release and palmar
B. Repeat injection and splinting with advice against surgery
fasciectomies
C. Carpal tunnel release, exsanguination, and
E. Carpal tunnel release and palmar fasciectomies
tourniquet use
in the same operation
D. Carpal tunnel release, tourniquet use, but no
exsanguination is allowed Preferred Response: E
E. Carpal tunnel release with prohibition on Discussion: While there are reports that carpal tunnel
exsanguination and tourniquet use release performed concomitantly with partial palmar
Preferred Response: C fasciectomy in Dupuytren’s patients can result in a flare
reaction and complex regional pain syndrome, comparative
Discussion: Patients with prior ipsilateral axillary studies have shown that simultaneous surgery is
dissection can undergo elective upper extremity surgery recommended and does not increase the complication rate.
using the normal precautions of sterile technique and
anesthesia. If there is no pre-existing lymphedema, these References:
individuals should not be prohibited from blood pressure 1. Gonzalez F, Watson HK. Simultaneous carpal tunnel release
measurements or elective surgery. and Dupuytren’s fasciectomy. J Hand Surg 1991;16B:175.
2. Nissenbaum M, Kleinert HE. Treatment considerations in carpal
References: tunnel syndrome with coexistent Dupuytren’s disease. J Hand
1. Hershko DD, Stahl S. Safety of elective hand surgery following Surg 1980;5A:544.
axillary lymph node dissection for breast cancer. Breast J
2007;13:287-290.
2. Dawson WJ, Elenz DR, Winchester DP, Feldman JL. Elective
hand surgery in the breast cancer patient with prior ipsilateral
axillary dissection. Ann Surg Oncol 1995;2:132-137.

Figure 1

74 | American Society for Surgery of the Hand


172. A 4-month-old female is referred by her pediatrician References:
for weakness in the left upper extremity that has been 1. Kjaer-Petersen K, Jurik AG, Petersen LK. Intra-articular fractures
present since birth. If present, which of the following at the base of the fifth metacarpal. A clinical and radiographical
findings would suggest a favorable outcome? study of 64 cases. J Hand Surg 1992;17B:144-147.
2. Yoshida R, Shah MA, Patterson RM, Buford WL, Knighten
A. Ptosis and anhydrosis J, Viegas SF. Anatomy and pathomechanics of ring and
B. Biceps contraction small finger carpometacarpal joint injuries. J Hand Surg
2003;28A:1035-1043.
C. Scapular winging
3. Bushnell BD, Draeger RW, Crosby CG, Bynum DK. Management
D. Meningoceles on MRI of intra-articular metacarpal base fractures of the second
through fifth metacarpals. J Hand Surg 2008;33:573-83.
E. Insertional activity on electromyography
Preferred Response: B
Discussion: Return of biceps function before six months is
an important prognosticating indicator for obstetric brachial
plexus palsies. The other findings listed indicate a pre-
ganglionic lesion, for which the prognosis is worse.
References:
1. Kirjavainen M, Remes V, Peltonen J, et al. The function of
the hand after operations for obstetric injuries to the brachial
plexus. J Bone Joint Surg 2008;90B:349-355.
2. Waters PM. Update on management of pediatric brachial
plexus palsy. J Pediatr Orthop 2005;25:116-126.

Figure 1 Figure 2
173. A 21 year-old right hand dominant man injures his
right hand in a martial arts competition three weeks
prior to presenting for treatment. Examination reveals
significant ulnar sided hand swelling and tenderness
at the base of the fifth metacarpal. Digital flexion is
nearly full with no malrotation. Figures 1 and 2 are
the plain radiographs and CT images are shown in
Figures 3 and 4. The most appropriate treatment is:
A. Cast immobilization
B. Hand therapy for edema control and range of
motion exercises
C. Application of an external fixator
D. Closed reduction percutaneous pin fixation Figure 3 Figure 4
E. Open reduction internal fixation
Preferred Response: E
Discussion: This fifth metacarpal base fracture is displaced,
comminuted, and impacted. It is also associated with dorsal
subluxation of the metacarpal on the hamate. The CT scan
confirms the impaction of a sizable portion of the articular
surface of the metacarpal base. In addition, the injury is
nearly three weeks old and is not likely to reduce by closed
means. Open reduction is therefore required. This technique
would enable a more accurate restoration of the articular
surface. If the patient had presented earlier, a closed
reduction percutaneous pinning may have been successful,
but may not have reduced the impacted articular segment.
The injury usually results from a longitudinally-directed
force along the axis of the fifth metacarpal.

2010 Self-Assessment Examination | 75


174. A 21 year-old right hand dominant man presents 175. After being outside for less than 24 hours, a 34
three months following an injury to his right ring year-old mountaineer presents with severe frostbite
finger that occurred while he was playing football. involving both hands. After rapid rewarming,
He reports persistent pain despite intermittent a technetium 99 bone scan is obtained that
splinting. Examination reveals DIP joint swelling demonstrates no digital blood flow to multiple digits
with active distal joint motion 15/55, with full range bilaterally. What intervention should now be utilized?
of motion of the PIP and MP joints. Radiographs
A. Daily dry dressing changes, elevation and warm soaks
are shown in Figures 1 and 2. The most appropriate
treatment at this time is: B. Surgical debridement
C. High dose intravenous corticosteroid therapy
A. Full time DIP static extension splinting for 6 weeks
D. Intravenous tissue plasminogen activator therapy
B. Supervised hand therapy including modalities
E. Systemic antibiotic therapy
C. Closed reduction percutaneous pin fixation
D. Open reduction internal fixation Preferred Response: D
E. DIP joint arthrodesis Discussion: The management of acute severe frostbite
should consist of several measures: restoration of core
Preferred Response: D
body temperature, rapid rewarming of the frozen extremity
Discussion: The radiographs demonstrate dorsal using a 40-44° C whirlpool bath along with tetanus
subluxation of the DIP joint without an obvious fracture or prophylaxis, topical aloe vera Q6h, and oral ibufrofen
arthrosis. The patient reports ongoing pain which is not 200mg QID. Following rewarming, a technetium 99 bone
likely to respond to further immobilization, or hand therapy, scan should be obtained, and if no digital blood flow is
given the persistent subluxation. Given the length of time detected, intravenous TPA is recommended. Surface blebs
since the injury, a closed reduction and pinning is not likely should be left intact and early surgical debridement is
to reduce the joint. An arthrodesis is not indicated at this contraindicated. An open dressing technique with topical
time, due to the patient’s age, apparent preservation of the aloe is recommended and occlusive dressings should
joint space, as well as his preserved range of motion. An be avoided. Antibiotics are not recommended unless a
open reduction and internal fixation is most appropriate, secondary infection ensues.
possibly in conjunction with a volar plate arthroplasty.
References:
References: 1. Bruen KJ, Ballard JR, Morris SE, Cochran A, Edelman LS,
1. Rettig ME, Dassa G, Raskin KB. Volar plate arthroplasty of the Saffle JR. Reduction of the incidence of amputation in frostbite
distal interphalangeal joint. J Hand Surg 2001;26A:940-944. injury with thrombolytic therapy. Arch Surg 2007;142:546-551.
2. Horiuchi Y, Itoh Y, Sasaki T, Tasaki K, Iijima K, Uchinishi K. 2. Bruen KJ, Gowski WF. Treatment of digital frostbite: Current
Dorsal dislocation of the DIP joint with fracture of the volar base concepts. J Hand Surg 2009;34A:553-554.
of the distal phalanx. J Hand Surg 1989;14B:177-182. 3. Murphy JV, Banwell PE, Roberts AH, McGrouther DA. Frostbite:
Pathogenesis and treatment. J Trauma 2000;48:171-178.
4. Twomey JA, Peltier GL, Zera RT. An open-label study to
evaluate the safety and efficacy of tissue plasminogen activator
in treatment of severe frostbite. J Trauma 2005;59:1350-1354.

Figure 1 Figure 2

76 | American Society for Surgery of the Hand


176. A 51 year-old left hand dominant male laborer with 177. A 19 year-old right hand dominant woman who fell
no history of injury presents with a progressive two 15' out of a tree presents with right elbow pain and
year history of left elbow pain and stiffness. Physical swelling. Physical examination demonstrates significant
examination reveals painful active and passive global right elbow swelling with active motion from
range of motion from 50 to 100° of flexion, with 40 to 80° of flexion, pronation and supination 30/30.
near full rotation. X-rays demonstrate significant X-rays are shown in Figures 1, 2 and 3. The most
ulnohumeral arthritis with significant osteophyte appropriate management should consist of:
formation. The most appropriate management at
A. A sling and immediate range of motion exercises
this point is:
B. Cast immobilization for four weeks followed by
A. Physical therapy with dynamic splinting supervised therapy
B. Resection arthroplasty C. Radial head implant arthroplasty
C. Interposition arthroplasty D. Open reduction and internal fixation of the olecranon
D. Unlinked total elbow arthroplasty E. Open treatment of the radial head fracture and
E. Semiconstrained total elbow arthroplasty triceps repair
Preferred Response: C Preferred Response: E
Discussion: This patient presents with primary Discussion: The radiographs demonstrate a displaced
osteoarthritis of his dominant elbow. Given radiographs radial head fracture and triceps avulsion fracture off of the
that demonstrate significant osteoarthritis, along with his olecranon. Clearly surgery is indicated to repair the triceps
markedly restricted active and passive range-of-motion, and radial head. Given the patient’s age, an ORIF of the
physical therapy is not likely to improve his pain or his radial head is preferable to implant arthroplasty, if at all
range-of-motion for any significant period of time. Because possible. This injury combination, though rare, has been
of his age and occupation, TEA is contraindicated. A reported by Levy et al. This individual underwent a triceps
resection arthroplasty is rarely indicated, except in the repair and ORIF of the radial head with plate fixation utilizing
setting of uncontrollable infection with inadequate bone a posterior skin incision (Figures 4 and 5).
stock. Two recent studies by Morrey and Stanley report
Reference:
favorable long-term results of ulnohumeral arthroplasty
1. Levy M, Fishel RE, Stern GM. Triceps tendon avulsion with
in this age group, with improved range of motion and
or without fracture of the radial head: A rare injury? J Trauma
decreased pain. 1978;18:677-679.
References:
1. Artuna SA, Morrey BF, Adams RA. Ulnohumeral arthroplasty for
primary degenerative arthritis of the elbow: Long-term outcome
and complications. J Bone Joint Surg 2002;84A:2168-2173.
2. Larson AN, Morrey BF. Interposition arthroplasty with an
Achilles tendon allograft as a salvage procedure for the elbow.
J Bone Joint Surg 2008;90A:2714-2723.
3. Phillips NJ, Ali A, Stanley D. Treatment of primary degenerative
arthritis of the elbow by ulnohumeral arthroplasty: A long-term Figure 1 Figure 2
follow-up. J Bone Joint Surg 2003;85B:347-350.

Figure 3 Figure 4

Figure 5
2010 Self-Assessment Examination | 77
178. A 58 year-old right hand dominant man with long Discussion: Digital vasospasm is seen in patients with
standing diabetes mellitus presents with a chief autoimmune diseases such as scleroderma, and in
complaint of right hand weakness for two years patients with diffuse atherosclerosis. A recent study by
and difficulty turning a key in a tight lock. Physical Hartzell et al, published in the Journal of Hand Surgery,
examination demonstrates advanced intrinsic atrophy, demonstrated that the results of sympathectomy are
moderate clawing and a sensory deficit in the ulnar considerably better in the autoimmune group. Despite
nerve distribution. NCV/EMG study demonstrates some relief in both groups, amputation was eventually
absent ulnar nerve function. The treatment most required in 26% of the autoimmune patients and 59% of
likely to improve activities of daily living is: the atherosclerotic patients.
A. Occupational therapy References:
B. Brand tendon transfer (ECRL to lateral bands) 1. Hartzell TL, Makhni EC, Sampson C. Long-term results of
periarterial sympathectomy. J Hand Surg 2009;34A:1454-60.
C. ECRB adductorplasty
2. Ruch DS, Holden M, Smith BP, Smith TL, Koman LA.
D. In situ ulnar nerve decompression Periarterial sympathectomy in scleroderma patients:
E. Submuscular ulnar nerve transposition intermediate-term follow up. J Hand Surg 2002;27A:258-264.

Preferred Response: C
Discussion: This patient presents with unimprovable ulnar 180. A 35 year-old executive presents complaining of
nerve dysfunction secondary to cubital tunnel syndrome in painless elbow stiffness three months following a fall
the setting of long-standing diabetes mellitus. It is highly resulting in a posterolateral elbow dislocation and a
unlikely that either occupational therapy or an ulnar nerve nondisplaced radial head fracture. Initial treatment
decompression would be beneficial, given the advanced consisted of a closed reduction and splinting for
atrophy noted. The Brand transfer does improve power one week, followed by supervised physical therapy.
for gross grip, but does not address power pinch—this is Physical examination reveals elbow motion from
the patient’s chief complaint. Power pinch can be reliably 50-120° of flexion with near full forearm rotation.
improved by using a radial wrist extensor lengthened by a X-rays demonstrate a concentric reduction,
palmaris longus graft to the thumb adductor. If the thumb preserved joint spaces and minimal heterotopic
MP joint is markedly unstable, an MP arthrodesis may be ossification. Recommended treatment is:
required in addition to the transfer.
A. Static progressive splinting
References: B. Elbow manipulation under anesthesia
1. Smith RJ. Extensor carpi radialis brevis tendon transfer C. Arthroscopic elbow release
for thumb adduction–a study of power pinch. J Hand Surg
1983;8:4-15. D. Open release using a lateral approach
2. Mondelli M, Aretini A, Rossi. Ulnar neuropathy at the elbow in E. Open release and radial head implant arthroplasty
diabetes. Am J Phys Med Rehabil 2009;88:278-85.
3. Hastings H, Davidson S. Tendon transfers for ulnar nerve palsy. Preferred Response: A
Evaluation of results and practical treatment considerations. Discussion: This patient presents with significant elbow
Hand Clin 1988;4:167-78.
stiffness following an uncomplicated elbow dislocation
in association with a nondisplaced radial head fracture.
A manipulation of the elbow under anesthesia is rarely, if
179. The long term results of periarterial sympathectomy ever, indicated. Though surgery may ultimately be required
for the treatment of digital vasospasm associated in this case, static progressive splinting may obviate the
with scleroderma compared with atherosclerotic need for operative intervention. Static progressive splinting
disease demonstrate: can improve passive range of motion even in the setting
A. Amputation is rarely necessary in both groups of “hard end-feel joints.” Conceptually, static progressive
splinting uses inelastic components to apply torque to a
B. Better results for scleroderma
joint, in order to statically position it as close to end range
C. Worse results for scleroderma as possible; thus, increasing passive range-of-motion.
D. Complete permanent relief in both groups
References:
E. Temporary relief only in atherosclerotic disease 1. Doornberg JN, Ring D, Jupiter JB. Static progressive
Preferred Response: B splinting for posttraumatic elbow stiffness. J Orthop Trauma
2006;20(6):400-404.
2. Schultz-Johnson, K. Static progressive splinting. J Hand Ther
2002;15:163-178.

78 | American Society for Surgery of the Hand


181. Which of the following is an example of an 182. A 25 year-old male presents having had a ring finger
appropriate relationship between the orthopaedic proximal phalanx fracture 8 weeks ago after minor
surgeon consultant and industry? trauma. The ring finger proximal phalanx base is
clinically enlarged, but non-tender. Radiographs
A. Reimbursement for the physician and his spouse to
are shown. Based on available evidence, the
attend a product development meeting
demonstrated advantage of routine bone grafting
B. Receiving a consultant fee for attending an (vs. curettage alone) the proximal phalanx is:
educational meeting
A. Decreased infections
C. Receiving a consultant fee based on volume of
implants used B. Decreased postoperative fractures
D. Receiving a consultant fee of $5000 for attending a full C. Decreased tumor recurrences
day product development meeting D. Quicker return to activities of daily living
E. Receiving a yearly stipend of $50,000 and be listed as E. No advantage
a consultant
Preferred Response: E
Preferred Response: D
Discussion: Multiple studies suggest that a routine
Discussion: The relationship between physicians and enchondroma can be treated with curettage, without
industry is currently under tremendous scrutiny by the cancellous autograft. No postoperative fractures were
Department of Justice and the public in general. In 2003, identified in the series of hand enchondromas and a series
the Advanced Medical Technology Association (AdvaMed) of foot enchondromas. Schaller and Baer compared 8
adopted a code of ethics on interactions with health care patients treated with bone graft, and 8 without, and were
professionals, which included sections on consulting unable to demonstrate radiographic differences in bone
arrangements and gifts to physicians. The group outlined density measured years later.
clear parameters for appropriate interactions between
physicians and corporations that would result in the References:
avoidance of conflict of interest. Consulting arrangements 1. Schaller P, Baer W. Operative treatment of enchondromas of
need bonafide, written agreements with specific services the hand: is cancellous bone grafting necessary? Scand J Plast
Reconstr Surg Hand Surg 2009;43(5):279-85.
outlined, providing compensation consistent with fair
market value for the service provided. The AAOS has similar 2. Goto T, Kawano H, Yamamoto A, Yokokura S, Iijima T, Motoi
T, Nakamura K. Simple curettage without bone grafting for
guidelines that outline appropriate rules of engagement. enchondromas of the foot. Arch Orthop Trauma Surg 2004
It is appropriate for consultants to industry, that provide Jun;124(5):301-5. Epub 2004 Mar 11.
genuine services, to receive reasonable compensation for 3. Sekiya I, Matsui N, Otsuka T, Kobayashi M, Tsuchiya D.
their services. These arrangements should be established in The treatment of enchondromas in the hand by endoscopic
writing and include documentation of an actual need for the curettage without bone grafting. J Hand Surg 1997;22B:230-4.
service, proof that the service was provided, and evidence 4. Hasselgren G, Forssblad P, Törnvall A. Bone grafting
that physician reimbursement for services be equal to unnecessary in the treatment of enchondromas in the hand.
fair market value. Inappropriate relationships include J Hand Surg 1991;16A:139-42.
receiving a consultant fee for simply attending a meeting,
and receiving remuneration for using a particular implant.
It is not appropriate for the physician to be reimbursed for
the added travel expenses of his or her spouse, and thus,
answer A is not correct. Answer E is not correct, since the
duties are not defined and likely exceed fair market value.
References:
1. Advanced Medical Technology Association. AdvaMed Code of
Ethics. Available at: http://www.advamed.org/MemberPortal/
About/code/codeofethics/htm.
2. AAOS. Opinions on ethics and professionalism. Available at:
http://www.aaos.org/about/papers/ethics/1204eth.asp.
3. Glickel SZ. The Ethics of Expediency. J Hand Surg
2009;34A:799-807.
Figure 1

2010 Self-Assessment Examination | 79


183. The transverse retinacular ligament stabilzes the a flexor. Similarly, any transfer of the FDP ring would be a
extensor mechanism at the level of the: static transfer only because of the loss of innervation. The
results of end-to-side nerve grafting, while some excellent
A. DIP joint
functional results have been reported, have not been
B. PIP joint universally stellar. In this question, the common digital
C. MCP joint nerve to 4th web would be expected to be a sensory nerve,
D. Wrist except for autonomic motor fibers, and would be expected
to be a poor motor donor.
E. Proximal Phalanx
References:
Preferred Response: B
1. Battiston B, Lanzetta M. Reconstruction of high ulnar nerve
Discussion: The extensor mechanism has several lesions by distal double median to ulnar nerve transfer. J Hand
supporting bands and ligaments. The triangular ligament Surg 1999;24A:1185-91.
supports the conjoined lateral bands at the level of the DIP 2. Brown JM, Mackinnon SE. Nerve transfers in the forearm and
joint. At the PIP joint, the transverse retinacular ligaments hand. Hand Clin 2008;24(4):319-40.
are the important stabilizers of the extensor mechanism and
the sagittal bands support the extensor tendon at the level
of the MCP joint. The extensor tendons are also supported 185. The dorsal sensory branch of the ulnar nerve is at
by the junctura tendinae at the dorsal hand level, and the risk with exposures of the distal ulna. Which of the
extensor retinaculum supports the extensor tendon at following best describes its expected position? The
the wrist. dorsal sensory branch of the ulnar nerve branches
from the ulnar nerve 8.3 centimeters:
Reference:
1. Yu HL, Chase RA, Strauch B. Extensor Apparatus. In: Atlas of A. Distal to the olecranon, passing dorsal to the FCU,
Hand Anatomy and Clinical Implications. St Louis, Missouri: and dorsal to the ulnar styloid
2004:336-356.
B. Distal to the olecranon, passing dorsal to the FCU,
at the level of the ulnar styloid
C. Distal to the olecranon, passing volar to the FCU,
184. A 27 year-old male sustains a soft tissue injury to
and volar to the ulnar styloid
the medial arm, elbow, and forearm, including a
20 cm segmental defect in the ulnar nerve starting D. Proximal to the pisiform, passing volar to the FCU,
at the mid-brachium. He has complete loss of and dorsal to the ulnar styloid
ulnar innervated intrinsic muscles. Which of the E. Proximal to the pisiform, passing dorsal to the FCU,
following would be the best choice to restore some and volar to the ulnar styloid
intrinsic function?
Preferred Response: E
A. Anterior interosseous nerve branch to pronator
Discussion: The ulnar dorsal sensory nerve arises from the
quadratus transferred to ulnar motor nerve
ulnar nerve at approximately 1/4 of the ulnar length from the
B. Ulnar nerve reconstruction using bilateral sural distal end of the ulna, or 8.3 cm proximal to the pisiform. It
nerve grafts penetrates the fascia dorsal to the FCU to run superficial
C. FDS ring tendon transfer to A1 pulley to the FCU, passing volar to the ulnar styloid, then onto the
(Zancolli procedure) dorsal ulnar hand.
D. FDP ring two-tailed split tendon transfer to ring and References:
small radial lateral band 1. Doyle JR, Botte, MJ. Surgical anatomy of the hand and upper
E. Ulnar nerve motor branch end-to-side into median extremity. Philadelphia: Lippincott Williams & Wilkins, 2003:
nerve common digital nerve to 3rd web 206-7.
2. Mok D, Nikolis A, Harris PG. The cutaneous innervation of the
Preferred Response: A dorsal hand: Detailed anatomy with clinical implications.
J Hand Surg 2006;31A:565-574.
Discussion: Battiston and Lanzetta reported 6/7 patients
recovering M4 or M5 recovery after PQ branch to ulnar
motor branch. With this proximal an ulnar nerve injury, the
sural nerve grafts are unlikely to reinnervate the muscle
prior to the motor end plates degenerating, and this is
a long defect. FDS ring transfer would be a poor choice
given that the FDP to the ring and small have lost their
innervation in the trauma, leaving the ring finger without

80 | American Society for Surgery of the Hand


186. A one day-old baby exhibits signs of worsening 187. A 55 year-old male presents to your office complaining
edema of the right arm associated with decreased of painful progressive stiffness of his fingers. He
hand mobility. The child has an ulcerated skin lesion recalls no trauma or recent overuse. He was referred
on the dorsal forearm. Appropriate initial treatment to you by his nephrologist. His only other medical
should include: issue is failed back surgery, but a recent MRI with
gadolinium showed no surgically correctable lesion.
A. MR angiography followed by an endovascular procedure
On exam, you note that the skin on the back of his
B. Wet-to-dry dressings for the skin ulceration and hands is thickened and hyperpigmented in some
elevation of the extremity areas. You should advise this patient that:
C. Emergent surgical intervention and compartment releases
A. The condition is benign and self-limited with resolution
D. IV antibiotics and edema control modalities expected with 8-10 weeks
E. Surgical debridement of the ulcer with plan for staged B. Surgical treatment with joint capsulotomies and
skin graft tenolysis is recommended
Preferred Response: C C. There is no curative medical treatment for this
condition and treatment is primarily symptomatic
Discussion: Forearm compartment syndrome in the
neonate is not as uncommon as previously thought. The D. Treatment after a biopsy would include parenteral
presence of a bullous or ulcerative lesion on the dorsal antibiotic therapy for 6 weeks
forearm, wrist, and/or hand is the salient initial diagnostic E. Symptoms are likely to be limited to the hand and
finding. Early treatment in the face of evolving compartment fingers with little chance of systemic involvement
syndrome may salvage an otherwise compromised limb.
Preferred Response: C
Compartment pressure measurement in the neonate is
not routinely performed and no standards for acceptable Discussion: Nephrogenic systemic fibrosis is a debilitating
pressure gradients are available. Because the average fibrosing disorder that occurs in patients with underlying
diastolic pressure in a term newborn is 40 mm Hg or less, kidney disease, following exposure to gadolinium contrast
a very small increase in compartment pressure may impair agents. It is associated with fibrosis of skin and connective
muscle function. The decision to intervene surgically with tissue throughout the body. The skin hyperpigments,
compartment release is therefore primarily a clinical one. thickens, becomes hard and rigid which severely restricts
motion of joints. Patients will often present to a hand
Patterns of involvement can range from mild skin lesions
surgeon seeking help for the disabling contractures of
to a severe Volkmann’s with tissue loss. Predisposing
their hands and fingers. Systemic involvement is common
factors may include mechanical compression, amniotic
including lungs, heart, diaphragm, esophagus, and skeletal
band constriction, malposition of the extremity, forceps
muscle. There is no curative medical treatment at this
extraction, oligo/polyhydramnios, pre-eclampsia, excessive
time, although imatinib mesylate shows some potential in
maternal weight gain, and maternal use of neuroleptic and
preliminary studies. Symptomatic treatment is focused on
anxiolytic agents.
intensive occupational therapy and pain control. Deaths
While MR angiography may play a role in the overall have been reported as a result of this condition.
evaluation of this condition, initial emergent treatment should
References:
not be delayed while obtaining this exam. Wound care,
1. Kay J. Nephrogenic systemic fibrosis: a gadolinium-associated
antibiotics, and edema control techniques may also play
fibrosing disorder in patients with renal dysfunction. Ann
a secondary role in treatment but they are not considered Rheum Dis 2008 Dec; 67 Suppl 3: iii66-9.
definitive treatment in an evolving compartment syndrome 2. Saxena SK, Sharma M. Nephrogenic systemic fibrosis: an
and will not stop progressive damage from ischemia. emerging entity. Int Urol Nephrol 2008;40(3):715-24.
References: 3. Marckmann P, Skov L. Nephrogenic systemic fibrosis: clinical
picture and treatment. Radiol Clin North Am 2009;47(5):833-40.
1. Ragland R 3rd, Moukoko D. Forearm compartment syndrome
in the newborn: report of 24 cases. J Hand Surg .
2005;30A:997-1003.
2. Caouette-Laberge L, Bortoluzzi P. Neonatal Volkmann’s
ischemic contracture of the forearm: a report of five cases.
Plast Recontr Surg 1992;90(4):621-8.

2010 Self-Assessment Examination | 81


188. A 35 year-old male construction worker sustained a nail 190. When performing a total wrist arthrodesis for
puncture wound to the ulnar side of his palm 4 days osteoarthritis, which of the following joints should
ago. Examination reveals tenderness and swelling over always be fused:
the hypothenar area of the palm and pain with thumb
A. Scaphoid-capitate
range of motion. The diagnosis is most likely:
B. Capitate-hamate
A. Thenar space infection
C. Triquetrum-hamate
B. Horseshoe abscess
D. Lunate-triquetrum
C. Retained foreign body
E. Trapezium-first metacarpal
D. Partial FPL laceration
Preferred Response: A
E. Mid-palmar space infection
Discussion: Joints commonly included in a total wrist
Preferred Response: B
arthrodesis are the radius-scaphoid, radius-lunate,
Discussion: The thenar space extends from the third scaphoid-capitate, and lunate-capitate. Fusion of the
metacarpal to the thumb. A puncture wound in the capitate and third metacarpal base is controversial.
hypothenar area would be unlikely to involve this space Optional fusions depending on extent of disease include
initially. An infection in the ulnar bursa can track to the the capitate-hamate, hamate-triquetrum, lunate-triquetrum,
thenar bursa and cause a horseshoe abscess. scaphoid-trapezoid-trapezium, and the trapezoid-second
metacarpal base.
The examination doesn’t support a retained foreign body
or a tendon laceration given the mechanism and area of the References:
wound. The midpalmar space doesn’t communicate with 1. Weiss A-PC, Hastings H II. Wrist arthrodesis for traumatic
the ulnar bursa. conditions: a study of plate and local bone graft application.
J Hand Surg 1995;20A:50-56.
References: 2. Osteoarthritis. In: Trumble TE, Budoff JE, eds. Hand Surgery
1. Neviaser RJ. Acute Infections. In: Green DP, Hotchkiss RN, Update IV. Rosemont, IL: American Society for Surgery of the
Pederson WC, eds. Green’s Operative Hand Surgery. 4th ed. Hand 2007;652.
Philadelphia: Chuchill Livingstone, 1999:1033-1047.

191. Biological response modifying medications such


189. The following structures have been shown to as Etanercept(Enbrel), Infliximab(Remicade),
block reduction of a complex dislocation of the and Adalimumab(Humira) used in the treatment
metacarpal phalangeal joint of the thumb: of rheumatoid arthritis work by the following
A. Flexor pollicis longus and opponens pollicis mechanism of action:
B. Opponens pollicis and adductor pollicis A. Blocking of tumor necrosis factor (TNF)
C. Flexor pollicis longus and adductor pollicis B. Binding of cell messenger protein IL-1
D. Adductor pollicis and flexor pollicis brevis C. Blocking of T-cell activation
E. Adductor pollicis and extensor pollicis longus D. Antagonist of DNA synthesis
Preferred Response: D E. B-cell depletion

Discussion: The volar plate most commonly interferes Preferred Response: A


with reduction of a complex thumb MCP dislocation. The Discussion: Glucocorticoids have long been at the core of
sesamoids can also block reduction in association with medical treatment for RA and are commonly associated
incomplete tears of their attachments of the adductor with the development of osteoporosis and diabetes. Other
pollicis to the ulnar sesamoid and the flexor pollicis brevis to first line treatments include anti-inflammatories. DMARDS
the radial sesamoid. The attachments function as a tension such as gold, MTX, sulfasalizine have served as a second
force that prevents reduction. Adduction of the metacarpal line of treatment, but their onset of action is much slower
reduces this tension. than the newer treatments, and treatment is often required
References: for extended periods of time. Mechanisms of action include
1. Stener B. Hyperextension injures to metacarpal phalangeal blocking the inflammatory cascade and prostaglandins, as
joint of the thumb – rupture of ligaments, fracture of sesamoid well as DNA and RNA synthesis. Newer medication called
bones, rupture of flexor pollicis brevis, anatomical and clinical biological response modifying medications (BRM) include
study. ACTA Chir Scand 1963;125:275-83. Infliximab, Etanercept, Adalimumab, and Anakinra, among
2. Dislocations and Ligamentous Injuries of the Digit. In: Green’s others. These work by blocking enzymes in the immune
Operative Hand Surgery. Online 5th ed. Part II-9. activation cascade, such as TNF, IL-1, B-cells and T-cells.
They have a much more targeted mechanism of action and
have been shown to have a powerful effect on arresting
82 | American Society for Surgery of the Hand
disease progression. 193. A 55 year-old man sustains a fracture of his
dominant right distal radius and ulnar styloid
References:
process. Pre-operative radiographs demonstrate
1. Doan T, Massarotti E. Rheumatoid Arthritis: An overview
displacement of the radius with intra-articular and
of new and emerging therapies. J Clin Pharmacology 2005
Jul;45(7):751-62. extra-articular comminution. The ulnar fracture is
2. Rheumatoid Arthritis. In: Trumble TE, Budoff JE, eds. Hand not comminuted, is through the base of the styloid
Surgery Update IV. Rosemont, IL: American Society for Surgery process and is displaced 3mm. After satisfactory
of the Hand, 2007;83. operative open reduction and internal fixation of the
radius, the distal radial ulnar joint is stable to stress
testing but the ulnar styloid remains displaced
192. The expected course of the vessel on which the 2mm. Recommended treatment for the ulnar styloid
ulnar artery distal perforator flap (Becker flap) is component of this injury is:
based arises from the ulnar artery: A. Excise the ulnar styloid fragment
A. Dorsal to the ulnar nerve, then penetrates the fascia B. No surgical treatment of the ulnar styloid fracture
dorsal to the ECU C. Open reduction and internal fixation of the ulnar styloid
B. Volar to the ulnar nerve, then penetrates the fascia D. Excise the ulnar styloid fragment and suture the
dorsal to the ECU triangular fibrocartilage to the ulna
C. Volar to the ulnar nerve, then penetrates the fascia E. Arthroscopic assisted reduction and percutaneous
volar to the FCU fixation of the ulnar styloid
D. Dorsal to the ulnar nerve, then penetrates the fascia
Preferred Response: B
volar to the FCU
E. Volar to the ulnar nerve, then penetrates the fascia Discussion: Potential complications of an untreated ulnar
dorsal to the FCU styloid base fracture include painful nonunion and instability
of the distal radioulnar joint, resulting from loss of integrity
Preferred Response: E of the styloid attachment of the triangular fibrocartilage.
Discussion: The Becker flap pedicle is based on a A recent study examined the effect of untreated styloid
perforator of the ulnar artery that was originally described fractures in patients undergoing internal fixation for an
as arising from the ulnar artery 2-5 cm proximal to the associated distal radius fracture. The authors found no
pisiform, then passing volar to the ulnar nerve, then passing difference in multiple outcome measures when compared to
dorsal to the FCU to penetrate the fascia and reach the a cohort of patients without styloid fractures. Prior studies
subcutaneous tissue. However, any significantly-sized have examined the potential instability associated with an
perforator of the ulnar or radial artery can probably be used ulnar styloid fracture and restoration of joint stability with
as the vascular supply for a pedicled regional forearm flap. internal fixation of styloid base fractures. However, these
studies have not demonstrated conclusively that internal
References: fixation improves outcome after this complex injury.
1. Page R, Chang J. Reconstruction of hand soft-tissue defects:
alternatives to the radial forearm fasciocutaneous flap. J Hand References:
Surg 2006;31A:847-56. 1. Greenberg J. Reconstruction of the distal ulna: instability,
2. Uygur F, Uygur M, Ulkür E, Sever C. Versatility of the reverse impaction, impingement, and arthrosis. J Hand Surg
dorsoulnar fasciocutaneous flap in coverage of hand defects: 2009;34A:351-356.
clinical experience with 36 cases. J Hand Surg 2. Kleinman WB. Stability of the distal radioulnar joint:
2009;34A:1327-33. biomechanics, pathophysiology, physical diagnosis, and
restoration of function. J Hand Surg 2007;32A:1086-1106.
3. Souer JS, Ring D, Matschke S, Audige L, Marent-Huber M,
Jupiter JB. Effect of an unrepaired fracture of the ulnar styloid
base on outcome after plate-and-screw fixation of a distal
radial fracture. J Bone Joint Surg 2009;91A:830-838.

2010 Self-Assessment Examination | 83


194. The terrible triad injury to the elbow is a traumatic Discussion: The patient has suffered a severe traumatic
fracture dislocation that includes a radial head open radio-carpal dislocation. Initial care requires open
fracture and a coronoid fracture. The most reduction, radio-carpal ligament repair, and stable fixation
commonly observed fracture pattern of the coronoid of an anatomic reduction. Despite open treatment, a high
in the terrible triad injury is fracture of the: percentage of patients will have recurrent instability often
manifested by recurrent ulnar translocation of the carpus.
A. Base of the coronoid
Resection or recession of the distal ulna will exacerbate
B. Tip of the coronoid instability and ligament reconstruction is unpredictable
C. Medial facet of the coronoid with a high risk of recurrent joint subluxation. A bony
D. Lateral facet of the coronoid surgical procedure to stabilize the wrist will have the most
predictable outcome. Fusion may require total or partial
E. Coronoid comprising about 50% of the height
wrist arthrodesis. Radiolunate fusion has been reported
Preferred Response: B with success.
Discussion: The most commonly observed coronoid References:
fracture pattern associated with the terrible triad is a small 1. Ilyas AM, Mudgal CS. Radiocarpal fracture-dislocations. J Am
tip fracture. These fractures are often comminuted, less Acad Orthop Surg 2008 Nov;16(11):647-55.
than 2 mm in height and represent an anterior capsular 2. Freeland AE, Ferguson CA, McCraney WO. Palmar radiocarpal
avulsion. The fracture is frequently too small to accept dislocation resulting in ulnar radiocarpal translocation and
internal fixation and is often stabilized with a suture lasso multidirectional instability. Orthopedics 2006 Jul;29(7):604-8.
technique. The presence of a small coronoid tip fracture
in the presence of an otherwise radiographically “ benign”
x-ray is an indication of significant elbow ligament injury and
potential instability.
References:
1. Mathew PK, Athwal GS, King GJW. Terrible triad injury of the
elbow: Current concepts. J Am Acad Orthop Surg
2009;17:137-151.
2. Ring D. Fractures of the coronoid process of the ulna. J Hand
Surg Am 2006;31:1679-1689.
3. O’Driscoll SW, Jupiter JB, Cohen M, Ring D, McKee MD.
Difficult elbow fractures: pearls and pitfalls. Inst Course
Lectures 2003;52:113-134.

195. 19 year-old male suffers an acute open injury to Figure 1


his non-dominant wrist (Figure 1 and 2). He has an
emergent operation to repair his wrist injury (Figure 3).
Pins are removed at 8 weeks post-injury and he is
placed in a brace and started on rehabilitation. At 6
months post-injury, he complains of wrist pain with
most activities and is unable to return to work. His
most recent radiograph is seen in Figure 4. The next
step in the patients care would likely involve:
A. Observation
B. Arthrodesis
C. Ligament reconstruction with tendon graft Figure 2 Figure 3 Figure 4
D. Distal ulna resection
E. Ulnar shortening
Preferred Response: B

84 | American Society for Surgery of the Hand


196. A 68 year-old retired engineer and avid golfer
presents to your office with a chief complaint of
difficulty bending his left small finger for 6 weeks.
His PCP refers him after failing a cortisone injection.
The patient only has a vague complaint of pain and
some mild tingling in his small finger. He is healthy,
except for non-insulin dependent diabetes treated
with oral medication. A clinical photograph is seen
in Figure 1 along with his radiographs in Figures 2
and 3. A CT scan is seen in Figure 4. The next step
in the care of this patient should include:
A. A second cortisone injection Figure 1

B. Trigger finger release surgery


C. Open reduction of the fracture with internal fixation
D. Fracture fragment excision and flexor
tendon reconstruction
E. Neurodiagnostic testing to determine the level of
nerve compression
Preferred Response: D
Discussion: The patient presents with a chronic rupture
of his FDP tendon to the small finger, due to attritional
rupture from a hamate hook non-union. The hamate hook
fracture may have occurred weeks, and even years, before
the tendon rupture. Golfers and players of racquet sports
are predisposed to hamate hook fracture. Osteosynthesis
Figure 2 Figure 3
of hamate hook fractures can be considered in fresh
fractures in elite athletes, otherwise hamate hook excision
usually results in a good outcome. Hamate hook fractures
should typically be treated with excision when nonunion is
established. Patients presenting with tendon rupture usually
involves the FDP to the small finger and treatment requires
hamate hook excision and flexor tendon reconstruction.
References:
1. Hartford JM, Murphy JM. Flexor digitorum profundus rupture of
the small finger secondary to nonunion of the hook of hamate: Figure 4
A case report. J Hand Surg 1996;21A:621-623.
2. Yamakazi H, Kato H, et al. Closed rupture of the flexor tendons
of the little finger secondary to non-union of fractures of the
hook of the hamate. J Bone Joint Surg 2006;31B:337-341.

2010 Self-Assessment Examination | 85


197. A 5 year-old boy with leukemia is in the hospital for 198. A 28 year-old professional skateboarder injures his
chemotherapy. The pediatric oncologist calls for a wrist and presents one week after a fall in the skate
hand surgery consult for a dark-colored lesion near an park. The patient was referred from the Emergency
IV site on the dorsum of the wrist (Figure 1). The child Department with a diagnosis of scaphoid fracture.
has no complaint of pain and is afebrile. His recent His wrist radiographs are seen in Figures 1 and 2. His
WBC is 850. A bedside scraping of the lesion showed neurovascular exam demonstrates normal median
nonseptate nonbranching hyphae under microscopy. and ulnar nerve function. Appropriate treatment for
The next appropriate step in management is: this injury is:
A. Observation, continue chemotherapy A. Short arm thumb spica cast and application of a
B. Topical antifungal agents on the lesion and correct bone stimulator
immunodeficiency B. Closed reduction and application of a long arm thumb
C. Systemic antifungal agent, correct immunodeficiency spica cast
and wide surgical excision C. Closed reduction, percutaneous pinning, and long
D. Apply splint and obtain radiographs to rule out arm cast
child abuse D. Percutaneous screw fixation of the scaphoid fracture
E. Correct the immunodeficiency. There is no need for and short arm cast
surgery in this case E. Open treatment of the wrist injury and short arm cast
Preferred Response: E
Preferred Response: C
Discussion: The patient presents with an unreduced
Discussion: This immunodeficient child has developed
greater arc perilunate injury. The variant in this case is
a case of mucormycosis. Mucormycosis occurs as an
a trans-scaphoid perilunate fracture dislocation of the
opportunistic infection in immunocompromised hosts. The
wrist. According to Herzberg, this injury is missed in about
infection often begins as a small blackish area near an IV
25% of cases and delayed presentation may occur as
Site. If unrecognized, the infection can invade quickly and
a result. Closed reduction should be performed in the
deeply making eradication difficult. The treatment should
acute setting, if possible, to reduce tension on the median
be aggressive and should include a wide surgical excision
nerve and realign the carpus. However, closed reduction
of the lesion, systemic antifungal agents and correction of
alone is not likely to produce a favorable result. Perilunate
immunodeficiency. Both fungal cultures and histopathology
injuries require stable fixation of an anatomic reduction to
should be studied at each debridement.
obtain a good long term result. Anatomic reduction of the
This child required 3 debridements, and the lesion resolved midcarpal joint, lunotriquetral joint and scaphoid is unlikely
after correction of the immunodeficiency. The wound after closed reduction; therefore, anatomic open reduction
required a skin graft for closure. should be performed as soon as possible. Fixation of
the scaphoid alone would result in neglect of the luno-
References:
triquetral dissociation and the probable development of
1. Jones NF, Shin EK, Eo S, Starzl TE. Succesful salvage of
VISI deformity. Therefore, fixation of both the scaphoid
mucormycosis infection of the forearm and osteomyelitis of the
ulna. Hand (NY) 2008 Dec;3(4):332-6. and lunotriquetral joint must be performed with an open
2. Arnaiz-Garcia ME, et al. Cutaneous mucormycosis: report of approach to obtain the best long term result.
five cases and review of the literature. J Plast Reconstr Aesthet References:
Surg 2009;62:434-441.
1. Redjal H, Rafijah G. Perilunate Inuries of the Wrist. In: Slutsky
3. Patel RP, Malaviya GN, Sugar AM. Chronic Infections. In: DJ, Osterman AL, eds. Fracture and Injuries of the Distal
Green DP, Hotchkiss RN, Pederson WC, Wolfe SW, eds. Radius and Carpus, The Cutting Edge. Philadelphia: Saunders,
Green’s Operative Hand Surgery. 5th ed. Philadelphia: Churchill 2008:425-435.
Livingstone/Elsevier, 2005:113-14.
2. Herzberg G, Comtet JJ, Amadio PC, et al. Perilunate
dislocations and fracture dislocations: a multicenter study.
J Hand Surg 1993;18A:768-779.

Figure 1

Figure 1 Figure 2

86 | American Society for Surgery of the Hand


199. In the medicolegal arena, the statement “within a References:
reasonable degree of medical certainty” implies 1. Mathew PK, Athwal GS, King GJW. Terrible triad injury of the
medical probability greater than: elbow: Current concepts. J Am Acad Orthop Surg
2009;17:137-151.
A. 25% 2. Ring D. Fractures of the coronoid process of the ulna. J Hand
B. 50% Surg Am 2006;31:1679-1689.
C. 75% 3. O’Driscoll SW, Jupiter JB, Cohen M, Ring D, McKee MD.
Difficult elbow fractures: pearls and pitfalls. Inst Course
D. 90% Lectures 2003;52:113-134.
E. 98%
Preferred Response: B
Discussion: To be “legally sufficient,” the physician’s
opinion must be based upon a “reasonable degree of
medical certainty.” This statement implies greater than
50% likelihood. In the case of causation determination (for
example), in order for a condition to be accepted as “work
related,” the physician must opine that there is more than a
50% probability that the condition arose from the injury in
question. “Possibility” implies less than 50% likelihood.
Figure 1 Figure 2
References:
1. Brigham CR, Babitsky S. Independent medical evaluations
and impairment ratings. In Occ Med: State of the art reviews
1998;13(2):325-343.
2. Kreisler L. With a reasonable degree of medical certainty.
J Occup Med 1972;5:313-314.

200. A 30 year-old physician crashes his mountain Figure 3 Figure 4


bike and injures his elbow. He presents to your
office 2 days after injury. His diagnostic images
are presented in Figures 1-5. The recommended
treatment for this injury is:
A. Open treatment with buttress plate via posterior or
medial approach
B. Immobilization for 3 weeks followed by protected
mobilization
C. Immediate protected range of motion of the elbow
D. Open treatment of coronoid fracture with suture
Figure 5
lasso technique
E. Radial head replacement
Preferred Response: A
Discussion: The patient has a coronoid fracture of the
medial facet. O’Driscoll classified coronoid fractures into
three types: tip, medial facet, and fractures at the base of
the coronoid. Fractures of the medial facet of the coronoid
are deceptive injuries and are easy to overlook. However,
if untreated, these fractures result in varus posteromedial
instability and a varus aligned elbow. The result is the rapid
development of elbow joint arthritis. This fracture should be
treated with a buttress plate and possible bone graft via a
medial approach. A posterior skin incision could be utilized
to access both side of the elbow if necessary.

2010 Self-Assessment Examination | 87


Notes

88 | American Society for Surgery of the Hand


6300 North River Road, Suite 600
Rosemont, IL 60018-4256
Phone: (847) 384-8300 | Fax: (847) 384-1435
E-mail: Info@assh.org | www.assh.org

Copyright © 2010

You might also like